Sunteți pe pagina 1din 119

MATH 2023 • Spring 2015-16 • Multivariable Calculus

Problem Set #0 • Dot and Cross Products (Review)

1. (F) Given three points in R3 :

A(1, 2, 3), B(4, 0, 5) and C ( x, 6, 4)

Determine the number of possible value(s) of x such that the triangle ABC has a right
angle.

−→ −→
Solution: ∠ A is a right-angle if and only if AB ⊥ AC, if and only if:
−→ −→
0 = AB · AC = h3, −2, 2i · h x − 1, 4, 1i
= 3x − 9

Hence, ∠ A is a right-angle when x = 3 .


−→ − →
Similarly, ∠ B is a right-angle if and only if BA ⊥ BC, if and only if:
−→ − →
0 = BA · BC = h−3, 2, −2i · h x − 4, 6, −1i
= 26 − 3x

∠ B is a right-angle when x = 26/3 .


−→ − →
Finally, ∠C is a right-angle if and only if CA ⊥ CB, if and only if:
−→ − →
0 = CA · CB = h1 − x, −4, −1i · h4 − x, −6, 1i
= x2 − 5x + 27

As the quadratic equation has ∆ = (−5)2 − 4(27) < 0, it has no real root.
To conclude, there are two possible values of x such that ABC has a right-angle.

2. (FF) Let u = 2i + j − 2k, v = i + 2j + 2k and w = u × v.


(a) Show that u, v and w are mutually orthogonal (i.e. u ⊥ v, v ⊥ w and w ⊥ u).

Solution: Clearly, u · v = (2)(1) + (1)(2) + (−2)(2) = 0, so u ⊥ v. By the


definition of cross product, u × v is a vector which is orthogonal to both u and
v. Therefore, we have w ⊥ u and w ⊥ v as well.

(b) Given any vector r = xi + yj + zk in R3 , show that:


r·u r·v r·w
r= 2
u+ 2
v+ w.
|u| |v| | w |2
[Hint: You may use the fact that since u, v and w are mutually orthogonal and
non-zero, the vector r can be expressed as a linear combination of u, v and w, i.e.

r = au + bv + cw.

Solve for the scalars a, b and c.]

Page 1
MATH 2023 Dot and Cross Products (Review) Problem Set #0

Solution: Let r = au + bv + cw. Taking the dot product with u on both sides,
we get:

r · u = ( au + bv + cw) · u
r · u = au · u + bv · u + cw · u
r · u = a | u |2 + b (0) + c (0) (since v ⊥ u and w ⊥ u)
r·u
a=
| u |2
Similarly, taking the dot product with v and w on both sides of r = au + bv + cw
will give
r·v r·w
b= 2
and c =
|v| | w |2
respectively. This shows the desired result.

(c) Express the vector i as a linear combination of u, v and w.

Solution: Note that the result in (b) applies to any arbitrary vector r = xi + yj +
zk including the vector i (for which we have x = 1, y = z = 0). Therefore,

i·u i·v i·w


i= 2
u+ 2
v+ w
|u| |v| | w |2
By straight-forward computations, we have:

i·u 2
2
=
|u| 9
i·v 1
2
=
|v| 9
w = u × v = 6i − 6j + 3k
i·w 6 2
2
= =
|w| 81 27

Therefore,
2 1 2
i= u + v + w.
9 9 27

Page 2
MATH 2023 Dot and Cross Products (Review) Problem Set #0

3. (F) The figure below shows two vectors a and b which span a parallelogram. The vectors
in blue and red represent the two diagonals of the parallelogram.

(a) Express the red and the blue vectors in terms of a and b.

Solution: Red vector is a − b; Blue vector is a + b.

(b) By considering the dot product, show that: |a| = |b| if and only if the diagonals of
the parallelogram are orthogonal to each other.

Solution: (=⇒) Given that |a| = |b|, we need to show the blue and the red
vectors are orthogonal:

(a − b) · (a + b) = a · a + a · b − b · a − b · b (expansion)
= | a |2 + a · b − a · b − | b |2
= | a |2 − | b |2
=0 (given |a| = |b|)

Therefore, the diagonals (which are represented by a − b and a + b) are orthog-


onal to each other.
(⇐=) Given that the diagonals a − b and a + b are orthogonal, we have:

(a − b) · (a + b) = 0
a·a+a·b−b·a−b·b = 0 (expansion)
2 2
|a| + a · b − a · b − |b| = 0
|a|2 − |b|2 = 0.

Therefore, we have |a|2 = |b|2 , and since lengths of vectors must be non-negative,
we get |a| = |b|.

4. (F) Let u = xi + yj + zk be a variable unit vector in R3 and v = i + 2j + 3k.


(a) Find x, y and z such that u · v is the maximum possible. Explain your answer.

Solution: u · v = |u| |v| cos θ = |v| cos θ since u is unit. As v is a fixed vector, the
value of u · v is completely determined by the angle θ between u and v. Since
cos θ is the maximum when θ = 0 (at which we have cos θ = 1), the dot product
u · v is the maximum possible when u is parallel to v. Therefore,
v i + 2j + 3k
u= = √ ,
|v| 14
and so x = √1 , y= √2 and z = √3 .
14 14 14

Page 3
MATH 2023 Dot and Cross Products (Review) Problem Set #0

(b) Find x, y and z such that |u × v| is the maximum possible. Explain your answer.

Solution: Since |u × v| = |u| |v| sin θ = |v| sin θ, it is the maximum when θ = π2
(at which sin θ = 1). Therefore, any unit vector u which is orthogonal to v will
make |u × v| achieve the maximum possible value. As u · v = x + 2y + 3z, the
set of ( x, y, z)’s such that |u × v| are those which satisfy:

x + 2y + 3z = 0 and x2 + y2 + z2 = 1.

5. (FF) Given two vectors a and b in R3 , prove the following:


(a) Cauchy-Schwarz’s Inequality: |a · b| ≤ |a| |b|

Solution:
|a · b| = |a| |b| |cos θ | ≤ |a| |b| .
Here θ is the angle between a and b. We have used the fact that cos θ ≤ 1.

(b) Triangle Inequality: |a + b| ≤ |a| + |b|

Solution: There is not much we can do with |a + b|. However, using the fact
that |u|2 = u · u for any vector u, we may consider:

| a + b |2 = ( a + b ) · ( a + b )
= a·a+a·b+b·a+b·b
= |a|2 + 2a · b + |b|2
≤ | a |2 + 2 | a · b | + | b |2 (since x ≤ | x | for any real x)
≤ | a |2 + 2 | a | | b | + | b |2 (from (a))
2
= (|a| + |b|)

As both |a + b| and |a| + |b| are positive, we conclude that:

|a + b| ≤ |a| + |b| .

(c) If a and b are orthogonal, show that |a + b|2 = |a|2 + |b|2 .

Solution: Proceed as in (b), the first three equalities give:

|a + b|2 = |a|2 + 2a · b + |b|2 .

If a and b are orthogonal, then a · b = 0 and so |a + b|2 = |a|2 + |b|2 .

Page 4
MATH 2023 Dot and Cross Products (Review) Problem Set #0

6. (F) Let A, B and C be the points ( a, 0, 0), (0, b, 0) and (0, 0, c) respectively in the three
dimensional space, and O be the origin (0, 0, 0). Denote [ ABC ] the area of the triangle
with vertices A, B and C (analogously for [OAB], [OBC ], etc.). Show that:

[ ABC ]2 = [OAB]2 + [OBC ]2 + [OCA]2 .

With the help of a diagram, explain why this result can be regarded as the three-dimensional
analogue of the Pythagoreas’ Theorem.

Solution: First we find [ ABC ] using cross product.


−→
AB = h0, b, 0i − h a, 0, 0i = h− a, b, 0i
−→
AC = h0, 0, ci − h a, 0, 0i = h− a, 0, ci
−→ −→
AB × AC = hbc, ac, abi
−→ −→ q
AB × AC = (bc)2 + ( ac)2 + ( ab)2

1 −→ −→ 1
q
[ ABC ] = AB × AC = (bc)2 + ( ac)2 + ( ab)2
2 2
1
[ ABC ]2 = (bc)2 + ( ac)2 + ( ab)2

4
The other triangles are right-angled ones whose bases and heights can be easily found
from the diagram.

ab
[OAB] =
2
ac
[OAC ] =
2
bc
[OBC ] =
2
Therefore,
 2  ac 2  2
2 2 2 ab bc
[OAB] + [OAC ] + [OBC ] = + +
2 2 2
1
(bc)2 + ( ac)2 + ( ab)2

=
4
= [ ABC ]2 ,

as desired.
[ ABC ] is analogous to the hypotenuse of a right-angled triangle in 2D, whiles [OAB],
[OAC ] and [OBC ] are analogous to the sides of the triangle. The 2D Pythagoreas’
Theorem asserts the the square of the hypotenuse is the sum of squares of the lengths
of the sides. Analogously, the squared area of the hypotenuse face ABC is the sum of
squares of the areas of the other three sides.

Page 5
MATH 2023 Dot and Cross Products (Review) Problem Set #0

7. (FF) Given three non-zero vectors u, v and w in R3 , provide a geometric explanation to


each of the following facts:
(a) u × u = 0

Solution: Since the “angle” between u and itself is 0, we have |u × u| = |u| |u| sin 0 =
0. The only vector with zero magnitude is the zero vector, so u × u = 0.

(b) (u × v) · u = (u × v) · v = 0

Solution: The cross product u × v is geometrically defined as a vector in R3


which is perpendicular to both u and v. Since the dot product between two
perpendicular vectors are zero, so:

(u × v) ⊥ u ⇒ (u × v) · u = 0
(u × v) ⊥ v ⇒ (u × v) · v = 0.

(c) (u × v) × w is a vector on the plane spanned by u and v.

Solution: Since (u × v) is perpendicular to both u and v, it is a normal vector


to the plane Π spanned by u and v. Now consider (u × v) × w. No matter what
w we pick, the product (u × v) × w must be a vector in R3 that is perpendicular
to (u × v). Recall that (u × v) is a normal vector to the plane Π. The vector
(u × v) × w being perpendicular to the normal vector (u × v) must lie on the plane
Π.

−→
8. (FFF) The diagram below shows a circle with radius r centered at O. Let a = OA,
−→ −→
b = OB and c = OC. The purpose of the problem is to use dot products to show that
the angle at the center of a circle is twice the corresponding angle at the circumference.
Precisely, with the notations in the diagram below, we want to show ∠ BOA = 2∠ BCA.
We will prove this by showing ϕ1 = 2θ1 , and ϕ2 = 2θ2 can be proved in a similar way.
Follow the steps structured below:

θ1
c θ2

O
b ϕ1
ϕ2 a
−c
B
A

Page 6
MATH 2023 Dot and Cross Products (Review) Problem Set #0

b·c
(a) Show that cos ϕ1 = − . Recall that r is the radius of the circle.
r2
Solution: According to the geometric definition of dot products (applied to vec-
tors b and −c), we have:

b · (−c) = |b| |−c| cos ϕ1 .

Since both b and −c represent the radii of the circle, their lengths are both r.
Therefore, by rearranging the above, we get:

b · (−c) −b · c b·c
cos ϕ1 = = =− 2 .
|b| |−c| r·r r

r2 − b · c
(b) Show that cos θ1 = .
|b − c| |c|
−→
Solution: Similarly, apply the geometric definition of dot product on vectors CO
−→ −→ −→
and CB. Note that CB = b − c and CO = −c, so:

(b − c) · (−c) = |b − c|| − c| cos θ1 = |b − c||c| cos θ1 .


Expanding the LHS: we have

(b − c) · (−c) = −b · c + c · c = −b · c + |c|2 = −b · c + r2 .

Therefore,
r2 − b · c = |b − c||c| cos θ1 ,
which yields the desired result after rearrangement.

(c) Showing that |b − c|2 = 2(r2 − b · c).

Solution: Use the fact that |u|2 = u · u:

| b − c |2 = ( b − c ) · ( b − c )
= b·b−b·c−c·b+c·c
= |b|2 − 2b · c + |c|2
= r2 − 2b · c + r2 = 2(r2 − b · c).

Page 7
MATH 2023 Dot and Cross Products (Review) Problem Set #0

r2 − b · c
(d) Using the result proved in the previous parts, show that cos2 θ1 = .
2r2
Solution:
(r 2 − b · c )2
cos2 θ1 = (from (b))
| b − c |2 | c |2
(r 2 − b · c )2
= (from (c))
2(r 2 − b · c ) r 2
r2 − b · c
= .
2r2

(e) Finally, find a relation between cos2 θ1 and cos ϕ1 , and conclude that ϕ1 = 2θ1 .
[Hint: Double angle formula for cos.]

Solution:
r2 − b · c b·c
 
2 1 1
cos θ1 = 2
= + − 2
2r 2 2 r
1 1
= + cos ϕ1 (from (a))
2 2
ϕ
= cos2 1 (double/half-angle formula)
2
ϕ1
Therefore, θ1 = 2 , as desired.

Page 8
MATH 2023 • Spring 2015-16 • Multivariable Calculus
Problem Set #1 • Lines, Planes and Curves
1. (F) Consider the two straight-lines:
L1 : r1 (t) = h1, 2, 3i + t h1, −1, −1i
L2 : r2 (t) = h2 + t, 3 − 3t, −2 + 3ti
(a) Show that L1 and L2 intersect each other. Find the coordinates of the intersection
point.

Solution: Note that r1 (t) represents the position of the “particle” travelling along
L1 at time t. Similarly for r2 (t). Therefore, even if the lines L1 and L2 intersect,
the two “particles” may not reach the the intersection point at the same time t.
To find the intersection point, we need to find s and t such that:

r1 ( t ) = r2 ( s )
h1 + t, 2 − t, 3 − ti = h2 + s, 3 − 3s, −2 + 3si

which is equivalent to the system:

1+t = 2+s
2 − t = 3 − 3s
3 − t = −2 + 3s

Solving the system, we get t = 2 and s = 1.

r1 (t = 2) = h3, 0, 1i
r2 (s = 1) = h3, 0, 1i

Therefore, the intersection point is (3, 0, 1) .

(b) Find an equation of the plane containing both L1 and L2 .

Solution: L1 is parallel to the vector h1, −1, −1i. For L2 , the parametrization can
be rewritten as:
r2 (t) = h2, 3, −2i + th1, −3, 3i
Therefore, L2 is parallel to the vector h1, −3, 3i.
The required plane contains both L1 and L2 , hence is parallel to both h1, −1, −1i
and h1, −3, 3i. The normal vector of the plane is, therefore, can be taken to be the
cross-product of these two vectors:

h1, −1, −1i × h1, −3, 3i = h−6, −4, −2i = −2h3, 2, 1i

For simplicity, we take n = h3, 2, 1i which is also a normal vector to the plane.
From (a), the plane contains the point (3, 0, 1). By substituting h A, B, C i = h3, 2, 1i
and ( x0 , y0 , z0 ) = (3, 0, 1), we find the equation of the plane is given by:

3x + 2y + z = 3(3) + 2(0) + 1(1)

After simplification, we get 3x + 2y + z = 10 .

Page 1
MATH 2023 Lines, Planes and Curves Problem Set #1

2. (F) Consider the following four points in three-dimensional space:


A(0, 2, −1), B(4, 0, −1), C (7, −3, 0) and D ( 31 , 16 , 91 )
Determine whether or not these four points are coplanar (i.e. contained in a single plane).

Solution: First we find an equation of the plane containing A, B and C. Then, we will
substitute the coordinates of D into the equation to see whether D lies on that plane.
The two “ingredients” of finding the equation of a plane are (i) a given point on the
plane; and (ii) a normal vector to the plane. In order to find the normal vector the
−→ −→
plane through A, B and C, we take the cross product of AB and AC.

−→
AB = h4, 0, −1i − h0, 2, −1i = h4, −2, 0i
−→
AC = h7, −3, 0i − h0, 2, −1i = h7, −5, 1i
−→ −→
Taking the cross product: AB × AC = h−2, −4, −6i. Any non-zero vector parallel to
this cross product is a normal vector to the plane. For simplicity, we can take:

n = h1, 2, 3i .

Take A(0, 2, −1) to be the given point P0 , then the equation of the plane through A, B
and C is given by:
1x + 2y + 3z = 1(0) + 2(2) + 3(−1)
| {z }
( x0 ,y0 ,z0 )=(0,2,−1) and n=h1,2,3i

After simplification: x + 2y + 3z = 1.
Substitute D ( 31 , 16 , 91 ) into the equation x + 2y + 3z = 1, we see:
   
1 1 1
LHS = + 2 +3 = 1 = RHS.
3 6 11 Practice
Chapter 9 Exercises 645

anes 3x + 6z = 1 and 2x + 2y - Therefore Da line.


z = 3 intersect in lies on this
a. plane + 3kd #so
s2i - 3j and ssx the
+ 2di four 1dj + zkdA,
+ s y -points = 0B, C and D are coplanar.
w that the planes are orthogonal. b. x = 3 - t, y = -11t, z = 2 - 3t
d equations for the line of intersection. c. sx + 2d + 11s y - 1d = 3z
n equation for the plane3. that(F ) Athrough
passes parallelogram
the point in R 3 - 3jvertices:
d. s2ihas + 3kd * ssx + 2di + s y - 1dj + zkd = 0
) parallel to u = 2i + 3j + k and v = i - j + 2k. e. s2i - j + 3kd * s -3i + kd # ssx + 2di + s y - 1dj + zkd
2i - 4j + k related in any special way to the plane A (2, −1,=40), B(1, 0, −1), C (1, 2, 3), D ( x0 , y0 , z0 )
= 5 ? Give reasons for your answer. 62. The parallelogram shown here has vertices at As2, -1, 4d,
uation n # P0 P = 0 represents the
1 asplane
shownthroughin
P0the figure below.
normal Answer
Bs1, 0, -1d, Cs1, 2, 3dthe following
, and D. Find questions:
What set does the inequality n # P0 P 7 0 represent?
1

e distance from the point P(1, 4, 0) to the plane through


z
0), Bs2, 0, -1d, and Cs2, -1, 0d .
D
he distance from the point (2, 2, 3) to the plane
y + 5z = 0 .
vector parallel to the plane 2x - y - z = 4 and orthogo-
+ j + k.
unit vector orthogonal to A in the plane of B and C if A(2, –1, 4)
i - j + k, B = i + 2j + k, and C = i + j - 2k.
C(1, 2, 3)
vector of magnitude 2 parallel to the line of intersection of
nes x + 2y + z - 1 = 0 and x - y + 2z + 7 = 0 .
e point in which the line through the origin perpendicular
y
plane 2x - y - z = 4 meets the plane 3x - 5y + 2z = 6 .
e point in which the line through P(3, 2, 1) normal to the B(1, 0, –1)
x
x - y + 2z = -2 meets the plane.
angle does the line of intersection of the planes a. the coordinates of D,
- z = 0 and x + y + 2z = 0 make with the positive b. the cosine of the interior angle at B,
1 1
c. the vector projectionPage 2 BC ,
of BA onto
e d. the area of the parallelogram,
e. an equation for the plane of the parallelogram,
L: x = 3 + 2t, y = 2t, z = t
f. the areas of the orthogonal projections of the parallelogram
cts the plane x + 3y - z = - 4 in a point P. Find the co- on the three coordinate planes.
MATH 2023 Lines, Planes and Curves Problem Set #1

(a) Find the coordinates of D.

Solution: Since ABCD is a parallelogram, we must have:


−→ −→
AB = DC

h−1, 1, −5i = h1 − x0 , 2 − y0 , 3 − z0 i
Solving the equation, we get: ( x0 , y0 , z0 ) = (2, 1, 8) .

(b) Find the area of the parallelogram ABCD.

Solution: −→ −→
Area of ABCD = AB × AD

−→ −→
AB × AD = h−1, 1, −5i × h0, 2, 4i
= h14, 4, −2i
−→ −→ √
AB × AD = 216.

(c) Find an equation of the plane containing the parallelogram ABCD.

−→
Solution: Similar to previous “Equation of planes” problems. Take n = AB ×
−→
AD. Answer: 7x + 2y − z = 8

(d) Project the parallelogram ABCD orthogonally onto the plane z = −1. Find the
coordinates the projection of each vertices, then find the area of the projected parallel-
ogram.

Solution: Such a projection will preserve the x- and y-coordinates, so the projec-
tion of each vertices are:

A0 (2, −1, −1), B0 (1, 0, −1), C 0 (1, 2, −1) and D 0 (2, 1, −1).

−−→ −−→
Area of A0 B0 C 0 D 0 = A0 B0 × A0 D 0

= |h−1, 1, 0i × h0, 2, 0i|


= |h0, 0, −2i| = 2.

Page 3
MATH 2023 Lines, Planes and Curves Problem Set #1

4. (F) Consider a particle whose path is represented by:


  p
r(t) = ln(t2 + 1) i + tan−1 t j + t2 + 1k


Find the velocity, speed and acceleration of the particle at t = 0.

Solution: By straight-forward computations (omitted), we can get:

2t 1 t
r0 (t) = 2
i+ 2
j+ √ k
1+t 1+t 2
t +1
s
0
r (t) = 4t2 1 t2
+ +
(1 + t2 )2 (1 + t2 )2 t2 + 1

t4 + 5t2 + 1
=
1 + t2
d 0
r00 (t) = r (t)
dt
2(1 − t2 ) 2t 1
= 2 2
i− 2 2
j+ k
(1 + t ) (1 + t ) (1 + t2 )3/2

At t = 0, we have:

r 0 (0) = j
0
r (0) = 1
r00 (0) = 2i + k

5. (FF) Consider a plane through the point P0 ( x0 , y0 , z0 ) with normal vector n = h A, B, C i.


Prove that the perpendicular distance d from a given point Q( x1 , y1 , z1 ) to the plane is
given by: −−→
P0 Q · n

Ax1 + By1 + Cz1 − D
d= = √
|n| A2 + B2 + C 2
where D = Ax0 + By0 + Cz0 .

Solution: Suppose R is the projection of the point Q onto the given plane, then we
−→

have d = QR , which is what we need to find. Consider the triangle P0 QR and let θ
be the angle ∠ P0 QR, then we have:
−→ −−→
d = QR = QP0 cos θ.

−−→
Since θ is also the angle between P0 Q and n, we can deduce:
−−→ −−→ 1 −−→ 1
d = QP0 cos θ = QP0 |n| cos θ · = QP0 · n ·

| {z } | n | |n|
dot product

as desired. The second equality follows from plugging in n = h A, B, C i, P0 ( x0 , y0 , z0 )


and Q( x1 , y1 , z1 ).

Page 4
MATH 2023 Lines, Planes and Curves Problem Set #1

6. (F) Suppose r(t) represents the path of a particle traveling on a sphere centered at the
origin. Show that the position vector r(t) and the velocity r0 (t) are orthogonal to each
other at any time.

Solution: Since r(t) travels on a sphere centered at the origin, we have |r(t)| = C for
some constant C. Since dtd |r(t)| is difficult to compute, we take the square on both
sides:

|r(t)|2 = C2
r(t) · r(t) = C2
d d
(r(t) · r(t)) = C2 = 0
dt dt
r0 (t) · r(t) + r(t) · r0 (t) = 0 (product rule)
0
2r(t) · r (t) = 0
r(t) · r0 (t) = 0.

Therefore, r(t) and r0 (t) are orthogonal at any time t.

7. (FF) Suppose that the path of a particle at time t is given by r(t) and the force exerted
on the particle at time t is F(t). By Newton’s Second Law, F(t) and r(t) are related by:

F(t) = mr00 (t),

where m is the mass of the particle. The angular momentum L(t) about the origin of the
particle at time t is defined to be:

L(t) := r(t) × mr0 (t)

(a) Show that


d
L ( t ) = r ( t ) × F ( t ).
dt

Solution: Use the product rule:

d d
r(t) × mr0 (t)

L=
dt dt
= r0 (t) × mr0 (t) +r(t) × mr00 (t)
| {z }
r0 k mr0

= 0 + r(t) × F(t) (Newton’s Second Law)


= r(t) × F(t)

Page 5
MATH 2023 Lines, Planes and Curves Problem Set #1

(b) When L(t) is a constant vector, we say that the angular momentum is conserved.
According to the result in (a), under what condition on r(t) and F(t) will the angular
momentum be conserved? Also, give one example in physics that this condition is
satisfied.

Solution: In order for L(t) to be conserved, we need dL dt = 0. According to (a), it


happens when r(t) × F(t) = 0, or equivalently, r(t) is parallel to F(t) at any t.
There are many situations in physics that F is parallel to r. For instance, the
gravitational force exerted by the Sun on the Earth is a clear example: Pick the
origin to be the center of the Sun, the gravitational force field due to the Sun is
radially symmetric, in a sense that the gravity F due to the Sun on the Earth is
always in the opposition direction of the position vector r. Precisely, physicists
assert that the gravitational force satisfies the inverse-square law:

GMm r
F=−
| r |2 | r |
where G is a constant, M is the mass of the Sun and m is the mass of the Earth.
In this case, r × F = 0 and so L is conserved. (Further remark: from class, we
proved that if L is conserved, then r(t) travels on a single plane – that explains
why the Earth rotates around the Sun on a fixed plane.)

8. (FF) Consider two point particles with masses m1 and m2 , and their trajectories are r1 (t)
and r2 (t) respectively. Denote F(t) to be the force exerted on the m1 -particle by the m2 -
particle at time t. By Newton’s Third Law, the force exerted on the m2 -particle by the
m1 -particle at time t (i.e. the reverse force) is given by −F(t). Assume there are no other
forces exerted on any of these particles.
(a) Consider the following vector:

m1 r1 ( t ) + m2 r2 ( t )
C(t) := .
m1 + m2
In physics, this vector is pointing at the center of mass of the two particles. Show
that C00 (t) = 0 for any t using Newton’s Second and Third Laws.

Solution: By Newtow’s Laws, we have F(t) = m1 r100 (t) and −F(t) = m2 r200 (t).
Using these, we get:

m1 r10 (t) + m2 r20 (t)


C0 (t) =
m1 + m2
m r 00 ( t ) + m r00 ( t )
1 1 2 2
C00 (t) =
m1 + m2
F(t) + (−F(t))
= = 0.
m1 + m2

Page 6
MATH 2023 Lines, Planes and Curves Problem Set #1

(b) Hence, show that there exist two constant vectors r0 and v such that

m1 r1 ( t ) + m2 r2 ( t )
= r0 + tv.
m1 + m2
[Question: What is the physical significance of this result?]

Solution: Using (a), we get C00 (t) = 0. Since C00 (t) = d 0


dt C ( t ), we can deduce:

C 0 ( t ) = c1

for some constant vector c1 . By integration, we get:


ˆ
C(t) = c1 dt = c1 · t + c2

where c2 is any constant vector. The required result follows from relabelling the
constant vectors c1 and c2 by v and r0 respectively.
Since the parametric equation of the form r0 + tv represents a straight-line, this
results assert that the center of masses is travelling along a straight path with
constant velocity.

9. (F) For each of the following curves, first reparametrize it by arc-length and then compute
its curvature function κ (s):

(a) r1 (t) = ( R cos ωt) i + ( R sin ωt) j, 0≤t≤ ω .

Solution: Note that we say 0 ≤ t ≤ 2πω . It implicitly infers that ω > 0. However,
R can be negative! Yet we can ignore the case R = 0 (since it would give a “point”
rather than a curve).
First compute:

r10 (t) = (− Rω sin ωt) i + ( Rω cos ωt) j


0 q
r (t) = R2 ω 2 (sin2 ωt + cos2 ωt) = | R| ω
1
ˆ t ˆ t
0
s= r (τ ) dτ =
1 | R| ω dτ = | R| ωt
0 0
s
t=
| R| ω

An arc-length parametrization for r1 is given by:


     
s s
r1 (s) = R cos ω · i + R sin ω · j
| R| ω | R| ω
   
s s
= R cos i + R sin j
| R| | R|

When t = 0, s = | R| ωt = 0. When t = 2πω , s = | R| ω ·



ω = 2π | R|. Therefore, the
range for the parameter s is 0 ≤ s ≤ 2π | R|.

Page 7
MATH 2023 Lines, Planes and Curves Problem Set #1

To compute its curvature, we differentiate r1 (s) twice with respect to s:


   
0 R s R s
r1 ( s ) = − sin i+ cos j
| R| | R| | R| | R|
! !
00 R s R s
r1 (s) = − 2 cos i + − 2 sin j
| R| | R| | R| | R|

00 R 1
κ 1 ( s ) = r1 ( s ) = − 2 = .

| R| | R|

(b) r2 (t) = h1, 2, 3i + (ln t) h1, 0, −1i, 0<t<∞



Solution: Straight-forward computations show: |r20 (t)| = t2 .
ˆ t ˆ t√
0
r2 (τ ) dτ = 2
However, the integral dτ does not converge. Instead, we
0 0 τ
set: ˆ t ˆ t√
0 2 √
s= r2 (τ ) dτ = dτ = 2 ln t.
1 1 τ

Solving for t in terms of s, we get: ln t = √s . Therefore, the arc-length parametriza-


2
tion of r2 is given by:
s
r2 (s) = h1, 2, 3i + (ln t) h1, 0, −1i = h1, 2, 3i + √ h1, 0, −1i.
2

From s = 2 ln t, the range for s is given by −∞ < s < ∞.
It is clear that r200 (s) = 0, we have κ2 (s) = |r200 (s)| = 0.

(c) r3 (t) = (cos3 t) i + (sin3 t) j, 0≤t≤ π


2.

Solution: Straight-forward computations give:

r30 (t) = (−3 cos2 t sin t) i + (3 sin2 t cos t) j


0 p
r3 (t) = 9 cos4 t sin2 t + 9 sin4 t cos2 t
q
= 3 cos2 t sin2 t(cos2 t + sin2 t)
p
= 3 cos2 t sin2 t = 3 cos t sin t
ˆ t ˆ t
0
s= r (τ ) dτ = 3 cos τ sin τ dτ
0 0
ˆ τ =t τ =t
3 sin2 τ

3
= 3 sin τ d(sin τ ) = = sin2 t
τ =0 2 τ =0 2
q
Solving for t, we get t = sin−1 2s 3 , hence an arc-length parametrization of r3 is:

r !3 r !3
2s 2s 3
r3 ( s ) = cos sin−1 i+ sin sin−1 j, 0≤s≤ .
3 3 2

Page 8
MATH 2023 Lines, Planes and Curves Problem Set #1

Although the above express gives an arc-length parametrization of the curve, it


is very tedious to√differentiate, let alone finding its curvature. Using the fact that
cos(sin−1 ( x )) = 1 − x2 and sin(sin−1 ( x )) = x, we can simplify r3 (s) as:
v 3
u r !2 r !3
u 2s 2s
r3 ( s ) =  1 −  i+ j
t 
3 3
 3/2  3/2
2s 2s
= 1− i+ j
3 3

which is much easier to work with. By straight-forward differentiations, we get:

2s 1/2
   1/2
2s
r30 (s) = − 1− i+ j
3 3
2s −1/2
 
1
r300 (s) = 1− i + (6s)−1/2 j
3 3
1
κ3 (s) = r30 (s) = p

2s(3 − 2s)

Give an example of a path whose arc-length parametrization cannot be explicitly found


even with computer softwares.

Solution: While the arc-length (and its arc-length parametrization) of most curves
appeared in textbook problems can be found explicitly, it is not the case in general. If
one randomly writes down a curve r(t), there is more than 90% chance that you can’t
find the explicit expression of its arc-length, let alone its arc-length parametrization.
It is because |r0 (τ )| involves a square-root, and the integral of a square-root is very
difficult to compute even with computer softwares. One “notorious” example is the
ellipse: rp(t) = ( a cos t) i + (b sin t) j, 0 ≤ t ≤ 2π, where a 6= b, for which we have
0
|r (t)| = a2 sin2 t + b2 cos2 t. However, it is impossible to find an explicit expression
for: ˆ tp
s= a2 sin2 τ + b2 cos2 τ dτ.
0
Even if one can express it as a infinite series in t, the next step: solving t in terms of s
is impossible to carry out.

Page 9
MATH 2023 Lines, Planes and Curves Problem Set #1

10. (FF) Suppose √


1 2 2 2 3
r(t) = t i + t 2 j + tk
2 3
represents the path of a race-car climbing up a hill from (0, 0, 0) at t = 0. A truck, on the
other hand, drives slowly in unit speed from (0, 0, 0) at time t = 0 along the same path
and direction as the race-car. Find a parametrization which represents the path of the
truck.

Solution: The path of the truck is exactly the arc-length parametrization of r(t) such
that s = 0 when t = 0. It has been done in class. See the worksheet for Lecture #02.

11. (FFF) We define the curvature of a path by κ (s) = |r00 (s)| where r(s) is the arc-length
parametrization of the path. However, the arc-length parametrization r(s) is often difficult
to find explicitly. The purpose of this exercise is to derive an equivalent formula for the
curvature which does not require finding an arc-length parametrization.
Given a path r(t), we let r(s) be its arc-length parametrization so that s and t are related
by:
ˆ t
0
s= r (τ ) dτ.
0
(a) Show, using the chain rule, that:

ds
r0 (t) = r0 (s)
dt
2
d2 s

00 00 ds
r (t) = r (s) + r0 (s)
dt dt2

Solution: By the chain rule, we have:

dr dr ds ds
= = r0 (s) (1)
dt ds dt dt
d2 r
   
d dr d 0 ds
= = r (s) from (1)
dt2 dt dt dt dt
0
dr (s) ds 2
d s
= + r0 (s) 2 (2)
dt dt dt
By the chain rule again, we get:

dr0 (s) dr0 (s) ds ds


= = r00 (s)
dt ds dt dt
Substitute this back to (2) , we obtain:
2
d2 r d2 s

ds
= r00 (s) + r0 (s) (3)
dt2 dt dt2

Page 10
MATH 2023 Lines, Planes and Curves Problem Set #1

(b) Show that:


 3
0 00 ds
r (t) × r (t) = r0 (s) × r00 (s)
dt

Solution: Taking the cross product of (1) and (3) yields:


3 3
dr d2 r d2 s ds 0
 
ds 0 00 ds
× 2 = r (s) × r (s) + 2 r (s) × r0 (s) = r0 (s) × r00 (s).
dt dt dt dt
| dt {z } dt
=0

(c) Using (a) and (b), show that the curvature, which is defined as κ (s) := |r00 (s)|, can be
expressed in terms of t as:
|r0 (t) × r00 (t)|
κ (t) =
|r0 (t)|3
Although it looks more complicated, this formula does not require the procedure of
finding arc-length parametrization.

Solution: As r(s) travels a constant speed, in class we showed r0 (s) and r00 (s) are
two orthogonal vectors. Therefore we have

r (s) × r00 (s) = r0 (s) r00 (s) sin π = κ (s).


0
| {z } | {z } 2
1 κ (s)

Taking the magnitude on both sides of the result obtained (b), we get:
3
dr d2 r

= κ ds .

×
dt dt2 dt

Therefore, we get:
|r0 (t) × r00 (t)|
κ= 3 .
ds
dt

The proof can be easily completed by the definition of s(t) and the Fundamental
Theorem of Calculus:
ˆ t
0
s= r (τ ) dτ
0
ds
= r0 (t)

dt

Page 11
MATH 2023 • Spring 2015-16 • Multivariable Calculus
Problem Set #2 • Multivariable Functions, Partial Derivatives
p
1. (F) Let f ( x, y) = y − x2
(a) What is the (largest possible) domain of f ?

Solution: The domain is {( x, y) : y ≥ x2 }, the region above the parabola y = x2


(including the parabola) in R2 .

(b) Sketch the level sets f = 0, f = 1 and f = 2 in the same diagram.

Solution:
20

15

10

-4 -2 2 4

2. (F) Let
1
f ( x, y) = p
x2 + y2 − 1
(a) What is the (largest possible) domain of f ?

Solution: The domain is {( x, y) : x2 + y2 > 1}, the region outside the unit circle
x2 + y2 = 1, excluding the circle itself.

(b) Sketch the level sets f = 1, f = 2 and f = 3 in the same diagram.

Solution:
2

-2 -1 1 2

-1

-2

Page 1
MATH 2023 Multivariable Functions, Partial Derivatives Problem Set #2

1
(c) Repeat (a) and (b) for the function g( x, y) = p .
1 − x 2 − y2

Solution: The domain is {( x, y) : x2 + y2 < 1}, the region inside the unit circle
x2 + y2 = 1, excluding the circle itself.
1.0

0.5

-1.0 -0.5 0.5 1.0

-0.5

-1.0

3. (F) Compute all the first and second partial derivatives of the following functions. For
the second partials f xy and f yx , compute both and verify that they are indeed the same.
(a) f ( x, y) = y2015 + 2x2 + 2xy

Solution: (Answer only)

f x = 4x + 2y f y = 2x + 2015y2014
f xx = 4 f xy = 2
f yx = 2 f yy = 2015 × 2014y2013

2y
(b) f ( x, y) = e x

Solution: (Answer only)


2 2
f x = 2e x y xy f y = e x y x2
2 2 2 2
f xx = 2e x y y + 4e x y x2 y2 f xy = 2e x y x + 2e x y x3 y
2 2 2y
f yx = 2e x y x + 2e x y x3 y f yy = x4 e x

x
(c) f ( x, y) = x 2 + y2

Solution: (Answer only)

y2 − x 2 2xy
fx = fy = −
( x2 + y2 )2 + y2 )2
( x2
2 x3 − 3xy 2 2y y2 − 3x2
 
f xx = f xy = −
( x 2 + y2 )3 ( x 2 + y2 )3
2y y2 − 3x2 2x x2 − 3y2
 
f yx = − f yy = −
( x 2 + y2 )3 ( x 2 + y2 )3

Page 2
MATH 2023 Multivariable Functions, Partial Derivatives Problem Set #2

(d) f ( x, y) = x ln( x2 + y2 )

Solution: (Answer only)

2x2 2xy
+ log x2 + y2

fx = 2 2
fy =
x +y x2 + y2
2 x3 + 3xy2 2y y2 − x2
 
f xx = f xy =
( x 2 + y2 )2 ( x 2 + y2 )2
2y y2 − x2 2x x2 − y2
 
f yx = f yy =
( x 2 + y2 )2 ( x 2 + y2 )2

∂f
4. (FF) Compute the first partial derivative ∂x of the following functions (where x, y > 0).
y
(a) f ( x, y) = e x

Solution:
∂f ∂ (xy ) ∂ (xy ) ∂ y
= e = e · x
∂x ∂x ∂( xy ) ∂x
y y
= e(x ) · yx y−1 = yx y−1 e x

x
(b) f ( x, y) = ey

Solution:
∂f ∂ (y x ) ∂ (y x ) ∂ x
= e = e · y
∂x ∂x ∂(y x ) ∂x
x x
= e(y ) · y x ln y = yey ln y

y
(c) f ( x, y) = x e

Solution: (Answer only)


∂f y
= e y x e −1
∂x

x
(d) f ( x, y) = ye

Solution: (Answer only)


∂f x
= ye e x ln y
∂x

e
(e) f ( x, y) = x y

Solution: (Answer only)


∂f e
= y e x y −1
∂x

Page 3
MATH 2023 Multivariable Functions, Partial Derivatives Problem Set #2

e
(f) f ( x, y) = y x

Solution: (Answer only)


∂f e
= ex e−1 y x ln y
∂x

5. (F) Compute both the third-order derivatives h xyy and hyyx of the following function, and
verify that they are indeed the same.

h( x, y, z) = cos( x2 + y3 z).

Solution:

h x = −2x sin x2 + y3 z


h xy = (h x )y = −6xy2 z cos x2 + y3 z


h xyy = h xy y = 18xy4 z2 sin x2 + y3 z − 12xyz cos x2 + y3 z


  

hy = −3y2 z sin x2 + y3 z


hyy = hy y = −6yz sin x2 + y3 z − 9y4 z2 cos x2 + y3 z


  

hyyx = hyy x = 18xy4 z2 sin x2 + y3 z − 12xyz cos x2 + y3 z


  

 
∂ ∂f
6. (FF) Find the second derivative ∂x ∂y of each function f ( x, y ) below. [Hint: There is
a smart way to compute each of them.]
(a)
f ( x, y) = sin( x + y) cos( x − y)

Solution:
∂f
= cos( x + y) cos( x − y) + sin( x + y) sin( x − y)
∂y
= cos( x + y − ( x − y))
= cos(2y)

Here we have used the compound-angle


 formula for cos. Since this does not
∂ ∂f
depend on x we have ∂x ∂y = 0 .

Page 4
MATH 2023 Multivariable Functions, Partial Derivatives Problem Set #2

(b)
1
! 2015
sin2016 y + cos2014 y
f ( x, y) = cos( xy) + .
sin2 log(y4 + 1) + 2015

Solution: Since the second term does not depend on y, we can switch the order
of the partial derivatives taken and obtain:
   
∂ ∂f ∂ ∂f ∂
= = (−y sin( xy) + 0) = − sin( xy) − xy cos( xy)
∂x ∂y ∂y ∂x ∂y

(c)
e x +y + e x −y
f ( x, y) =
e x +y − e x −y

Solution: By factoring out e x in both numerator and denominator we obtain:

e x ey + e x e−y e x ( ey + e−y ) ey + e−y


f ( x, y) = = =
e x ey − e x e−y e x ( ey − e−y ) ey − e−y

Therefore it makes sense to switch the order of partial differentiation and calcu-
late
∂f
=0
∂x
   
∂ ∂f ∂ ∂f
Thus we also have ∂x ∂y = ∂y ∂x = 0.

∂2 f
7. (FF) Suppose that f ( x, y) is a function such that ≡ 0. Show that f can be decom-
∂x∂y
posed into the form:
f ( x, y) = F ( x ) + G (y)
where F ( x ) and G (y) are some single-variable functions.

Solution: Given that:


∂2 f
 
∂ ∂f
= = 0,
∂x ∂y ∂x∂y
∂f
we know ∂y is independent of x, and depends only on y. Therefore, it can be written
as:
∂f
= g(y)
∂y
for an arbitrary differentiable function g(y).
Now, we obtained that the y-derivative of f is g(y). To find the function f , we can
integrate g(y) with respect to y:
ˆ
f ( x, y) = g(y) dy + F ( x ).

Page 5
MATH 2023 Multivariable Functions, Partial Derivatives Problem Set #2


As ∂y is a partial derivative, the integration “constant” is not really a constant but is a
quantity not depending on y. In other words, the integration “constant” is a function
F ( x ) of x.
ˆ
Since g(y) dy is also an arbitrary function of y, for simplicity we relabel it as G (y).
Therefore, we get f ( x, y) = F ( x ) + G (y).

8. (FF) Let u( x, y, z, t) be the temperature at the point ( x, y, z) at the time t. Combining with
several important laws in thermodynamics, including the Fourier’s Law and conservation
of energy, it can be derived (detail omitted) that the temperature function u( x, y, z, t)
satisfies the following equation:
 2
∂ u ∂2 u ∂2 u

∂u
=k + 2+ 2
∂t ∂x2 ∂y ∂z
where k is a positive constant depending only on the medium. This equation is known as
the heat equation.
The study of the heat equation is an important topic in physics, engineering and mathe-
matics (both pure and applied). Through solving the heat equation with an initial condi-
tion u( x, y, z, 0) = g( x, y, z), it predicts how heat diffuses for a given an initial heat profile
g( x, y, z) at time t = 0.
Your task in this problem is to verify that the following given function is a solution to the
heat equation:
x 2 + y2 + z2
 
1
ϕ( x, y, z, t) = 3 exp − .
(4πkt) 2 4kt

This particular solution ϕ represents the heat diffusion with highly concentrated heat
source at the origin (0, 0, 0) at time t = 0. As time goes by, the temperature profile
becomes more and more uniformly distributed. (In physics, this solution is also closely
related to the Dirac delta function.)
By following the outline below, show that ϕ satisfies the heat equation:
(a) Show that:
3 3 x 2 + y2 + z2
ln ϕ( x, y, z, t) = − ln(4πk ) 2 − ln t − .
2 4kt

Solution: Take ln on the function ϕ:


!
x 2 + y2 + z2

1
ln ϕ( x, y, z, t) = ln 3 exp −
(4πkt) 2 4kt
x 2 + y2 + z2
   
1
= ln + ln exp −
(4πk)3/2 t3/2 4kt
2
x +y +z 2 2
= − ln((4πk)3/2 t3/2 ) − (ln exp u = u)
4kt
x 2 + y2 + z2
= − ln(4πk)3/2 − ln t3/2 −
4kt
3 x 2 + y2 + z2
= − ln(4πk)3/2 − ln t −
2 4kt

Page 6
MATH 2023 Multivariable Functions, Partial Derivatives Problem Set #2

(b) Using (a), show that:


x 2 + y2 + z2
 
∂ϕ 3
= − ϕ.
∂t 4kt2 2t

Solution: Differentiating both sides of (a) with respect to t:

x 2 + y2 + z2
 
∂ ∂ 3/2 3
ln ϕ = − ln(4πk) − ln t −
∂t ∂t 2 4kt
d ∂ϕ 3 2 2
x +y +z ∂ 12
ln ϕ · = 0− −
dϕ ∂t 2t 4k ∂t t
x 2 + y2 + z2
 
1 ∂ϕ 3 1
=− − − 2
ϕ ∂t 2t 4k t
x 2 + y2 + z2 3
= 2
− .
4kt 2t
 2
x + y2 + z2

∂ϕ 3
= − ϕ.
∂t 4kt2 2t

(c) Using (a) again, show that:

∂2 ϕ x2
 
∂ϕ xϕ 1
=− and 2
= − 1 ϕ.
∂x 2kt ∂x 2kt 2kt

Solution: Differentiating both sides of (a) with respect to x:

x 2 + y2 + z2
 
∂ ∂ 3 3/2
ln ϕ = − ln(4πk) − ln t −
∂x ∂x 2 4kt
 2 2 2

d ∂ϕ ∂ x +y +z
ln ϕ · = 0+0−
dϕ ∂x ∂x 4kt
1 ∂ϕ 2x + 0 + 0 x
=− =−
ϕ ∂x 4kt 2kt
∂ϕ xϕ
=− .
∂x 2kt
Differentiate both sides of the above result by x:

∂2 ϕ ∂  xϕ 
= −
∂x2 ∂x 2kt
1
=− ( ϕ + xϕ x )
2kt
1  xϕ  ∂ϕ
=− ϕ−x· (from our result of )
2kt 2kt ∂x
 2 
1 x ϕ
= −ϕ
2kt 2kt
 2 
1 x
= −1 ϕ (factor out ϕ).
2kt 2kt

Page 7
MATH 2023 Multivariable Functions, Partial Derivatives Problem Set #2

(d) Hence, verify that ϕ satisfies the heat equation: ϕt = k( ϕ xx + ϕyy + ϕzz ).

Solution: Similar calculation as in (c) shows:

∂2 ϕ
 2
∂2 ϕ
  2 
1 y 1 z
= −1 ϕ and = − 1 ϕ.
∂y2 2kt 2kt ∂z2 2kt 2kt

Therefore,

k ( ϕ xx + ϕyy + ϕzz )
  2   2   2  
1 x 1 y 1 z
= k· −1 ϕ+ −1 ϕ+ −1 ϕ
2kt 2kt 2kt 2kt 2kt 2kt
 2   2   2 
1 x 1 y 1 z
= −1 ϕ+ −1 ϕ+ −1 ϕ
2t 2kt 2t 2kt 2kt 2t
 2
y2 z2

x 1 1 1
= − + − + − ϕ
4kt2 2t 4kt2 2t 4kt2 2t
 2
x + y2 + z2

3
= − ϕ
4kt2 2t
= ϕt (from part (b)).

(e) (Optional) Show that


(
∞ if ( x, y, z) = (0, 0, 0)
lim ϕ( x, y, z, t) =
t → 0+ 0 if ( x, y, z) 6= (0, 0, 0)

Solution: Since we are taking t → 0+ , we can regard t > 0.


If ( x, y, z) = (0, 0, 0), then
 2
0 + 02 + 02

1 1
ϕ( x, y, z, t) = ϕ(0, 0, 0, t) = 3 exp − = 3 .
(4πkt) 2 | 4kt (4πkt) 2
{z }
=1
1
lim ϕ(0, 0, 0, t) = lim 3 = ∞.
t → 0+ t → 0+ (4πkt) 2
If ( x, y, z) 6= (0, 0, 0), then x2 + y2 + z2 6= 0. For simplicity, denote:
x 2 + y2 + z2
A := − 6= 0
4k
then A
1 − At e− t
ϕ( x, y, z, t) = 3 e = .
(4πkt) 2 (4πk)3/2 t3/2
A
When taking t → 0+ , we regard ( x, y, z) as constants. As e− t goes to 0 much
faster than t3/2 does, we can conclude that:
A
e− t
lim 3/2 = 0.
t → 0+ t

Therefore, it concludes that lim ϕ( x, y, z, t) = 0 when ( x, y, z) 6= 0.


t → 0+

Page 8
MATH 2023 • Spring 2015-16 • Multivariable Calculus
Problem Set #3 • Chain Rule, Directional Derivatives, Gradients

1. (F) Suppose w = f ( x, y, z) where x = g(s), y = h(s, t) and z = k (t). Assume all functions
involved are C1 . Draw the tree diagram to showcase the relations between w, x, y, z, s and
∂w ∂w
t. Hence, write down the chain rule for calculating the partial derivatives: and .
∂s ∂t
Use the symbols ∂ and d appropriately.

Solution:
W
∂w ∂w dx ∂w ∂y
∂s
=
∂x ds
+
∂y ∂s *
∂w
=
∂w ∂y ∂w dz
+
# +
∂t ∂y ∂t ∂z dt

2. (F) Recall that the rectangular-polar coordinates conversion rules are given as follows:

x = r cos q
y = r sin q

∂f
A function f ( x, y) is said to be rotationally/radially symmetric if ∂q = 0, i.e. when
:
regarded as a function of (r, q ), it depends only the radial variable r but not the angular
variable q. For instance, f ( x, y) = x2 + y2 is rotationally symmetric since f (r, q ) = r2 .
Using the chain rule, show that f is rotationally symmetric if and only if:

∂f ∂f
y =x .
∂x ∂y

Solution: By the chain rule:

∂f ∂ f ∂x ∂ f ∂y
= +
∂q ∂x ∂q ∂y ∂q

a.
∂f ∂ ∂f ∂
= (r cos q ) + (r sin q )
∂x ∂q ∂y ∂q
'
∂f ∂f .
= ( r sin q ) + (r cos q )
∂x ∂y
∂f ∂f
= y +x .
∂x ∂y
∂f
Therefore, ∂q = 0 if and only if y ∂∂xf = x ∂∂yf .

Page 1
MATH 2023 Chain Rule, Directional Derivatives, Gradients Problem Set #3

3. (F) Suppose f (u, v) is a C2 function, and u = s2 t and v = s + t2 . Express the second


∂2 f
partial derivative in terms of f uu , f uv , f vv , s and t.
∂s∂t

Solution:
∂f ∂ f ∂u ∂ f ∂v
= +
∂t ∂u ∂t ∂v ∂t
= f u · ( 1) + f v · 2t = f u + 2t f v fax
∂2 f ∂ ∂f
✓ ◆ st
=
∂s∂t ∂s ∂t

= ( f u + 2t f v )
∂s✓ ◆ ✓ ◆
∂ f u ∂u ∂ f u ∂v ∂ f v ∂u ∂ f v ∂v
= + + 2t +
∂u ∂s ∂v ∂s ∂u ∂s ∂v ∂s
= ( f uu · 2s + f uv · 1) + 2t ( f vu · 2s + f vv · 1)
2s f uu + (4t 1) f uv + 2t f vv .
=
* t
Here we have used the fact that f is C2 , and so f uv = f vu .

4. (F) Let f ( x, y, z) be a C1 function of three variables, and z be a C1 function of ( x, y) such


that
f ( x, y, z( x, y)) = 0.
Using the chain rule, show that:

∂z fx ∂z fy
= and = .
∂x fz ∂y fz

Solution:
∂ ∂ f
f ( x, y, z( x, y)) = 0=0
∂x
∂f
∂x
∂ f ∂z
* Z

+ =0 ^
∂x ∂z ∂x

:
∂f
∂z ∂x fx
= ∂f
=
∂x fz
∂z

Similarly,

∂ ∂
f ( x, y, z( x, y)) = 0=0 i
∂y ∂y = Z

∂f ∂ f ∂z ^
+ =0 X T
∂y ∂z ∂y
∂f
∂z ∂y fy
= ∂f
=
∂y fz
∂z

Page 2
MATH 2023 Chain Rule, Directional Derivatives, Gradients Problem Set #3

5. (FF) Let f ( x, y) be a C1 function. Consider two parametric curves r1 (t) = x1 (t)i + y1 (t)j
and r2 (t) = x2 (t)i + y2 (t)j which satisfy:

r1 (0) = r2 (0) and r10 (0) = r20 (0).

(a) Show that


d d
f ( x1 (t), y1 (t)) = f ( x2 (t), y2 (t)).
dt t =0 dt t =0

Solution:
Using the chain rule, we get:

d ∂ f dx ∂ f dy
f ( x1 (t), y1 (t)) = +
dt ∂x dt ∂y dt
∂f 0 ∂f 0
= x1 ( t ) + y (t) (since x = x1 and y = y1 in this case)
∂x ∂y 1

At t = 0, we get:

d ∂f ∂f
f ( x1 (t), y1 (t)) = x10 (0) + y10 (0)
dt t =0 ∂x ( x1 (0),y1 (0)) ∂y ( x1 (0),y1 (0))

Similarly, one can also show:

d ∂f ∂f
f ( x2 (t), y2 (t)) = x20 (0) + y20 (0)
dt t =0 ∂x ( x2 (0),y2 (0)) ∂y ( x2 (0),y2 (0))

It is given from the problem that:

r1 (0) = r2 (0) and r10 (0) = r20 (0).

Therefore, we have:

x1 (0) = x2 (0) x10 (0) = x20 (0)


y1 (0) = y2 (0) y10 (0) = y20 (0)

and so:
d d
f ( x1 (t), y1 (t)) = f ( x2 (t), y2 (t)).
dt t =0 dt t =0

(b) Give a geometric interpretation of the above result.

Solution: This result shows that any two parametric curves with the same posi-
tion and velocity at t = 0 will give the same rate of change of a function f along
these two curves at t = 0.
"

toffee
"

Page 3
MATH 2023 Chain Rule, Directional Derivatives, Gradients Problem Set #3

6. (FF) The wave equation is an important partial differential equation which governs the
propagation of waves. Let u( x, y, z, t) be the displacement of the wave at position ( x, y, z)
at time t. It can be shown by several physical laws (such as the Hooke’s Law) that u
satisfies: ✓ 2 ◆
∂2 u 2 ∂ u ∂2 u ∂2 u
=c + 2+ 2 (1)
∂t2 ∂x2 ∂y ∂z
where c is a constant (which is the wave speed).
In one (spatial) dimension, the wave equation can be stated as:

∂2 u 2
2∂ u
= c . (2)
∂t2 ∂x2

It turns out that the chain rule of several variables has a nice application on solving
the one dimensional wave equation. The following exercise guides you to show that if
u( x, t) is a solution to the one dimensional wave equation, then it must take the form
u( x, t) = F ( x ct) + G ( x + ct) where F and G are arbitrary differentiable functions of
single variable.
Let u( x, t) solve the one dimensional wave equation (2).
(a) Define x = x ct and h = x + ct. Regard u as a function of x and h, and x and h are
functions of x and t. Using the chain rule of multivariable functions, show that:

ut = c(uh ux ) and utt = c2 (uxx 2uxh + uhh ).

Solution: By chain rule:

∂u ∂u ∂x ∂u ∂h
= + x
∂t ∂x ∂t ∂h ∂t
= ux · ( x ct)t + uh · ( x + ct)t m
= ux · ( c) + uh · c
= c ( u h u x ).

tx
∂ut
utt =
∂t

= c ( u h u x ).
∂t

Since uh and ux are also functions of (x, h ), the chain rule applies to uh and ux in
the same way as it does to u:

∂uh ∂uh ∂x ∂uh ∂h

*
= +
∂t ∂x ∂t ∂h ∂t
= uhx · ( x ct)t + uhh · ( x + ct)t
÷
'

cuhx + cuhh .
=
∂ux ∂ux ∂x ∂ux ∂h
= +
∂t ∂x ∂t ∂h ∂t
= uxx · ( x ct)t + uxh · ( x + ct)t
A
×
He

= cuxx + cuxh .

Page 4
MATH 2023 Chain Rule, Directional Derivatives, Gradients Problem Set #3

Substitute these two results into utt , we have:



utt = c (uh ux )
∂t
✓ ◆
∂uh ∂ux
=c
∂t ∂t
= c( cuhx + cuhh + cuxx cuxh )
= c2 (uxx uxh uhx + uhh )
2
= c (uxx 2uxh uhh ).

(b) Using the chain rule agin, show that

u x = ux + uh and u xx = uxx + 2uxh + uhh .

Solution: Apply the chain rule again:


×
u x = ux x x + uh hx
= ux ( x ct) x + uh ( x + ct) x
= ux + uh .
=
Att
u xx = (u x ) x
= (ux + uh ) x
= (ux ) x + (uh ) x
= (ux )x x x + (ux )h hx + (uh )x x x + (uh )h hx
= uxx ( x ct) x + uxh ( x + ct) x + uhx ( x ct) x + uhh ( x + ct) x
= uxx + uxh + uhx + uhh
= uxx + 2uxh + uhh .

(c) Combining the results of (a), (b) and the wave equation, show that uxh = 0.

Solution: Results in (a) and (b) show:

utt = c2 (uxx 2uxh + uhh ), u xx = uxx + 2uxh + uhh .

Substitute them into the wave equation utt = c2 u xx , we have:

c2 (uxx 2uxh + uhh ) = c2 (uxx + 2uxh + uhh )


uxx 2uxh + uhh = uxx + 2uxh + uhh
2uxh = 2uxh
0 = 2uxh + 2uxh = 4uxh
uxh = 0.

Page 5
MATH 2023 Chain Rule, Directional Derivatives, Gradients Problem Set #3

(d) Finally, deduce that u, as a function of x and h, must be in the form of:

u(x, h ) = F (x ) + G (h )

where F and G are arbitrary functions. Hence, in terms of the original variables x
and t, u must take the form u( x, t) = F ( x ct) + G ( x + ct).

Solution: Finally, we have uxh = 0. Therefore,

∂ux
= 0.
∂h

Thus ux is independent of h, so ux is a function of x only. Let f (x ) be this function


and so
∂u
ux = f (x ), or equivalently, = f (x )
∂x
Integrating both sides by x, we get:
ˆ
u= f (x )dx + ‘integration constant’.

Note that the integration ‘constant’ is no longer a constant, but a quantity not de-
pending on the integration variable x. In other words, the ‘integration constant’
now becomes a function of h.
Denote this function by G (h ), then:
ˆ
u(x, h ) = f (x )dx + G (h ).

Since f is an arbitrary function of f (x )dx is arbitrary too. We rewrite


´
( x ) x,
f (x )dx as F (x ), where F is an arbitrary function of x. Finally, we have:
´

u(x, h ) = F (x ) + G (h ).

Since x = x ct and h = x + ct, in terms of the ( x, t)-variables:

u( x, t) = F ( x ct) + G ( x + ct).

FYI: The general solution u( x, t) = F ( x ct) + G ( x + ct) of the wave equation describes
the superposition of two waves – one has a graph given by the function F and it shifts to
the right by c unit lengths per unit time, another has a graph given by the function G and
it shifts to the left by c unit lengths per unit time. The results in this exercise show that
all solutions to the one-dimensional wave equation are superposition of these waves.

7. (FFF) In many physics, geometry and engineering applications, it is often more conve-
nient to use polar or sphereical coordinates since many physical quantities are rotationally
symmetric.
The conversion rule of rectangular and polar coordinates is given by:

x = r cos q
y = r sin q

Page 6
MATH 2023 Chain Rule, Directional Derivatives, Gradients Problem Set #3

Let u be a function of x and y. Since ( x, y) can be converted into (r, q ), we can also regard
u as a function of (r, q ). The chain rule can be used to derive some conversion formulae
between u x , uy and ur , uq .
An important operator in physics, geometry and engineering is called the Laplacian. In
two dimensions, it is defined as:
∂2 u ∂2 u
r2 u = + 2 = u xx + uyy .
∂x2 ∂y
In this exercise, we will show that r2 u can be expressed in polar form as:
∂2 u 1 ∂u 1 ∂2 u
r2 u = + + .
∂r2 r ∂r r2 ∂q 2
The polar form of the Laplacian is often used when dealing with rotationally symmetric
functions, i.e. a function u which does not depend on q but only on r. For such functions,
their Laplacian is simply:
1
r2 u = urr + ur .
r
p y
2 2
(a) Use the fact that r = x + y and tan q = x , show that:
∂r x ∂r y ∂q y ∂q x
= , = , = , = 2.
∂x r ∂y r ∂x r2 ∂y r

Solution: Using the fact that r2 = x2 + y2 , by differentiating both sides with


respect to x, we get:
∂ 2 ∂ 2
r = ( x + y2 )
∂x ∂x
∂r ∂r x
2r = 2x =) =
∂x ∂x r
Similarly, by differentiating both sides with respect to y, we get:
∂ 2 ∂ 2
r = ( x + y2 )
∂y ∂x
∂r ∂r y
2r = 2y =) =
∂y ∂y r
y
To find ∂x
∂q
and ∂y
∂q
, we consider the fact that tan q = x. Differentiate both sides
with respect to x, we get:
∂ ∂ ⇣y⌘
tan q =
∂x ∂x x
2 ∂q y
sec q =
∂x x2
∂q y
=
∂x x sec2 q
2

y y2
Since sec2 q = 1 + tan2 q, and recall that tan q = x, we have sec2 q = 1 + x2
.
Therefore, after simplication, we can obtain:
∂q y y y y
= = ⇣ ⌘ = = .
∂x x2 sec2 q x2 · 1+ y2 x 2 + y2 r2
x2

Page 7
MATH 2023 Chain Rule, Directional Derivatives, Gradients Problem Set #3

y
Similarly, differentiating both sides of tan q = x by y, we get:

∂ ∂ ⇣y⌘ 1
tan q = =
∂y ∂y x x
∂q 1
sec2 q =
∂y x
∂q 1
=
∂y x sec2 q

y2
We have previously derived that sec2 q = 1 + x2
, and so:

∂q 1 1 1 x
= ⇣ ⌘ = ⇣ ⌘ = = .
∂y x 1+
y2
x
x 2 + y2 r2 r2
x2 x2 x

(b) Regard u as a function of (r, q ), and (r, q ) are functions of ( x, y). Sketch a tree
diagram to showcase these relations. Using the chain rule, show that:
xur yuq
ux = ,
r r2
yur xu
uy = + 2q .
r r

Solution:
∂u ∂r ∂u ∂q
ux = +
∂r ∂x ∂q ∂x
⇣ y⌘
x
= ur · + u q · from (a)
r r2
xur yuq
=
r r2
∂u ∂r ∂u ∂q
uy = +
∂r ∂y ∂q ∂y
y x
= ur · + u q · 2 from (a)
r r
yur xu
= + 2q
r r

Page 8
MATH 2023 Chain Rule, Directional Derivatives, Gradients Problem Set #3

(c) Using quotient and product rules, show that:

ur xurx x 2 ur yuqx 2xyuq


u xx = + 3 2
+
r r r r r4
ur yury 2
y ur xuqy 2xyuq
uyy = + 3
+ 2
r r r r r4

Solution:
∂u x ∂ ⇣ xur yuq ⌘
u xx = = from (b)
∂x ∂x r r2
r · ∂x ( xur ) xur ∂x
∂ ∂r
r2 · ∂x
∂ ∂ 2
(yuq ) yuq · ∂x r
= 2 4
quotient rule
r r
x 2
r (ur + xurx ) xur · r r · yuqx yuq · 2r · ∂x
∂r
= product rule
r2 r4
ur xurx 2
x ur yuqx 2ryuq · xr
= + + break it down
r r r3 r2 r4
which is exactly what we need to show after simplifying the last term.
Similarly for uyy :

∂ ∂ ⇣ yur xu ⌘
uyy = uy = + 2q
∂y ∂y r r
r ∂y

(yur ) yur ∂y
∂r
r2 ∂y

( xuq ) xuq · ∂ 2
∂y r
= +
r2 r4
r ur + yury
y
yur · r r2 xuqy xuq · 2r · ∂r
∂y
= +
r2 r4
ur yury y2 ur xuqy 2rxuq · yr
= + +
r r r3 r2 r4
as required (after simplifying the last term).

(d) Since ur and uq are functions of (r, q ), and (r, q ) are functions of ( x, y), they share the
same tree diagram as u in part (b), and hence we have

∂ur ∂ur ∂r ∂ur ∂q


urx = = +
∂x ∂r ∂x ∂q ∂x
and similar for other second derivatives ury , uqx and uqy . Show that:

xurx + yury = rurr


xuqy yuqx = uqq

Solution: We apply the chain rule for each of urx , ury , uqx and uqy :

∂ur ∂ur ∂r ∂ur ∂q


urx = = +
∂x ∂r ∂x ⇣ ∂q⌘∂x
x y
= urr · + urq · from (a)
r r2

Page 9
MATH 2023 Chain Rule, Directional Derivatives, Gradients Problem Set #3

⇣ ⌘
∂ur ∂2 u
Note that ∂r = ∂
∂r
∂u
∂r = ∂r2
= urr . Similar for urq .
Similarly,

∂ur ∂ur ∂r ∂ur ∂q


ury = = +
∂y ∂r ∂y ∂q ∂y
y x
= urr · + urq · 2
r r
Combining these two results, we then have:
⇣ x y⌘ ⇣ y x⌘
xurx + yury = x urr · urq · 2 + y urr · + urq · 2
r r r r
x2 xy y2 xy
= urr · urq · 2 + urr · + urq · 2
r r r r
x 2 + y2
= urr · cancellation
r
r2
= urr · = rurr
r

The second identity can be proven in a similar way:

∂ ∂uq ∂r ∂u ∂q
uqy = uq = + q
∂y ∂r ∂y ∂q ∂y
y x
= uqr · + uqq · 2
r r
∂ ∂uq ∂r ∂u ∂q
uqx = uq = + q
∂x ∂r ∂x⇣ ∂q ∂x
x y⌘
= uqr · + uqq ·
r r2
Therefore, we can show:
⇣ y x⌘ ⇣ x ⇣ y ⌘⌘
xuqy yuqx = x uqr · + uqq · 2 y uqr · + uqq ·
r r r r2
xy x 2 xy y 2
= uqr · + uqq · 2 uqr · + uqq ·
r r r r
x 2 + y2
= uqq ·
r2
r2
= uqq · 2 = uqq
r
as required.

Page 10
MATH 2023 Chain Rule, Directional Derivatives, Gradients Problem Set #3

(e) Combining the results proved in previous parts, show that:


1 1
u xx + uyy = urr + ur + 2 uqq .
r r

Solution: OK! Finally, we can combine everything together:

u xx + uyy
ur xurx x 2 ur yuqx 2xyuq
= + 3 2
+
r r r r r4
ur yury y2 ur xuqy 2xyuq
+ + + from (c)
r r r3 r2 r4
2ur xurx + yury 2 2
( x + y ) ur
= +
r r r3
xuqy yuqx
+ factorization, tidy up
r2
2ur rurr r 2 ur u
= + 3
+ qq from (d)
r r r r2
2ur ur u
= + urr + qq
r r r2
1 1
= urr + ur + 2 uqq
r r
which is exactly as required! Cheers!

8. (F) Compute the directional derivative of the following functions at the given point P in
the direction of the given vector v. Moreover, find the unit direction u along which the
function increases most rapidly.
(a) f ( x, y) = x2 y2 , P( 1, 3), v = 35 i 4
5 j.

Solution:
∂f ∂f
rf = i+ j = 2x i 2y j
∂x ∂y
r f ( P) = r f ( 1, 3) = 2( 1) i 2( 3) j = 2i + 6j
s
✓ ◆2 ✓ ◆
3 4 2
|v| = + =1
5 5
v 3 4
v̂ = =v= i j
|v| 5 5
✓ ◆
3 4
Dv̂ f ( P) = r f ( P) · v̂ = ( 2i + 6j) · i j = 6
5 5
Since Du f ( P) = r f ( P) · u = |r f ( P)| |u| cos q = |r f ( P)| cos q for any unit vector
u. Here q is the angle between r f ( P) and u. It is the largest when q = 0 (i.e.
cos q = 1). Therefore, Du f ( P) achieves its maximum when r f ( P) and u are
parallel. Therefore, the unit direction u along which the function increases most
rapidly is given by:
r f ( P) 2i + 6j
u= = p .
|r f ( P)| 40

Page 11
MATH 2023 Chain Rule, Directional Derivatives, Gradients Problem Set #3

(b) g( x, y) = e x y, P(ln 2, ln 3), v = i + j.

Solution:
x y x y
rg = e i e j
ln 2 ln 3 ln 2 ln 3 1 1
r g( P) = e i e j= i j
6 6
v i+j
v̂ = = p
|v| 2
✓ ◆
1 1 i+j 2
Dv̂ g( P) = r g( P) · v̂ = i j · p = p
6 6 2 6 2
r g( P) 1 1
u= = p i p j
|r g( P)| 2 2

(c) h( x, y) = e xy , P(1, 0), v = 5i + 12j.

Solution: (Answer only)

12
Dv̂ h( P) = , u = j.
13

(d) F ( x, y, z) = xy + yz + zx + 4, P(2, 2, 1), v = j k.

Solution: (Answer only)

3 i + 3j
Dv̂ F ( P) = p , u= p .
2 10

(e) G ( x, y, z) = e xyz 1, P(0, 1, 1), v = 2i + 2j k.

Solution: (Answer only)

2
rv̂ G ( P) = , u= i.
3

9. (F) For each surface and the given point P, find the value a such that P lies on the surface,
and then find an equation of the tangent plane to the surface at the point P:
(a) x2 + y + z = 3, P(2, 0, a)

Solution: To solve for a, we put ( x, y, z) = (2, 0, a) into the given equation:

22 + 0 + a = 3 =) a = 1

To find the equation of the tangent plane at P, we need a normal vector to the
surface:
r( x2 + y + z 3) = 2xi + j + k

Page 12
MATH 2023 Chain Rule, Directional Derivatives, Gradients Problem Set #3

Therefore, n = (2xi + j + k)| P = 2(2)i + j + k = h4, 1, 1i. Equation of the tangent


plane at P is: 4x + 1y + 1z = 4(2) + 1(0) + 1( 1). After simplification:

4x + y + z = 7.

(b) xy sin z = 1, P( a, 2, p/6)

Solution: (Answer only)

a=1
1 p
r ( xy sin z
1)| P = i + j + 3k
2
1 p p
x + y + 3z = 2 + .
2 2

(c) yze xz = 8, P(0, a, 4)

Solution: (Answer only)

a=2
xz
r (yze 8)| P = 32i + 4j + 2k
16x + 2y + z = 8

(d) z = e xy , P(1, 0, a)

Solution: (Answer only)

a=1
xy
r(z e )| P = j+k
y+z = 1

(e) z = ln(1 + xy), P(1, 2, a).

Solution:

a = ln 3
2 1
r(z ln(1 + xy))| P = i j+k
3 3
2 1 4
x y+z = + ln 3.
3 3 3

Page 13
MATH 2023 Chain Rule, Directional Derivatives, Gradients Problem Set #3

10. (F) Let


GMm
V ( x, y, z) = p
x2 + y2 + z2
where G, M and m are constants. Define F( x, y, z) = rV ( x, y, z).
(a) Verify that:
xi + yj + zk
F( x, y, z) = GMm .
( x2 + y2 + z2 )3/2

Solution:
!
∂V ∂ GMm
= p
∂x ∂x x 2 + y2 + z2
∂ 1/2
= GMm x 2 + y2 + z2
∂x✓ ◆
1 2 3/2 ∂ 2
= GMm · x + y2 + z2 ( x + y2 + z2 )
2 ∂x
1 2x
= GMm · ·
2 ( x2 + y2 + z2 )3/2
x
= GMm ·
( x + y + z2 )3/2
2 2

Similarly, we have:

∂V y
= GMm ·
∂y ( x + y + z2 )3/2
2 2

∂V z
= GMm ·
∂z ( x + y + z2 )3/2
2 2

Therefore,
∂V ∂V ∂V
rV = i+ j+ k
∂x ∂y ∂z
xi + yj + zk
= GMm
( x + y2 + z2 )3/2
2

as required. Note that F = rV.

(b) Show that |F( x, y, z)| is inversely proportional to the squared distance from ( x, y, z)
to the origin in R3 .

Page 14
MATH 2023 Chain Rule, Directional Derivatives, Gradients Problem Set #3

Solution:

xi + yj + zk
|F( x, y, z)| = GMm
( x2 + y2 + z2 )3/2
1
= GMm | xi + yj + zk|
( x2 + y2 + z2 )3/2
1/2
x 2 + y2 + z2 1
= GMm = GMm
( x 2 + y2 + z2 )
3/2 x2 + y2 + z2
p
Note that the distance from ( x, y, z) to the origin is given by r = x 2 + y2 + z2 .
Hence, we have:
GMm
|F( x, y, z)| =
r2
as required.

11. (FF) Consider the function


f ( x, y) = cos( x + y)
as well as the plane P given by the equation

x y=0.

The intersection of the graph of f with P is a curve C. Find the slope of the tangent line
to C at the point (p, p ) using directional derivatives. [Hint: First sketch a diagram of the
graph, the plane and the curve.]

Solution: The intersection between the plane P and the xy-plane is the straight-line
yD = x. EThere are two unit vectors
D in the E x-y-plane parallel to P, given by u =
p1 , p1 and its negative u = p1 , p1 . The directional derivative of f in the
2 2 2 2
direction of u is now equal to

∂f ∂f
rf · u = , ·u
∂x ∂y
1 1
= p sin( x + y) p sin( x + y)
2 2
p
= 2 2 sin( x + y)

So the slope of C at (p, p ) is therefore equal to 0, by evaluating this equation at


( x, y) = (p, p ).
Alternatively one can also work with u instead of u and one would obtain the
negative of the above solution, which is also 0 in our situation.

12. (FF) One approach for finding the normal vector of the tangent plane at a given point
( x0 , y0 ) to a graph z = f ( x, y) is by writing the graph equation as a level surface z
Df ( x, y) = 0 ofE a three-variable function g( x, y, z) := z f ( x, y). Then, the gradient r g =
∂f ∂f
∂x , ∂y , 1 at point ( x0 , y0 , f ( x0 , y0 )) is perpendicular to the level surface { g = 0},

Page 15
MATH 2023 Chain Rule, Directional Derivatives, Gradients Problem Set #3

and so we can take it to be a normal vector of the tangent plane as long as r g 6= 0 at


( x0 , y0 , f ( x0 , y0 )).
D E
∂f ∂f
In fact, it is also possible to show the normal vector is ∂x , ∂y , 1 using a purely two-
variable argument instead of going up one higher dimension.

(a) Consider a given function f ( x, y), and a given point ( x0 , y0 ). Find a parametrization:
r1 (t) =?i+?j+?k
of the curve on the graph z = f ( x, y) travelling in the x-direction while keeping y
fixed at y0 (i.e. the red curve in the diagram). Hence, find the tangent vector of the
curve r1 (t) at the point ( x0 , y0 , f ( x0 , y0 )). Label this tangent vector by T1 .

Solution: The projection of the red curve on the xy-plane is a straight line pass-
ing through ( x0 , y0 ) and is parallel to the x-axis, i.e. vector i. Therefore, the
( x, y)-coordinates of the path is given by:
x ( t ) = x0 + t
y ( t ) = y0
The z-coordinate of the red curve is determined by the function f , i.e. z(t) =
f ( x (t), y(t)) = f ( x0 + t, y0 ). Therefore, the parametrization of the red curve is:
r1 (t) = x (t)i + y(t)j + z(t)k = ( x0 + t)i + y0 j + f ( x0 + t, y0 ) k.
When t = 0, the “particle” is at the point ( x0 , y0 , f ( x0 , y0 )). Therefore, the tangent
vector T1 to the curve at this point is given by:
d
T1 = r10 (0) = i + 0j + f ( x0 + t, y0 ) k.
dt t =0
d
To compute dt f ( x0 + t, y0 ), we apply the chain rule:
d ∂f d ( x0 + t ) ∂ f d ( y0 )
f ( x0 + t, y0 ) = ( x0 + t, y0 ) + ( x0 + t, y0 )
dt ∂x dt ∂y dt
∂f
= ( x0 + t, y0 ).
∂x
Evaluate at t = 0, we get:
d ∂f
f ( x0 + t, y0 ) = ( x0 , y0 ).
dt t =0 ∂x
Therefore, we get:
∂f
T1 = i + ( x0 , y0 ) k.
∂x

Page 16
MATH 2023 Chain Rule, Directional Derivatives, Gradients Problem Set #3

(b) Find a parametrization r2 (t) of the curve on the graph z = f ( x, y) travelling in the y-
direction while keeping x fixed at x0 (i.e. the blue curve in the diagram). Hence, find
the tangent vector of r2 (t) at the point ( x0 , y0 , f ( x0 , y0 )). Label this tangent vector by
T2 .

Solution: Similar to (a), the parametrization is given by:

r2 ( t ) = x0 i + ( y0 + t ) j + f ( x0 , y0 + t ) k

and the tangent vector at ( x0 , y0 , f ( x0 , y0 )) is:

∂f
T2 = r20 (0) = j + ( x0 , y0 ) k
∂y

(c) Since both T1 and T2 are tangent vectors to the graph, they are parallel to the tangent
plane. Therefore, the normal vector to the tangent plane must be perpendicular to
both T1 and T2 . Using this fact, show that the normal vector to the tangent plane is
given by ⌧
∂f ∂f
( x0 , y0 ), ( x0 , y0 ), 1 .
∂x ∂y

Solution: Since the normal vector n to the tangent plane is orthogonal to the
tangent vectors T1 and T2 , it can be computed by taking cross product: n =
T1 ⇥ T2 which will yield the vector stated in the problem.

Optional

13. The spherical coordinates (r, q, f) is another important coordinate system in R3 . We


will learn that in later chapters. The conversion rules between spherical and rectangular
coordinates are given by:

x = r sin f cos q
y = r sin f sin q
z = r cos f

Given a C2 function f ( x, y, z), it can be regarded as a function of (r, q, f) as well under the
above conversion rule. Show that the Laplacian r2 f := f xx + f yy + f zz can be expressed
in spherical coordinates as:
✓ ◆ ✓ ◆
∂2 f ∂2 f ∂2 f 1 ∂ 2∂f 1 ∂ ∂f 1 ∂2 f
+ + = 2 r + 2 sin f + .
∂x 2 ∂y 2 ∂z 2 r ∂r ∂r r sin f ∂f ∂f r2 sin2 f ∂q 2

[Note: It is a very time consuming exercise. It took me 4 hours to do it when I was an


undergraduate.]

Page 17
MATH 2023 • Spring 2015-16 • Multivariable Calculus
Problem Set #4 • Critical Points, Lagrange’s Multiplier
1. (F) Find all local extrema (a.k.a. critical points) of the following functions f ( x, y). Deter-
mine the nature (a local minimum, a local maximum or a saddle) of each of them using
the Second Derivative Test whenever possible. If the Second Derivative Test is inconclu-
sive, use some other methods to determine its nature.
(a) f ( x, y) = 4 + x3 + y3 − 3xy

∂f ∂f
Solution: To find critical points, we solve ∂x = 0 and ∂y = 0, i.e.

3x2 − 3y = 0
3y2 − 3x = 0

After simplification: x2 = y and y2 = x. Substitute y = x2 into y2 = x, we get:


2
x2 = x =⇒ x4 − x = 0 =⇒ x ( x3 − 1) = 0

Therefore, we have x = 0 or x3 = 1. The latter case gives x = 1.


Since y = x2 , we get y = 0 when x = 0; whereas y = 1 when x = 1. There are
two critical points:
( x, y) = (0, 0), (1, 1).
Next, compute the second derivatives:

f xx = 6x f xy = −3
f yx = −3 f yy = 6y
 
P f xx ( P) f yy ( P) f xy ( P) 2
f xx f yy − f xy ( P) P is:
(0, 0) 0 0 −3 −9 saddle
(1, 1) 6 6 −3 (6)(6) − (−3)2 > 0 local minimum

(b) f ( x, y) = x2 + 4y2 − 2x2 y + 4

Solution: (Answer only)


Critical Point f xx 2
f xx f yy − f xy Nature
(√0, 0) 2 16 local minimum
(−√ 2, 12 ) 0 −32 saddle
( 2, 12 ) 0 −32 saddle

(c) f ( x, y) = sin x cos y

∂f ∂f
Solution: To solve for critical points, we solve ∂x = 0 and ∂y = 0, i.e.

cos x cos y = 0 and − sin x sin y = 0

From the first equation, we get two cases: Case A: cos x = 0, or Case B: cos y = 0.
Case A: Since cos x = 0, we have x = π2 + kπ where k is any integer. Then
sin x = ±1 (depending on whether k is even or odd), and the second equation

Page 1
MATH 2023 Critical Points, Lagrange’s Multiplier Problem Set #4

sin x sin y = 0 then tells us that sin y = 0, and so y = mπ where m is any integer.
Critical points obtained in this case are:
π 
( x, y) = + kπ, mπ , where m and k are any integers.
2

Case B: From cos y = 0, we get y = π2 + nπ where n is any integer, then substitute


it into the second equation, we get sin x · (±1) = 0 where ± depends on whether
k is even or odd. Therefore, x = qπ where q is any integer. Critical points
obtained in this case are:
π
( x, y) = (qπ, + nπ ) where n and q are any integers.
2

To apply the Second Derivative Test, we first compute:

f xx = − sin x cos y
f xy = − cos x sin y
f yy = − sin x cos y
2
f xx f yy − f xy = sin2 x cos2 y − cos2 x sin2 y
 
P f xx f yy f xy 2
f xx f yy − f xy P is:
( π2
+ kπ, mπ ) k odd, m odd −1 −1 0 1 local max
( π2
+ kπ, mπ ) k odd, m even 1 1 0 1 local min
( π2
+ kπ, mπ ) k even, m odd 1 1 0 1 local min
( π2
+ kπ, mπ ) k even, m even −1 −1 0 1 local max
(qπ, π2 + nπ ) any integers q, n 0 0 ±1 −1 saddle

(d) f ( x, y) = x4 + y4

Solution: By solving ∇ f = 4x3 i + 4y3 j = 0, we get (0, 0) as the only critical point
2 = 0 at (0, 0), and so the Second
of f . However, one can check that f xx f yy − f xy
Derivative Test does not conclude anything.
However, this function f ( x, y) = x4 + y4 is always non-negative. Therefore,
f ( x, y) ≥ f (0, 0) = 0 for any ( x, y). In other words, (0, 0) is a local minimum
point (in fact a global minimum).

Page 2
MATH 2023 Critical Points, Lagrange’s Multiplier Problem Set #4

2. (FF) Give an example of a C2 function f ( x, y) such that:


2 = 0 at (0, 0).
(a) (0, 0) is a saddle point and f xx f yy − f xy

Solution: One of many examples: f ( x, y) = x4 − y4 . Along the x-axis, the graph


is f ( x, 0) = x4 which is concave up. However, along the y-axis, the graph
is f (0, y) = −y4 which is concave down. It is straight-forward to verify that
f xx (0, 0) = f yy (0, 0) = f xy (0, 0) = 0.

2 = 0, f
(b) (0, 0) is a local minimum, f xx f yy − f xy xx > 0 and f yy = 0 at (0, 0).

Solution: One of many examples: f ( x, y) = x2 + y4 . It must be non-negative,


and f (0, 0) = 0. Therefore, the origin must be a minimum point (in fact the
absolute minimum). It is straight-forward to verify that f xx (0, 0) = 2 whereas
f yy (0, 0) = f xy (0, 0) = 0.

2 = 0, f
(c) (0, 0) is a local maximum, f xx f yy − f xy xx = 0 and f yy < 0 at (0, 0).

Solution: One of many examples: f ( x, y) = − x4 − y2 .

3. (F) Using Lagrange’s Multipliers, find the maximum and minimum values of the given
function subject to the given constraint:
(a) f ( x, y) = x + 2y subject to x2 + y2 = 4

Solution: Let g( x, y) = x2 + y2 , then the constraint is the level-set g( x, y) = 4.


The Lagrange’s Multiplier system is:

∂f ∂g
=λ 1 = λ · 2x
∂x ∂x
∂f ∂g
=λ 2 = λ · 2y
∂y ∂y
g( x, y) = 4 x 2 + y2 = 4

From the first equation, we must have λ 6= 0 (otherwise if λ = 0, then 1 = 0).


Therefore, by rearranging the first and second equations, we get:

1 1
= 2x and =y
λ λ
Therefore, 2x = y and combine with the third equation, we get:
r r
2 2 2 2 4 4 4
x + (2x ) = 4 =⇒ 5x = 4 =⇒ x = =⇒ x = or − .
5 5 5
Since y = 2x, the candidate points are:
√ √ √ √
( x, y) = ( 4/5, 2 4/5), (− 4/5, −2 4/5).

Page 3
MATH 2023 Critical Points, Lagrange’s Multiplier Problem Set #4

Substitute them into f ( x, y), we get:


r r ! r
4 4 4
f , 2 =5
5 5 5
r r ! r
4 4 4
f − , −2 = −5
5 5 5

The former is the maximum, the latter is the minimum.

(b) f ( x, y) = x − y subject to x2 + y2 = 20 + 3xy

Solution: Note that the constraint is not yet in the level-set form. By rearrange-
ment, we get:
x2 + y2 − 3xy = 20
Therefore, we let g( x, y) = x2 + y2 − 3xy and then the constraint becomes the
level-set g( x, y) = 20. We need to solve the Lagrange’s Multiplier system ∇ f ( x, y) =
λ∇ g( x, y) and g( x, y) = 20:

1 = λ(2x − 3y)
−1 = λ(2y − 3x )
2 2
x + y − 3xy = 20

Solutions are: ( x, y) = (−2, 2), (2, −2). Substitute them into f , we get:

f (−2, 2) = −4, and f (2, −2) = 4.

The maximum is 4 and the minimum is −4.

(c) f ( x, y, z) = xyz subject to x2 + 2y2 + 4z2 = 9

Solution: Let g( x, y, z) = x2 + 2y2 + 4z2 , then the constraint is g( x, y, z) = 9. The


Lagrange’s Multiplier system is:

yz = λ · 2x
xz = λ · 4y
xy = λ · 8z
x + 2y + 4z2 = 9
2 2

The strategy of solving this system is as follows: Suppose the third equation
xy = λ · 8z 6= 0, then by dividing the first equation by the third one gives:

yz 2λx z x
= =⇒ = =⇒ 4z2 = x2 .
xy 8λz x 4z

Also, divide the second equation by the third one gives:

xz 4yλ z y
= =⇒ = =⇒ y2 = 2z2
xy 8zλ y 2z

Page 4
MATH 2023 Critical Points, Lagrange’s Multiplier Problem Set #4

Then, substitute x2 = 4z2 and y2 = 2z2 into the fourth equation gives: 4z2 + 4z2 +
4z2 = 9. This gives the solutions for z, which can be used to determine the values
of x and y.
Next, we need to find the solutions for the case when the third equation is zero,
i.e. xy = λ · 8z = 0.
We skip the detail of computations here. Below are the results:
( x, y, z) f ( x, y, z)
(±3, 0, 0) 0
(0, 0, ±√ 3/2) 0
(0, ±3/ 2, 0) 0q
√ √ √
(± 3, ± 3/2, ± 3/2) ± 2 32 3
q q
The maximum is 32 32 and the minimum is − 32 32 .

(d) f ( x1 , x2 , . . . , xn ) = x1 + 2x2 + . . . + nxn subject to x12 + x22 + . . . + xn2 = 1

Solution: Let g( x1 , . . . , xn ) = x12 + . . . + xn2 , then the constraint becomes the level-
set g = 1. The Lagrange’s Multiplier system is:
∂f ∂g
=λ 1 = 2λx1
∂x1 ∂x1
∂f ∂g
=λ 2 = 2λx2
∂x2 ∂x2
.. ..
. .
∂f ∂g
=λ n = 2λxn
∂xn ∂xn
g ( x1 , . . . , x n ) = 1 x12 + . . . + xn2 = 1
It is a very large system. However, it is helpful to note that λ cannot be zero
(otherwise from the first equation we would have 1 = 0). By rearrangement, we
can get:
1 2 n
x1 = , x2 = , . . . , xn = .
2λ 2λ 2λ
Substitute them into the constraint equation, we get:
 2  2  n 2
1 2
+ +...+ =1
2λ 2λ 2λ
After simplification, we get:
12 + 22 + . . . + n 2 p
= 1 =⇒ 2λ = ± 12 + 22 + . . . + n2 .
(2λ)2
1 2 n
Therefore, the candidate points are: x1 = 2λ , x2 = 2λ , . . . , xn = where
√ 2λ
2λ = ± 12 + 22 + . . . + n2 .

1 2 n 12 + 22 + . . . + n2
f ( x1 , . . . , x n ) = +2· +...+n· = ±√
2λ 2λ 2λ 12 + 22 + . . . + n2
2 +22 + ... + n2 2 2 2
The minimum is − √1 and the maximum is √1 +2 +...+n .
12 +22 +...+n2 12 +22 +...+n2

Page 5
MATH 2023 Critical Points, Lagrange’s Multiplier Problem Set #4

4. (FF) The rationale behind Lagrange’s Multipliers method is that if a function f achieves
its maximum or minimum on the constraint g = c at points P, then the level sets of f and
g at P must be tangent to each other, and so ∇ f ( P) is parallel to ∇ g( P). Therefore, we
solve the system:
∇ f ( P) = λ∇ g( P) and g( P) = c
to locate all such P’s.
However, another way to determine whether ∇ f ( P) and ∇ g( P) are parallel is by their
cross product:

∇ f ( P) k ∇ g( P) if and only if ∇ f ( P) × ∇ g( P) = 0.

By solving the vector equation ∇ f ( P) × ∇ g( P) = 0 for P (instead of using Lagrange’s


Multiplier), try to redo Problems #3(a)(b)(c).
What is the limitation of this method when compared to Lagrange’s Multiplier?

Solution: Take (a) as an example, in which we have f ( x, y) = x + 2y and g( x, y) =


x2 + y2 . By direct computation, their gradients are:

∇ f = i + 2j, ∇ g = 2xi + 2yj

and their cross product is given by ∇ f × ∇ g = (−4x + 2y)k. By setting ∇ f × ∇ g = 0,


we get −4x + 2y = 0, and so: y = 2x.
constraint equation x2 + y2 = 4, we have x2 + (2x )2 = 4, whose solutions
Using the √
are x = ± 4/5. Therefore, using y = 2x, we get:
√ √ √ √
( x, y) = ( 4/5, 2 4/5), (− 4/5, −2 4/5)

which are exactly the same as in #3(a).


Similar approaches can be applied to solve #3(b)(c).
However, since cross product is not defined in dimensions higher than 3, we cannot
apply this approach to maximize/minimize a function for 4 variables or higher.

5. (F) A closed rectangular water tank is to be made with three different materials. The
top part will be made by a thin material which costs $1 per cm2 . The four sides will use
stronger material which costs $2 per cm2 . To support the weight of water, the bottom of
the tank has to be made with very durable and strengthened material which costs $5 per
cm2 .
Suppose the volume of the water tank is to be 96cm3 . What dimensions of the tank will
minimize the cost of construction?

Solution: (Sketch) Let l be the length, w be the width and h be the height.

minimize: f (l, w, h) = |{z}


wl + 2(2hl ) + 2(2wh) + 5(wl ) = 6wl + 4hl + 4wh
| {z } | {z }
top sides base
subject to: g(l, w, h) = lwh = 96

Page 6
MATH 2023 Critical Points, Lagrange’s Multiplier Problem Set #4

Answer: (l, w, h) = (4, 4, 6) is the only solution to the Lagrange’s Multiplier system. It
must give the minimum, since the maximum does not exist. Given any fixed volume,
we can always construct a rectangular cube with arbitrarily large surface area.

6. (F) Find the point(s) on the cone:


q
z= x 2 + y2

that is/are closest to the point (1, 3, 1).

Solution: (Sketch) To ease computations, we rewrite the cone equation as:

z2 = x2 + y2 , or equivalently z2 − x2 − y2 = 0.

Let g( x, y, z) = z2 − x2 − y2 , then g = 0 is the constraint.

minimize: f ( x, y, z) = ( x − 1)2 + (y − 3)2 + (z − 1)2


subject to: g( x, y, z) = z2 − x2 − y2 = 0
 √ √ √   √ √ √ 
10+ 10 3(10+ 10) 1+ 10 10− 10 3(10− 10 1− 10
Answer: ( x, y, z) = 20 , 20 , 2 , 20 , 20 , 2 .
p
The second point is rejected since the original cone equation z = x2 + y2 requires
that z ≥ 0. It is straight-forward to check that:
√ √ √ !
10 + 10 3(10 + 10) 1 + 10 11 √
f , , = − 10
20 20 2 2

 √ √ √ 
3(10+ 10) 1+ 10
Therefore, the required point is 10+20 10 , 20 , 2 and the distance from this
q √
point to (1, 3, 1) is given by 11
2 − 10.

7. (FF) Consider the surface given by the equation

x2 y2 z2 = 1.

(a) Show that for any point ( a, b, c) on the surface, its tangent plane does not contain the
origin.

Solution: Let g( x, y, z) = x2 y2 z2 , then the surface is a level-set g( x, y, z) = 1. To


find the normal vector to the surface at ( a, b, c), we compute:

∇ g( x, y, z) = 2xy2 z2 i + 2x2 yz2 j + 2x2 y2 zk

Therefore, the normal vector to the required tangent plane can be taken to be:

n = ab2 c2 i + a2 bc2 j + a2 b2 ck

Page 7
MATH 2023 Critical Points, Lagrange’s Multiplier Problem Set #4

As the plane passes through ( a, b, c), the equation of the tangent is then given by:

ab2 c2 x + a2 bc2 y + a2 b2 cz = ab2 c2 · a + a2 bc2 · b + a2 b2 c · c

As ( a, b, c) lies on the surface x2 y2 z2 , we can assume that all of a, b and c are non-
zero (otherwise if any of them are zero, we would get 0 = 1). After simplification,
we get:
x y z
ab2 c2 x + a2 bc2 y + a2 b2 cz = 3a2 b2 c2 =⇒ + + = 3.
a b c
By substituting ( x, y, z) = (0, 0, 0), we get LHS = 0 whereas RHS = 3. Therefore,
(0, 0, 0) is not contained on the tangent plane.

(b) Find all points ( a, b, c) on the surface such that the tangent plane at these points are
closest to the origin.

Solution: By Q5 in Problem Set #1, the distance between the plane x


a + yb + zc = 3
and the origin (0, 0, 0) is given by:

3
d= q
1 1 1
a2
+ b2
+ c2

Since ( a, b, c) lies on the given surface, they are subject to the constraint a2 b2 c2 =
1. Therefore, the distance is also given by:

3 3
d= q =√
b2 c2 + a2 c2 + b2 c2 b2 c2 + a2 c2 + b2 c2
a2 b2 c2

To minimize d is equivalent to maximize a2 b2 + b2 c2 + c2 a2 . Let:

f ( a, b, c) = a2 b2 + b2 c2 + c2 a2 (to be maximized)
2 2 2
g( a, b, c) = a b c = 1 (constraint)

Solving Lagrange’s Multiplier system ∇ f = λ∇ g and g = 1, we can get these


candidate points:
( a, b, c) f ( a, b, c)
(−1, −1, −1) 3
(−1, −1, 1) 3
(−1, 1, −1) 3
(−1, 1, 1) 3
(1, −1, −1) 3
(1, −1, 1) 3
(1, 1, −1) 3
(1, 1, 1) 3
3

In all of the above cases, we have d = √3 = 3. Therefore, they are all the
maximum points for f and hence are also points on the surface which their
tangent plane closest to the origin.

Page 8
MATH 2023 Critical Points, Lagrange’s Multiplier Problem Set #4

8. (FF) Suppose
f ( x, y) = 2x2 + xy − 8x − y + 6
Let T be the triangular region (boundary included) in the xy-plane with vertices (0, 0),
(0, 3) and (3, 0).
(a) Find all the interior critical point(s) of f in the region T.

Solution: ∇ f = h4x + y − 8, x − 1i. Set ∇ f = 0, we get x = 1 and y = 4. The


only possible interior critical point for f is ( x, y) = (1, 4), which is beyond the
range of T. Therefore, there is no interior critical point in the region T.

(b) Find the maximum and minimum values of f ( x, y) when ( x, y) is restricted on the
vertical side of T, i.e. the line segment joining (0, 0) and (0, 3).

Solution: The vertical side of T is given by the equation x = 0. Substitute this


into the function, we get:
f (0, y) = −y + 6
which is a strictly decreasing function of y. Therefore, on the vertical side, the
maximum and minimum of f are attained at the end points (0, 0) and (0, 3).
Since f (0, 0) = 6 and f (0, 3) = 3. The maximum value of f is 6 and minimum
value is 3 when restricted on this vertical side.

(c) Find the maximum and minimum values of f ( x, y) when ( x, y) is restricted on the
horizontal side of T, i.e. the line segment joining (0, 0) and (3, 0).

Solution: The horizontal side has equation y = 0. Regarding x as the variable,


the function is given by:
f ( x, 0) = 2x2 − 8x + 6.
d
Since dx f ( x, 0) = 4x − 8, the only critical point is x = 2. Compute f at this critical
point and the end-points of the horizontal sides, we get: f (2, 0) = −2, f (0, 0) = 6
and f (3, 0) = 0. Therefore the max of f is 6 and the min is −2 when restricted
on the horizontal side.

(d) Write the equation of the line joining the vertices (0, 3) and (3, 0). Hence find the
maximum and minimum values of f ( x, y) when ( x, y) is restricted to the hypotenuse
of T.

Solution: The equation of the hypotenuse is x + y = 3. Let g( x, y) = x + y, then


the hypotenuse is the level set g = 3. Using Lagrange’s Multiplier:

f x = λgx 4x + y − 8 = λ
f y = λgy x−1 = λ
g( x, y) = 3 x+y = 3

From the first two equations, we get 4x + y − 8 = x − 1 (Be flexible! There is no


need to divide cases for this system!) Hence, 3x + y = 7. Combining with the
third equation x + y = 3, we get x = 2 and y = 1. The only critical point on
the hypotenuse is ( x, y) = (2, 1). Check the values of f at the critical point and

Page 9
MATH 2023 Critical Points, Lagrange’s Multiplier Problem Set #4

end-points, we get:

f (2, 1) = −1, f (3.0) = 0, f (0, 3) = 3.

Therefore, the maximum of f is 3 and the minimum is −1 when restricted to the


hypotenuse.

(e) Determine the absolute maximum and minimum of f ( x, y) over the domain T.

Solution: Since f has no interior critical point in the region T, its absolute max
and min must take place on the boundary of the region, i.e. the three sides of the
triangle. From previous parts, we got:
side max min
vertical 6 3
horizontal 6 -2
hypotenuse 3 -1
Therefore, the absolute maximum of f is 6 and the absolute minimum is −2 over
the region T.

9. (FF) This purpose of this problem


938
isChapter
to explain why the Second Derivative Test works
13 • Functions of Several Variables
for two-variable functions. Let f ( x, y) be a C2 function with a critical point P0 ( a, b). Given
any unit direction û = u1 i + u2 j, the path:

x = a + tu1 ,
13.6
y = b + tu
Directional Derivatives and
2
Partial derivatives tell us a lot about the rate of cha
is a straight-line passing through ( a, b) in the direction of û. ever,
As such,
they dothe
not intersection
directly answer some important
curve of the vertical plane shown in the diagram below can be are as:a point 1a, b, f1a, b22 on the surfa
standing at
expressed
fx and fy tell you the rate of change (or slope) of th
z = f ( a + tu1 , b + tu2 ) parallel to the x-axis and y-axis, respectively. But y
directions from that point and find a different rate
observation in mind, we pose several questions.
z • Suppose you are standing on a surface and you w
dinate direction—say, northwest or south-southea
function in such a direction?
• Suppose you are standing on a surface and you re
(a,, b,
b, f (a,
(a b)) In which direction will it roll?
z  f (x, y) • If you are hiking up a mountain, in what direction
you want to follow the steepest path?
These questions will be answered in this section a
tive, followed by one of the central concepts of cal
x P0(a, b)
u y
Directional Derivatives
Unit vector u
Let 1a, b, f1a, b22 be a point on the surface z = f
FIGURE 13.63 xy-plane (Figure 13.63). Our aim is to find the ra
P01a, b2. In general, this rate of change is neither f

dz
Therefore, the first derivative dt 兩u兩 ⫽measures the slope of tangent at ( a, b, f ( a, b)), whereas
t =0 1 u or u = 8 0, 1 9 ), but it turns out to be a combinatio
2
the second derivative dt2 d z
indicates whether u ⫽ sin
2 the curve is concaveFigure
up 13.64a
or downshows the unit vector u at an an
around
t =0 nents are u = 8 u 1, u 2 9 = 8 cos u, sin u 9 . The deri
the point ( a, b, f ( a, b)). u1 ⫽ cos the line / in the xy-plane through P0 in the directio
(a) h units from P0 along /, has coordinates P1a + h co

Now imagine the plane Q perpendicular to the
y P(a ⫹ h cos , b ⫹ h sin ) the surface z = f1x, y2 in a curve C. Consider two
they have z-coordinates f1a, b2 and f1a + h cos u,
u
Page 10 h sin of the secant line between these points is
h
f1a + h cos u, b + h sin
P0(a, b) h cos h
u ⫽ 具u , u 典 ⫽ 具cos , sin 典 The derivative of f in the direction of u is obtaine
MATH 2023 Critical Points, Lagrange’s Multiplier Problem Set #4

(a) Using the chain rule, show that:

dz
= f x u1 + f y u2
dt
d2 z
= f xx u21 + 2 f xy u1 u2 + f yy u22
dt2

Solution: Given that z = f ( a + tu1 , b + tu2 ), then f is a function of ( x, y), and


( x, y) is a function of t. Precisely, we have: x = a + tu1 and y = b + tu2 (note that
a, b, u1 and u2 are constants). Using the chain rule, we get:

dz ∂ f dx ∂ f dy
= +
dt ∂x dt ∂y dt
d d
= f x ( a + tu1 ) + f y (b + tu2 )
dt dt
= f x u1 + f y u2
d2 z
 
d dz
=
dt2 dt dt
d 
= f x u1 + f y u2 (from above)
dt
d fx d fy
= u1 + u2
dt dt 
∂ f x dx ∂ f x dy
= + u1
∂x dt ∂y dt
∂ f y dx ∂ f y dy
 
+ + u2
∂x dt ∂y dt

= f xx u1 + f xy u2 u1 (recall that x = a + tu1

+ f yx u1 + f yy u2 u2 and y = b + tu2 )
= f xx u21 + f xy u2 u1 + f yx u1 u2 + f yy u22
= f xx u21 + 2 f xy u1 u2 + f yy u22 (Mixed Partials Theorem)

Page 11
MATH 2023 Critical Points, Lagrange’s Multiplier Problem Set #4

(b) By completing-the-square, show further that:


  2  f f − f 2  
f xy
 f xx u1 + f u2 + xx yy2 xy u22
 if f xx 6= 0
 xx f xx
d2 z

  2  f f − f 2  
= f f xy xx yy xy
u21 if f yy 6= 0
dt2 
 yy f yy u1 + u2 + 2
f yy


2 f xy u1 u2 if f xx = f yy = 0

Solution: If f xx 6= 0, then:

d2 z
= f xx u21 + 2 f xy u1 u2 + f yy u22
dt2
2 f xy f yy 2
 
2
= f xx u1 + u1 u2 + u
f xx f xx 2
" 2  2 #
2 f xy f xy f xy f yy 2

2
= f xx u1 + u1 u2 + u2 − u2 + u
f xx f xx f xx f xx 2
" ! #
2 2
f xy
f xy f yy
= f xx u1 + u2 + − 2 + u22
f xx f xx f xx
" ! #
2
f xx f yy − f xy2
f xy
= f xx u1 + u2 + 2
u22
f xx f xx

For the case f yy 6= 0, we can obtain the result using symmetry argument: by
switching x with y, and u1 with u2 .
d2 z
The last case is trivial (since dt2
= f xx u21 + 2 f xy u1 u2 + f yy u22 ).

(c) The nature of the point ( a, b, f ( a, b)) can be determined by the following argument:
2 > 0 and f d2 z
i. If f xx f yy − f xy xx > 0, what can you say about dt2
? Using this observa-
tion, conclude the nature of the point ( a, b, f ( a, b)).

Solution: In this case, we have:


 
!
2 2
f xx f yy − f xy
d2 z
 
 f xy 2 

= f xx
 u 1 + u 2 + u 2
dt2 |{z} | f xx 2
f xx |{z}  
+ {z } | {z } + or 0
+ or 0 +

d2 z d2 z
and therefore dt2
≥ 0 for any u1 and u2 . The only situation with dt2
= 0 is
f xy
that both u1 + f xx u2= 0 and u2 = 0. However, it would imply u1 = u2 = 0,
which contradicts to the fact that u = u1 i + u2 j is a unit (hence non-zero)
vector.
2
Therefore, ddt2z > 0 for any unit direction u. The graph z = f ( x, y) is concave
up in all directions u, and so the critical point must be a local minimum.

Page 12
MATH 2023 Critical Points, Lagrange’s Multiplier Problem Set #4

2 > 0 and f
ii. How about if f xx f yy − f xy xx < 0?

Solution: In this case we have:


 
!
2 2
f xx f yy − f xy
d2 z
 
 f xy 2

2
= f xx  u1 + u2 + 2
u2 
dt |{z}  f xx f xx |{z}  
− | {z } | {z } + or 0
+ or 0 +

d2 z
and therefore ≤ 0. By similar discussion as in (i), the only situation in
dt2
d2 z
which dt2
= 0 is that u1 = u2 = 0 (that is impossible for a unit vector u).
2
Therefore, ddt2z < 0 for any unit vector u. The graph z = f ( x, y) is concave
down in all directions u, and so the critical point must be a local maximum.

2 > 0 and f
iii. How about if f xx f yy − f xy xx = 0?

2 > 0.
Solution: This is not possible. Otherwise we would have − f xy

2 < 0?
iv. What if f xx f yy − f xy

Solution: In the case f xx 6= 0, we have:


 
! 
2 2
f xx f yy − f xy
d2 z

f xy
u22 
 
2
= f xx  u1 + u2 + 2
dt 
| f xx f xx 

{z } | {z }
+ or 0 − or 0

and in the case f yy 6= 0, we have:


 
! 
2 2
f xx f yy − f xy
d2 z

 f xy 2

= f yy
 u 1 + u 2 + u 1
dt2 | f yy {z
 2
f yy 

} | {z }
+ or 0 − or 0

2
In either case, whether ddt2z is positive or negative depends on the choice of
u1 and u2 . In other words, the graph z = f ( x, y) is concave up in some
directions and concave down in some other directions. The critical point is a
saddle.
2
In the last case where f xx = f yy = 0, we then have ddt2z = 2 f xy u1 u2 . Therefore,
whether it is positive or negative depends only the choice of u. The critical
point is again a saddle.

Page 13
3 -2 2 3 0
40 45 215
= + = . Simplify.
3 2 6

Related Exercises 75–80

QUICK CHECK 4 Consider the triangle R with vertices 1-1, 02, 11, 02, and 10, 12 as a re-
MATH
gion 2023 •IfSpring
of integration. 2015-16
we integrate • Multivariable
first with respect to x, doesCalculus
R need to be subdivided?

Problem Set #5 • Double Integrals
If we integrate first with respect to y, does R need to be subdivided?

1. (F) Set-up the lower and upper bounds of each double integral below using both dxdy
TION 14.2 EXERCISES and dydx orders. Compute the integral using both orders and verify that they give the
same value.
ew Questions ¨ Basic Skills
Describe and sketch a region that is bounded above and below by 7–8. Regions of integration Consider the regions R shown in the
wo curves.
(a) 2xy dA where R is the region as shown below:
figures and write an iterated integral of a continuous function f over R.
R
Describe and a sketch a region that is bounded on the left and on 7. y 8. y y  2x  24
he right by two curves.
10
40
Which order of integration is preferable to integrate f 1x, y2 = xy (2, 8)
over R = 5 1x, y2: y - 1 … x … 1 - y, 0 … y … 1 6 ? y  4x (4, 32)

Which order of integration would you use to find the area of


y  x3 R
he region bounded by the x-axis and the lines y = 2x + 3 and R y  2x2
2 10
y = 3x - 4 using a double integral?
1 1y
Change the order of integration in the integral 10 1y2 f 1x, y2 dx dy. 1 x 6 2 2 6 x
2 4
Sketch the region of integration for 1-2 1x2 e xy dy dx.
Solution:
¨ ˆ y=8 ˆ x =y1/3
2xy dA = 2xy dxdy
y
R y =0 x= 4
ˆ y =8  x=y1/3
x2 y

= x= 4
y dy
y =0
ˆ y =8
y2
 
2/3
= y ·y− ·y dy
y =0 16
ˆ y =8 
y3

5/3
= y − dy
y =0 16
= 32
¨ ˆ x =2 ˆ y=4x
2xy dA = 2xy dydx
R x =0 y= x3
ˆ x =2
y=4x
xy2

= y= x3
dx
x =0
ˆ x =2
x · 16x2 − x · x6 dx

=
x =0
ˆ x =2
16x3 − x7 dx

=
x =0
= 32

Page 1
gion of integration. If we integrate first with respect to x, does R need to be subdivided? x = 6 - y, and

1 centered at the origin.
If we integrate first with respect to y, does R need to be subdivided?
39–46. Evaluating in
16. R is the region in the first quadrant bounded by the y-axis and the
evaluate the following
parabolas y = x 2 and y = 1 - x 2.
2 4-y
MATH 2023 Double
17–26. Integrals
Evaluating Problem
integrals Evaluate the following integrals as theySet #5
39. dx dy
are written. L-1 Ly
¨
Basic Skills
ounded above and below by 7–8. Regions of integration Consider the regions
1 1 R shown in the 1 2x 4 216 - y2
y= 2
(b) 1 dA
figures write an R
andwhere is the
iterated region
integral of abounded
17. between
6y dyfunction
continuous dx f over 18.+ 24 and
R. 2x 15yxy=2 2x :
dy dx 41. 2xy
R L0 Lx L0 L0 L0 L-216 - y2
bounded on the left and on 7. y 8. 2 2x y y  2x  24 3 x+6 ln 2 2
y
10 19. xy dy dx 20. 1x - 12 dy dx 43. dx dy
L0 Lx2 40 L0 Lx2 L0 Ley x
ble to integrate f 1x, y2 = xy (2, 8)
- y, 0 … y … 1 6 ? y  4x
p>4 cos x (4, 32) 1 21 - x2 p>2 p>2
21. dy dx 22. 2x 2y dy dx 45. 6 sin
u use to find the area of L-p>4 Lsin x L0 L-21 - x2 L0 Ly
y x3 R
the lines y = 2x + 3 and R 2 8 - x2 10 y  2x2 ln 2 2 p>2 cos y
2
23. x dy dx 24. dy dx 46. e sin y
1 1y L-2 Lx2 L0 Lex L0 L0
10 1y2 f 1x,
integral y2 dx dy. 1 x 6 2 2 6 x
1 x 1
3
p>2 x
2 4 2 47–52. Evaluating in
1-2 1x2 e xy dy dx. 25. 2e x dy dx 26. y cos x 3 dy dx
L0 L0 L0 L0 evaluate the following
Solution: (Answer only) 27–30. Evaluating integrals Evaluate the following integrals. 47. 4R 12y dA; R is
√ yR is bounded by
ˆ 27. ˆ xy dA; ˆ x = 0,ˆy = √ 2xy + 1, and 48. 4R y 2 dA; R is b
y=184R x = 2 y=32 x= 2
dxdy order: y = - 2x + 1
5. dxdy + 1 dxdy
√y y−24
49. 4R 3xy dA; R is
=0
y28. =−+ y2
x1x y=18 in the
2 dA; R is the region x =first
2 quadrant bounded by in the first quadr
ˆ x=4 x4
ˆ=R y0,
=2xy = x , and y = 8 - x .
+ 24 2 2
50. 4R 1x + y2 dA;
dydx order: 1 dydx
x29.
=−34Ry=
2
y 2xdA;
2 R is bounded by x = 1, y = 2x + 2, and 51. 4R 3x 2 dA; R is
y = -x - 1.
343 52. 4R x 2 y dA; R i
Answer: 2
330. 4R x y dA; R is the region in quadrants 1 and 4 bounded by the
semicircle of radius 4 centered at (0, 0). 53–56. Volumes Use
¨ following regions.
31–32. Regions of integration Write an iterated integral of a continu-
(c) x2 dA where R is the region bounded between y = 2x, y = 3x − 9 and the x-axis:53. The tetrahedron
R ous function f over the region R shown in the figure.
1x = 0, y = 0, z
31. y 32. y
20 54. The solid in the f
x  兹25  y2 the surface z =
(9, 18)
y  2x
10 2 R 55. The segment of t
y  3x  9 plane z = 12 +
R 2 6 x
x 56. The solid beneat
4 10
(3, 4) R = 5 1x, y2: 0
6 y  3x  5

Solution: (Answer only)


ˆ y=18 ˆ x =
y +9
3
dxdy order: x2 dxdy
y
y =0 x= 2
ˆ x =3 ˆ y=2x ˆ x =9 ˆ y=2x
dydx order: x2 dydx + x2 dydx
x =0 y =0 x =3 y=3x −9
1053
Answer:
2

Page 2
MATH 2023 Double Integrals Problem Set #5
¨ q
2. (F) Evaluate the integral a2 − y2 dA where T is the triangle with vertices (0, 0),
T
(0, a) and ( a, a). Set-up the integral in both dxdy and dydx orders, and choose the easier
one to compute.

Solution:
1.0

0.8

0.6

0.4

0.2

0.0
0.0 0.2 0.4 0.6 0.8 1.0

Below is the case where a ≥ 0. The case where a < 0 is similar (but different).
¨ q ˆ x = a ˆ y= a q
2 2
a − y dA = a2 − y2 dydx
T x =0 y= x
ˆ y= a ˆ x =y q
= a2 − y2 dxdy
y =0 x =0

Easier to integrate using dxdy-order:


ˆ y= a ˆ x =y q ˆ y= a  q  x =y
a2 − y2 dxdy = x a2 − y2 dy
y =0 x =0 y =0 x =0
ˆ y= a q
= y a2 − y2 dy
y =0
ˆ y= a
1
q
=− a2 − y2 d ( a2 − y2 )
2 y =0
 y= a
1 2 2 3/2
=− a − y2
2 3 y =0
1
= a3
3

Page 3
MATH 2023 Double Integrals Problem Set #5

ˆ 1 ˆ x1/3 q
3. (FF) Consider the integral 1 − y4 dydx. It is almost impossible to compute
0 x
the inner integral. Try to switch the order of integration to evaluate it. [Hint: You should
first sketch the region of integration.]

Solution:
1.0

0.8

0.6

0.4

0.2

0.0
0.0 0.2 0.4 0.6 0.8 1.0

According to the region represented by the integral, switching the integration order
gives:
ˆ 1 ˆ x1/3 q ˆ 1 ˆ x =y q
1 − y4 dydx = 1 − y4 dxdy
0 x 0 x = y3
ˆ 1  q  x =y
= x 1−y 4 dy
0 x = y3
ˆ 1 q q 
4 3 4
= y 1−y −y 1−y dy
0
ˆ 1 q ˆ 1 q
3
= y 1 − y4 dy − y 1 − y4 dy
0 0
π 1
= −
8 6
Remark: Let u = y2 for the first integral, and let v = 1 − y4 for the second integral
(detail omitted).

Page 4
MATH 2023 Double Integrals 14.2 Double Integrals
Problem Setover
#5 General Regions
¨ Reverse the order of¨
57–62. Changing order of integration integra- 70. The solid above the z
1 1
4. (FF
tion) in
Evaluate theintegrals.
the following integrals dA and dA. Try region integration-by-
to avoid
parabolic
2 2x R 3 −3 x 6 - 2x R y−6 R = 5 1x , y2: 0 … x … 1,
parts
57.if possible.
f 1x, y2 dy dx 58. f 1x, y2 dy dx 0 … y … 1 - x 2 6 and
L0 Lx 2 L0 L0 between the planes z = 1 z
y y
and z = 2 - y

(2, 4)
y  2x
y  6  2x
z
y  x2
1 R R
1
0 x
1 0 x R
1
1 -ln y 1 ey
59. f 1x, y2 dx dy 60. f 1x, y2 dx dy x
L1>2 L0 L0 L1
1 cos-1 y e ln x 71. The solid bounded by the parabo- z
Solution:
61. f 1x, y2 dx dy 62. f 1x, y2 dy dx loid z = x 2 + y 2 and the plane
L0 L0 L1 L0 ˆ x =3 ˆ y=6−2x
¨ z = 9
1 1
63–68. Changing order of integration The =
dAfollowing integrals can dydx
be evaluated only by reversing R
3−x y=0the re-3 − x
x =0 Sketch
the order of integration.
ˆ xand
gion of integration, reverse the order of integration, y they=6−2x
=3 evaluate
 
integral. = dx
1 1 p p x =0
3 − x y =0
63.
2
e x dx dy 64. ˆsinxy=2 3dy dx
L0 Ly L0 L
6 − 2x
=x dx
x =0 3 − x
ˆ x =3
1>2 1>4
65. y cos 116px 22 dx dy 2(3 − x )
L0 Ly 2
= dx
x =0 (3 − x )
4 2
x ˆ x =3 72. The solid bounded by the pa-
66. dy dx
L0 L1x y + 1
5
= 2 dx = 6 raboloids z = x 2 + y 2 and
1p
3 3
1p x =0 z = 50 - x 2 - y 2
67. x cos 1x y2¨dx dy 1
4 ˆ y =6 ˆ x = 6− y
2
2 1
L0 Ly
dA = dxdy
2 4 - x2 R y − 6 y = 0 x = 0 y − 6
xe 2y
68. dy dx ˆ y =6   x= 6−2 y
L0 L0 4 - y x
= dy
69–74. Regions between surfaces Find the volumeyof
y − 6 x =0
=0the following
solid regions.
ˆ y =6
1
z
= − dy = −3.
69. The solid above the region y =0 2
R = 5 1x , y2: 0 … x … 1,
0 … y … 1 - x6
73. The solid above the region z
bounded by the parabo- 2x  y 
loids z = x 2 + y 2 and z2 x2  y2
R = 5 1x, y2: 0 … x … 1,
z = 2 - x 2 - y 2 and the 0 … y … 2 - x 6 and between
coordinate planes in the the planes - 4x - 4y + z = 0
first octant and -2x - y + z = 8

4x  4
1
z  x2  y2
R
R 1
y
y
x
x

Page 5
z
In polar coordinates, the upper bounding surfac
z  8  x2  y 2 lower bounding surface is z = r 2. The volume
Intersection 2p 2
curve C
MATH 2023 Double Integrals V = Set #5 a 18 - r 22 - r
Problem L0 L0 (+)+* ()*

upper lowe
¨ 2p 2
5. (F) Evaluate ( x + y) dA using polar coordinates
C where R is the region in the
= first 18r - 2r 32 dr d
R √ L0 L0
quadrant lying inside the disk x2 + y2 ≤ a2 and under the line y = 3x. 2p 4 2
r
= a 4r 2 - b ` du
y 4 x2 z x2  y2 L0 2 0
Solution: (Sketch only) 2p
R
= 8 du
1.0 2 2 L0
x y
Projection of C y 4  x2 = 16 p.
on xy-plane
0.5

FIGURE 14.32
EXAMPLE 3 Annular region Find the vo
0.0 z = xy + 10 and above the annular region R =
(An annulus is the region between two concen

-0.5
SOLUTION The region of integration suggests
(Figure 14.33). Substituting x = r cos u and y
xy + 10 = 1r cos u21r sin u2
-1.0
-1.0 -0.5 0.0 0.5 1.0
= r 2 sin u cos u + 10
= 12 r 2 sin 2u + 10.
Substituting the integrand into the volume inte
¨ ˆ θ = π3 ˆ r=a 2p 4
( x + y) dA = (r cos θ + r sin θ ) rdrdθ V = 1 12 r 2 sin 2u + 10 2 r dr
R θ =− 2π
3 r =0 L0 L2
ˆ θ = π3 ˆ r=a
√ 2p 4
3−1 3
= 2
r (cos θ + sin θ ) drdθ = a = 1 12 r 3 sin 2u + 10r 2 dr
θ =− 2π
3 r =0 3 L0 L2
2p
r4 4
¨ = a sin 2u + 5r 2 b ` du
 L0 8 2
6. (F) Consider the annular region R below. Express the integral x2 + y2 dA2pin both
R
rectangular and polar coordinates. Choose the easier system to compute the = 130 sin 2u + 602 du
integral.
L0
2p
y = 1151-cos 2u2 + 60u2 ` = 12
4 r2 0
r 4

More General Polar Regions


4 4 x In Section 14.2 we generalized double integra
R grals over nonrectangular regions. In an analog
polar rectangle may be extended to more gene
4 two rays u = a and u = b, where b - a … 2
R  {(r, ␪): 2  r  4, 0  ␪  2␲} (Figure 14.34):
FIGURE 14.33 R = 5 1r, u2: 0 … g1u2 …
Solution: See Solutions to Worksheet #11 Q5 and Worksheet #12 Q2. Obviously it is
easier to compute it using polar coordinates.

Page 6
MATH 2023 Double Integrals Problem Set #5

7. (FF) Evaluate each of the following integrals:


ˆ 2π ˆ 1
2
(a) e− x sin y dxdy
0 0

2
Solution: Since it is impossible to integrate e− x , we try to switch the order of
integration to see if there is any luck. The bounds are all constants, so we simply
switch the integral signs:
ˆ 2π ˆ 1 ˆ 1 ˆ 2π
− x2 2
e sin y dxdy = e− x sin y dydx
0 0 0 0
ˆ 1 h iy=2π
2
= −e−x cos y dx
0 y =0
ˆ 1  
− x2 − x2
= −e +e dx
0
ˆ 1
= 0 dx = 0
0

ˆ 0 ˆ √ y +1  5/2
x3
(b) x− dxdy
−1 0 3

Solution: It is very difficult to integrate the given function by dx, we try to switch
the order of integration to see if there is any luck. Note that the bounds involve
variables, so we have to sketch the diagram of the domain first. In the p dxdy-order,
the x-sample strips enter the region from x = 0 and leaves from x = y + 1 (i.e.
part of the parabola y = x2 − 1), see the red strip in the diagram.
To switch order, we draw a sample blue strip in the diagram, which enters the
region at y = x2 − 1 and leaves at y = 0. The minimum value of x is 0 and the
maximum of x is 1. Therefore, after switching dx and dy, we get:
ˆ 0 ˆ √ y +1  5/2 ˆ 1ˆ 0  5/2
x3 x3
x− dxdy = x− dydx
−1 0 3 0 x 2 −1 3
ˆ 1 " 5/2 #y=0
x3
= x− y dx
0 3 2 y = x −1
ˆ 1 3 5/2

x
= x− (1 − x2 ) dx
0 3
ˆ u=2/3
= u5/2 du
u =0

x3
where we let u = x − 3, then du = (1 − x2 )dx. Keep going:
ˆ u=2/3  u=2/3  7/2
5/2 2 7/2 2 2
u du = u = .
u =0 7 u =0 7 3

Page 7
MATH 2023 Double Integrals Problem Set #5

0.5

0.0

-0.5

-1.0

-1.5
-1.5 -1.0 -0.5 0.0 0.5 1.0 1.5

¨
1
(c) dA where Q is the entire first quadrant of the xy-plane
Q (1 + 2x2 + 2y2 )3

Solution: The term x2 + y2 appears in the integrand, so it is typically better


to be done in polar coordinates. The first quadrant Q is represented in polar
coordinates by 0 ≤ r < ∞ and 0 ≤ θ ≤ 2π. Therefore,
¨ ˆ π ˆ ∞
1 2 1
dA = rdrdθ
Q (1 + 2x2 + 2y2 )3 0
2 3
0 (1 + 2r )
ˆ π  r = ∞
2 1
= − dθ
0 8(1 + 2r2 )2 r=0
ˆ π  
2 1 π 1
= −0 + dθ = ·
0 8 2 8
π
= .
16

¨
(d) ( x2 − y + 1) dA where Q is the region {( x, y) | x ≥ 0, y ≥ 0, x2 + y2 ≤ 4}
Q

Solution: The domain Q is a sector, so it’s again better be done by polar coor-
dinates. In polar coordinates, the domain Q is represented by 0 ≤ r ≤ 2 and
0 ≤ θ ≤ π2 . Therefore,
¨ ˆ π/2 ˆ 2
( x2 − y + 1) dA = (r2 cos2 θ − r sin θ + 1) rdrdθ
Q 0 0
ˆ π/2 ˆ 2
= (r3 cos2 θ − r2 sin θ + r ) drdθ
0 0
8
= 2π −
3
The Half/Double-Angle Formula should be used to compute the integral for
cos2 θ.

Page 8
MATH 2023 Double Integrals Problem Set #5
¨
4 +2x2 y2 + y4 )
(e) ( x2 + y2 )e−(x dA
R2

Solution: It is useful to note that x4 + 2x2 y2 + y4 = ( x2 )2 + 2( x2 )(y2 ) + (y2 )2 =


( x2 + y2 )2 , and so it suggests that the integral had better be done by polar coor-
dinates. The domain R2 can be expressed in polar coordinates as 0 ≤ r < ∞ and
0 ≤ θ ≤ 2π:
¨ ˆ 2π ˆ ∞
2 −( x4 +2x2 y2 +y4 ) 2 2
2
( x + y )e dA = r2 e−(r ) rdrdθ
R2 0 0
ˆ 2π ˆ ∞
4
= r3 e−r drdθ
0 0
ˆ 2π " −r4 #r=∞
e
= − dθ
0 4
r =0
ˆ 2π  
1
= −0 + dθ
0 4
1 π
= 2π · = .
4 2

8. (FF) Some single-variable integrals are “notoriously” difficult to compute. One exam-
´ 2
ple is e− x dx despite the fact that this integral is of central importance in mathematics
(pure/applied), physics, statistics and engineering. However, some of these difficult inte-
grals can be evaluated via double integral methods.
This problem investigates another well-known integral which has no closed-form anti-
derivative: ˆ
log(1 − x )
dx.
x
The goal of this problem is to show that this integral over 0 ≤ x ≤ 1 can be written as an
infinite series.
Consider the function
1
f ( x, y) = , 0 ≤ x ≤ 1, 0 ≤ y ≤ 1.
1 − xy

It is defined almost everywhere on the rectangle 0 ≤ x ≤ 1 and 0 ≤ y ≤ 1 (we say ‘almost’


because it’s undefined only at ( x, y) = (1, 1), but this single point is negligible).

Page 9
MATH 2023 Double Integrals Problem Set #5

ˆ 1
1 log(1 − x )
(a) Show that: dy = − .
0 1 − xy x

Solution: Let u(y) = xy regarding x is a constant and y is the variable. Then


du = xdy. When y = 0, u = 0; when y = 1, u = x. Therefore, by u-substitution:
ˆ 1 ˆ x
1 1
dy = du
0 1 − xy 0 x (1 − u )
ˆ
1 x 1
= du
x 0 1−u
1
= [− log(1 − u)]uu==
x
0
x
1 log(1 − x )
= − (log(1 − x ) − log(1 − 0)) = − .
x x

(b) Note that | xy| < 1 except for the negligible point ( x, y) = (1, 1), so the function
f ( x, y) can be expressed as a geometric series:
1
= 1 + ( xy) + ( xy)2 + ( xy)3 + . . .
1 − xy
Using this geometric series, show that
ˆ 1ˆ 1 ∞
1 1 1 1
dydx = 1 + 2 + 2 + . . . = ∑ 2 .
0 0 1 − xy 2 3 k =1
k

Solution: By geometric series expansion:


1
= 1 + ( xy) + ( xy)2 + ( xy)3 + . . .
1 − xy
Integrate both sides:
ˆ 1ˆ 1
1
dydx
0 0 1 − xy
ˆ 1ˆ 1
= (1 + xy + x2 y2 + x3 y3 + . . .)dydx
0 0
ˆ 1ˆ 1 ˆ 1ˆ 1 ˆ 1ˆ 1 ˆ 1ˆ 1
2 2
= dydx + xydydx + x y dydx + x3 y3 dydx + . . .
0 0 0 0 0 0 0 0
ˆ 1 ˆ 1  y =1 ˆ 1  y =1 ˆ 1  y =1
y2 y3 y4
  
y =1
= [y]y=0 dx + x· dx + x2 · dx + x3 · dx + . . .
0 0 2 y =0 0 3 y =0 0 4 y =0
ˆ 1 ˆ ˆ 1 2
1 ˆ 1 3
x x x
= dx + dx + dx + dx + . . .
0 0 2 0 3 0 4
 x =1   x =1   x =1
1 x2 1 x3 1 x4

x =1
= [ x ] x =0 + · + · + · +...
2 2 x =0 3 3 x =0 4 4 x =0
1 1 1
= 1+ 2 + 2 + 2 +...
2 3 4

1
= ∑ 2.
k =1
k

Page 10
MATH 2023 Double Integrals Problem Set #5

(c) Using (a) and (b), show that


ˆ 1 ∞
log(1 − x ) 1
− dx = ∑ 2 .
0 x k =1
k

Solution: Simply combine the results obtained in (a) and (b):


ˆ 1 ˆ 1 !

1 1
dy dx = ∑ 2 (from (b))
0 0 1 − xy k =1
k
ˆ 1 ∞
log(1 − x )

1
− dx = ∑ 2 (from (a))
0 x k =1
k
ˆ 1 ∞
log(1 − x ) 1
− dx = ∑ 2 .
0 x k =1
k

(d) Using the above approach, mutatis mutandis, show that for any 0 ≤ z ≤ 1, we have:
ˆ z ∞
log(1 − x ) zk
− dx = ∑ 2 .
0 x k =1
k

[Remark: Mutatis mutandis is a Latin phrase meaning “changing only those things
which need to be changed”.]
´1 ´z ´1
Solution: Replace the outer 0 by 0 (while keeping the inner 0 unchanged):
ˆ zˆ 1
1
dydx
0 0 1 − xy
ˆ zˆ 1
= (1 + xy + x2 y2 + x3 y3 + . . .)dydx
0 0
ˆ zˆ 1 ˆ zˆ 1 ˆ zˆ 1 ˆ zˆ 1
2 2
= dydx + xydydx + x y dydx + x3 y3 dydx + . . .
0 0 0 0 0 0 0 0
ˆ z ˆ  y =1
z ˆ z 3 y =1 ˆ z 4 y =1
y2
  
y =1 2 y 3 y
= dx +
[ y ] y =0 x· dx + x · dx + x · dx + . . .
0 0 2 y =0 0 3 y =0 0 4 y =0
ˆ z ˆ z ˆ z 2 ˆ z 3
x x x
= dx + dx + dx + dx + . . .
0 0 2 0 3 0 4
 x =z   x =z   x =z
1 x2 1 x3 1 x4

x =z
= [ x ] x =0 + · + · + · +...
2 2 x =0 3 3 x =0 4 4 x =0
z2 z3 z4
= z+ + + +...
22 32 42

zk
= ∑ 2.
k =1
k

From (a), we get:


ˆ z ∞
log(1 − x ) zk
− dx = ∑ 2 .
0 x k =1
k

Page 11
MATH 2023 Double Integrals Problem Set #5

9. (FFF) The purpose of this problem is to use double integrals to derive a somewhat
surprising result in electrostatics, that is the electric force exerted on a charged particle
by an infinite sheet of uniformly distributed charges is independent of how far the particle
and the sheet are apart from each other.
The paragraphs below describe the physical set-up of the problem. Although it may
be possible to proceed to the problem without knowing the physics background, it is
strongly recommend to read through the paragraphs below so as to understand the mo-
tivation of this problem.
According to the Coulomb’s Law, the electric force F exerted on a point particle with
charge Q located at ( x0 , y0 , z0 ), by a point particle with charge q located at ( x, y, z), is
given by:
qQ ( x0 − x ) i + ( y0 − y ) j + ( z0 − z ) k
F=
4πε 0 (( x0 − x )2 + (y0 − y)2 + (z0 − z)2 )3/2
where ε 0 is positive constant (depending on the medium).
The Coulomb’s Law is also called the Inverse Square Law because one can easily verify
that the magnitude of the force satisfies:

qQ
|F| =
4πε 0 d2
p
where d = ( x0 − x )2 + (y0 − y)2 + (z0 − z)2 is the distance between the two particles.
If there is a sequence of discrete charged particles located at ( x1 , y1 , z1 ), ( x2 , y2 , z2 ), . . .,
each with charge q, then the resultant electric force exerted on a particle with charge Q
located at ( x0 , y0 , z0 ), is given by the vector sum of all forces:

qQ ( x0 − x i ) i + ( y0 − y i ) j + ( z0 − z i ) k
F= ∑ 4πε 0 .
i =1 (( x0 − xi )2 + (y0 − yi )2 + (z0 − zi )2 )3/2

This is called the Principle of Superposition by physicists.


Now given there is an infinite sheet of uniformly distributed charges on the xy-plane,
and for each small area element dA on the xy-plane, the amount of charges is given by
σ dA, where σ is a constant that represents the area density of charges. Suppose there is
a particle with charge Q located above the xy-plane at (0, 0, z0 ), i.e. z0 > 0. For simplicity,
call this the Q-particle.
Now regard a small area element located at ( x, y, 0) on the xy-plane as a charged “parti-
cle” with charge q = σ dA, then the force exerted on the Q-particle by this area element
is given by substituting ( x, y, z) = ( x, y, 0) and ( x0 , y0 , z0 ) = (0, 0, z0 ):

Q(σ dA) (0 − x ) i + (0 − y ) j + ( z0 − 0) k Qσ − xi − yj + z0 k
3/2
= dA.
4πε 0 ((0 − x )2 + (0 − y)2 + (z0 − 0)2 ) 4πε 0 x2 + y2 + z2 3/2

0

Therefore, by the Principle of Superposition, the resultant electric force exerted on the
Q-particle by the sheet of charges is given by this double integral over the entire xy-plane
(i.e. R2 ): ¨
Qσ − xi − yj + z0 k
Fresultant = 3/2 dA.
R 2 4πε 0 x 2 + y2 + z2
0
Here integrating a vector simply means integrating each component of the vector treating
i, j and k as “constants”.

Page 12
MATH 2023 Double Integrals Problem Set #5

(a) Show that i and j-components of Fresultant are zero.

Solution: Since x2 + y2 = r2 , we use polar coordinates again to simplify our


calculations.

Fresultant
ˆ 2π ˆ ∞
Qσ −r cos θ i − r sin θ j + z0 k
= rdrdθ
0 0 4πε 0 (r2 + z20 )3/2
ˆ 2π ˆ ∞ ˆ 2π ˆ ∞
−r2 cos θ drdθ −r2 sin θ drdθ
 

= i+ j
4πε 0 0 0 (r2 + z20 )3/2 0 0 (r2 + z20 )3/2
ˆ 2π ˆ ∞  
z0 r drdθ
+ 2 2 3/2
k
0 0 (r + z0 )

Note that the integral for the i-component has constant bounds and the integrand
can be decomposed into a product of an r-function and a θ-function, i.e.
ˆ 2π ˆ ∞ ˆ 2π  ˆ R !
−r2 cos θ drdθ r2
= − lim cos θ dθ dr
0 0 (r2 + z20 )3/2 R→∞ 0
2 2 3/2
0 (r + z0 )

´ 2π
Since 0 cos θ dθ = 0, the i-component is zero. Similar argument applies to the
j-component.
Alternatively, you may argue that the integrand of the i-component:
x

( x2 + y2 + z20 )3/2

is an odd function of x, and the domain R2 is symmetric about the axis x = 0.


The positive contribution on the right of the axis exactly cancels out the negative
contribution on the left. Similar for the j-component.

(b) Derive that:



Fresultant =
k.
2ε 0
[Remark: The result in (b) asserts that the resultant force on the Q-particle does not
depend on how far it is from the infinite sheet! Believe it or not?]

Solution:
ˆ 2π ˆ ∞ ˆ 2π  ˆ ∞ 
z0 r drdθ z0 r
= dθ dr
0 0 (r2 + z20 )3/2 0 0 (r2 + z20 )3/2
 r = ∞
z0
= 2π − 2
(r + z20 )1/2 r=0
 
z0
= 2π −0 + = 2π
(0 + z20 )1/2
Combine the results with (a), we obtained:
Qσ Qσ
Fresultant = 2π k = k.
4πε 0 2ε 0

Page 13
MATH 2023 • Spring 2015-16 • Multivariable Calculus
Problem Set #6 • Triple Integrals

1. (F) Consider the triple integral: On as plane :


×=y
or
-

OTIYTZ
-0
ˆ 1ˆ 1ˆ x z
-
On yz
-
plane : y=
-
z

's '
shadow f ( x, y, z) dydxdz.


oonn
Elaine
"I!
?
× nn
0 z 0
on xz -
plan
(a) Sketch the solid described by the integral.

-
z

1
X = •

y=x z
-

(b) Express the integral using each of the other five orders, i.e. dydzdx, dxdydz, dxdzdy,
dzdxdy and dzdydx.

Solution: (Answer only)


ˆ 1ˆ 1ˆ x z ˆ 1ˆ x ˆ x z
f ( x, y, z) dydxdz = f ( x, y, z) dydzdx
0 z 0 0 0 0
ˆ 1ˆ 1 zˆ 1
= f ( x, y, z) dxdydz
0 0 y+z
ˆ 1ˆ 1 yˆ 1
= f ( x, y, z) dxdzdy
0 0 y+z
ˆ 1ˆ 1ˆ x y
= f ( x, y, z) dzdxdy
0 y 0
ˆ 1ˆ x ˆ x y
= f ( x, y, z) dzdydx
0 0 0

Page 1
MATH 2023 Triple Integrals Problem Set #6

2. (FF) Consider the triple integral:


ˆ 1ˆ 1ˆ x
3
e x dydxdz.
0 z 0

(a) Sketch the solid described by the integral.

x=y

(b) Pick a good order of integration and compute the integral by hand.

3
Solution: We use dzdydx-order since the integrand e x depends only on x. That
way we should be able to compute the inner- and middle integral easily. [Note:
dydzdx-order should works as well.]
Use z as the “pillar” variable, so that ( x, y) are the base variables. The triple
integral can be rewritten as:
ˆ 1ˆ x ˆ x ˆ 1ˆ x
x3 3
e dzdydx = xe x dydx
0 0 0 0 0
ˆ 1
3
= x2 e x dx
0
 x =1
1 x3
= e
3 x =0
1
= (e 1) .
3

Page 2
- 2 … y … 2, - 3 … z … 3 6

l
2
- y2
14. xyze -x dV; D = 5 1x, y, z2: 0 … x … 1ln 2,
D
19. The wedge above the xy-plane formed
6 x 2 + y 2 = 4 is cut by the planes z =
MATH 2023 0 … y … 1ln 4, 0 Triple
… z … 1Integrals Problem Set #6
z
15–24. Volumes of solids Find the volume of the following solids
3. (FF) Consider using triple integrals.
the right tetrahedron solid T in the first octant bounded by the xy-, yz-,
xz-planes and the
15. plane P with
The region in thevertices 6, 0, 0),by(0,
first octant(bounded the4,plane
0) and (0, 0, 2).
2x + 3y + 6z = 12 and the coordinate planes
z

) 20. The region bounded by the parabolic c


planes z = 3 - y and z = 0
z

16. The region in the first octant formed when the cylinder z = sin y,
(a) Show that the equation
for 0 … y …ofp,the plane
is sliced P is
by the given
planes y = by 2xx+= 3y
x and 0 + 6z = 12.

Solution: Straight-forward.

(b) Evaluate the following triple integral:


21. The region between the sphere x 2 + y
˚ ✓ ◆ hyperboloid z 2 - x 2 - y 2 = 1, for z
1 1 1
+ + dV. z
T 12 3y 6z 12 2x 6z 12 2x 3y

Please do the computations by hand. Pick carefully the orders of integration to sim-
plify your computations.
17. The region bounded below by the cone z = 2x 2 + y 2 and
2
bounded
Solution: Denote T above
thebyprojection
the sphere x of+Ty 2on z 2 =yz-plane.
+ the 8 Similar
for Txy and Txz .
yz
We split the integral into three and
z use different order of integration for each of
them: (0 0 !8)
! 2 2 2 8
x = 12 (12 3y 6z) 22. The region bounded by the surfaces z =
1 1
˚ ¨ ˆ
dV = dxdydz
rectangle 5 1x, y2: 0 … x … 1, 0 … y
T 12 3y 6z Tyz x =0 12 3y 6z
z
1 1 y2 1 4⇥2
¨
= dA = area of Tyz = =2
Tyz 2 2 2 2
¨ ˆ y= 1 (12 2x 6z)
1 1
˚
3
dV = dydxdz
T 12 2x 6z Txz y=0 12 2x 6z ln 2

1 1 6⇥2
¨
1
= dxdz = =2
Txz 3 3 2

z= 16 (12 2x 3y)
1 1
˚ ¨ ˆ
dV = dzdxdy
T 12 2x 3y Txy z =0 12 2x 3y
1 1 6⇥4
¨
= dxdy = =2
Txy 6 6 2

Therefore,
˚ ✓ ◆
1 1 1
+ + dV = 2 + 2 + 2 = 6.
T 12 3y 6z 12 2x 6z 12 2x 3y

Page 3
MATH 2023 Triple Integrals Problem Set #6

4. (FF) Let a be a positive constant. Given that f ( x ) is a continuous function of x, show


that: ˆ aˆ zˆ y ˆ a
( a x )2
f ( x ) dxdydz = f ( x ) dx
0 0 0 0 2

Solution: The triple integral represents the following solid:

xi y - rly=z

Note that the integrand f ( x ) depends only on x. We switch the order of integration
to: dzdydx (so that one can compute the inner and middle integrals without any
problem):
ˆ aˆ zˆ y ˆ aˆ aˆ a
f ( x ) dxdydz = f ( x ) dzdydx
0 0 0 0 x y
ˆ a ˆ a
= f (x) (a y) dydx
0 x
a  y= a
(a y )2
ˆ
= f (x) · dx
0 2 y= x
a
(a x )2
ˆ
= f (x) · dx
0 2

Page 4
MATH 2023 Triple Integrals Problem Set #6

p
˚
5. (F) Evaluate ( x2 + y2 ) dV over the solid D which lies above the cone z = c x 2 + y2
D
and inside the sphere x2 + y2 + z2 = a2 .

Solution: The cone can be expressed in spherical coordinates as:

1 1
r cos f = cr sin f =) f = tan .
| {z } | {z } c
z p2 2
c x +y

Hence, the solid D can be expressed in spherical coordinates as:

1 1
0  r  a, 0  f  tan , 0  q  2p.
c
Therefore, we have:
˚ ˆ 2p ˆ tan 1 1 ˆ a
c
2 2
( x + y ) dV = r2 sin2 f · r2 sin f drdfdq
D 0 0 0 | {z } | {z }
x 2 + y2 dV
ˆ 2p ˆ tan 1 1 ˆ a
c
= r4 sin3 f drdfdq
0 0 0
tan 1 1
2pa5
ˆ
c
= sin3 f df
5 0
tan 1 1
2pa5
ˆ
c
= cos2 f 1 d(cos f)
5 0
 tan 1 1
2pa5 cos3 f c
= cos f
5 3 0
✓ ◆
2pa5 1 1 1 2
= cos3 tan 1 cos tan 1
+
5 3 c c 3

You may use the fact that cos tan 1 x = p 1 to simplify the final answer, but it is
1+ x 2
not necessary.

p
6. (F) Find the volume of the solid bounded by the xy-plane, the cone z = 2a x 2 + y2
and the cylinder x2 + y2 = 2ay.

Solution: (Sketch only) In cylindrical coordinates, the cone is given by z = 2a r and


the cylinder is r2 = 2ar sin q, or equivalently, r = 2a sin q. Therefore,
q =p r =2a sin q z=2a r
2
˚ ˆ ˆ ˆ
volume = 1 dV = 1 r dzdrdq = (9p 16) a3
D q =0 r =0 z =0 9

Here we presented the case when a > 0. The other case is similar (yet different).

Page 5
MATH 2023 Triple Integrals Problem Set #6
✓ ◆
1 x 2 + y2 + z2
7. (FF) Let f( x, y, z) = exp where t > 0. Show that for each
(4pkt) 2
3
4kt
fixed t > 0, we have: ˚
f( x, y, z) dV = 1.
R3

Solution: The appearance of the term x2 + y2 + z2 suggests it may be best to use


spherical coordinates, since x2 + y2 + z2 = r2 . The bounds for the whole space R3 is:

0  r < •, 0  f  p, 0  q  2p.

2p ✓ ◆

1 p
r2
˚ ˆ ˆ ˆ
f( x, y, z) dV = 3/2
exp r2 sin f drdfdq
R 3 0 0 0 ( 4pkt ) 4kt
✓ˆ 2p ◆ ✓ˆ p ◆ ✓ˆ • ◆
1 2 r2 /4kt
= dq sin f df r e dr
(4pkt)3/2 0 0 0
✓ˆ • ◆
1 2 r2 /4kt
= · 2p · 2 · r e dr
(4pkt)3/2 0
ˆ •
2
It comes down to computing r2 e r /4kt dr. Note that
0
✓ ◆
d ⇣ r2 /4kt

r2 /4kt 2r r r2 /4kt
e =e · = ·e .
dr 4kt 2kt

ˆ • ˆ • ⇣ r ⌘
2 r2 /4kt 2
r e dr = 2kt r· e r /4kt dr
0 0 | 2kt {z }
⇣ ⌘
d e r2 /4kt

⇢h ir!• ˆ •
r2 /4kt r2 /4kt
= 2kt re e dr
r =0 0
⇢ p • p 2
✓ ◆
r
ˆ
= 2kt [0 0] 4kt e (r/ 4kt)
d p
0 4kt
p ˆ • p
u2 p
= 2kt 4kt e du = 4k3/2 t3/2 · .
0 2
p
The last two steps follows from integration by substitutions (let u = r/ 4kt) and
Lecture Notes P.64.
Combining these results, we get:
p
1 p
˚
3/2 3/2
f( x, y, z) dV = 3/2
· 2p · 2 · 4k t · = 1.
R3 (4pkt) 2

Alternatively, one can also breakdown f( x, y, z) into:

1 x2 /4kt y2 /4kt z2 /4kt


e e e
(4pkt)3/2
and set up the integral using rectangular coordinates.

Page 6
MATH 2023 Triple Integrals Problem Set #6

8. (FF) Consider a right circular solid cone (denoted by K) with radius R, height h, mass
m and uniform density d.

The moment of inertia about the z-axis of the solid is defined to be:
˚
Iz := Dz ( x, y, z)2 ddV
K
where Dz ( x, y, z) is the perpendicular distance between the point ( x, y, z) and the z-axis.
(a) Set up, but do not evaluate, the integral Iz using each of the following coordinates:
i. rectangular coordinates
ii. cylindrical coordinates
iii. spherical coordinates

Solution: Dz ( x, y, z) is the distance from ( x, y, z) to the z-axis, which is also the


distance from ( x, y, z) top(0, 0, z) – draw a picture to convince yourself on that!
Therefore, Dz ( x, y, z) = x2 + y2 .
The equation of the cone is given by:
h
r z= (cylindrical)
R
q
h
z= x 2 + y2 (rectangular)
R
R
j = tan 1 (spherical)
h
The equation of the flat top of the cone is given by:
z=h (both cylindrical and rectangular)
r = h sec q (spherical)

p
ˆ y= R ˆ x= R2 y2 ˆ z=h
Iz = p p d( x2 + y2 ) dzdxdy
y= R x= R2 y2 z= Rh x 2 + y2
ˆ q =2p ˆ r=R ˆ z=h
Iz = dr2 rdzdrdq
q =0 r =0 z= Rh r
ˆ q =2p ˆ r=R ˆ z=h
= dr3 dzdrdq
q =0 r =0 z= Rh r
ˆ q =2p ˆ j=tan 1 R ˆ r= h sec q
h
Iz = d(r2 sin2 j cos2 q + r2 sin2 j sin2 q ) r2 sin jdrdjdq
q =0 j =0 r =0
ˆ q =2p ˆ j=tan 1 R ˆ r= h sec q
h
= dr4 sin3 j drdjdq
q =0 j =0 r =0

Page 7
MATH 2023 Triple Integrals Problem Set #6

(b) Rank the ease of computations of the above coordinate systems for evaluating the
integral Iz , then compute Iz using the easiest coordinate system. Express your final
answer in terms of the mass m, not the density d.

Solution: From the easiest to the hardest: cylindrical, spherical, rectangular. Us-
ing rectangular coordinates would involve some difficult trig substitution. Using
spherical coordinates will amount to integrating sec5 q
ˆ q =2p ˆ r=R ˆ z=h
Iz = dr3 dzdrdq
q =0 r =0 z= Rh r

dpR4 h
=
10
pR4 h m
= 1
10 2
3 pR h
3
= mR2
10

9. (FF) Given a solid T with mass m and uniform density d, the center of mass ( x̄, ȳ, z̄) is
defined to be:
x ddV y ddV z ddV
˝ ˝ ˝
T T
x̄ = ˝ , ȳ = ˝ , z̄ = ˝T
T ddV T ddV T ddV

The moment of inertia of T about the z-axis is defined as:


˚
Iz := Dz ( x, y, z)2 ddV
T

where Dz ( x, y, z) is the perpendicular distance between the point ( x, y, z) and the z-axis.
Now consider the axis L passing through the center of mass ( x̄, ȳ, z̄) and parallel to the
z-axis. The moment of inertia of the solid about the axis L is defined as:
˚
Icm := DL ( x, y, z)2 ddV
T

where DL ( x, y, z) is the perpendicular distance between the point ( x, y, z) and the axis L.
Prove the following result (which is called the Parallel Axis Theorem):

Iz = Icm + md2

where d is the distance between the z-axis and the axis L.

Solution: As in Problem 2, Dz ( x, y, z)2 = x2 + y2 . Therefore,


˚
Iz = d( x2 + y2 ) dV
T

DL ( x, y, z) is the distance from ( x, y, z) to the axis L. Since L is a vertical line pass-


ing through ( x̄, ȳ, z̄), the x- and y-coordinates of every point on L must be x̄ and ȳ.
The distance DL ( x, y, z) is measured between the points ( x, y, z) and ( x̄, ȳ, z), i.e. the
perpendicular distance. Therefore, DL ( x, y, z)2 = ( x x̄ )2 + (y ȳ)2 .

Page 8
MATH 2023 Triple Integrals Problem Set #6

The distance d between the two vertical axes (z-axis and L) is the distance between
any two points at the same altitude. In other words, d2 = x̄2 + ȳ2 .
Consider Icm + md2 :
˚
2
Icm + md = d ( x x̄ )2 + (y ȳ)2 dV + m( x̄2 + ȳ2 )
˚T
= d x2 2x̄x + x̄2 + y2 2ȳy + ȳ2 dV + m( x̄2 + ȳ2 )
˚T ˚ ˚
= d( x2 + y2 ) dV 2 d ( x̄x + ȳy) dV + d x̄2 + ȳ2 dV + m( x̄2 + ȳ2 )
T T T

Note that x̄ and ȳ are constants, we get:


˚ ˚ ˚
Icm + md2 = Iz 2x̄ dx dV 2ȳ dy dV + x̄2 + ȳ2 ddV + m( x̄2 + ȳ2 )
T T T
= Iz 2x̄ · m x̄ 2ȳ · mȳ + m( x̄2 + ȳ2 ) + m( x̄2 + ȳ2 )
2m( x̄2 + ȳ2 ) + m( x̄2 + ȳ2 ) + m( x̄2 + ȳ2 ) = Iz .
= Iz

Here we have used the fact that m = T d dV and the definition of x̄ and ȳ.
˝

10. (F) The change-of-variable formula for the volume element dV is given by:
∂( x, y, z)
dxdydz = det dudvdw. (*)
∂(u, v, w)
(a) Using (*), verify that:
dxdydz = r2 sin f drdfdq.

Solution: It suffices to show:


∂( x, y, z)
det = r2 sin f.
∂(r, f, q )
Using the conversion rules x = r sin f cos q, y = r sin f sin q and z = r cos f (here
we used MATH convention), we get:
2 ∂x ∂x ∂x 3
∂r ∂f ∂q
∂( x, y, z) 6 ∂y 7
= 4 ∂y
∂r
∂y
∂f ∂q 5
∂(r, f, q ) ∂z ∂z ∂z
∂r ∂f ∂q
2 3
sin f cos q r cos f cos q r sin f sin q
4
= sin f sin q r cos f sin q r sin f cos q 5
cos f r sin f 0
Then by direct computations, we get:
∂( x, y, z)
det = r2 cos2 f sin f cos2 q + r2 sin3 f sin2 q
∂(r, f, q )
( r2 cos2 f sin f sin2 q r2 sin3 f cos2 q )
= r2 cos2 f sin f + r2 sin3 f = r2 sin f(cos2 f + sin2 f) = r2 sin f.

Page 9
MATH 2023 Triple Integrals Problem Set #6

(b) Let u = 2x, v = 3y and w = 5z. Using (*), express dxdydz in terms of dudvdw.

Solution: By rearrangement, we get x = u2 , y = v3 and z = w


5
2 3
∂x ∂x ∂x
∂( x, y, z) 6 ∂u
∂y
∂v
∂y
∂w
∂y 7
det = det 4 ∂u ∂w 5
∂(u, v, w) ∂z
∂v
∂z ∂z
∂u ∂v ∂w
21 3
0 0 2 1 1
= det 4 0 13 0 5 = = .
2⇥3⇥5 30
0 0 15

Therefore,
∂( x, y, z) 1
dxdydz = det dudvdw = dudvdw.
∂(u, v, w) 30

11. (FFF) Consider a solid sphere with radius R centered at the origin in R3 which carries
a uniform distribution of charges with density d. Each volume element dV in the sphere
can be regarded as a particle with charge d dV.
Fix a particle with charge q at (0, 0, z0 ) where z0 > R, i.e. outside the sphere, and call it
the q-particle. As in the previous Problem Set, the electric force exerted on the q-particle
by a charged element d dV at ( x, y, z) in the solid sphere is given by the Coulomb’s Law
(in vector form):
q d dV (0 x ) i + (0 y ) j + ( z0 z ) k
dF =
4p# 0 ((0 x )2 + (0 y)2 + (z0 z)2 )3/2

Similar to the previous Problem Set, the Principle of Superposition asserts that the resul-
tant force exerted on the q-particle by the whole sphere is given by “summing-up”, i.e.
integrating, each the force element dF over the sphere:
˚
Fresultant = dF.
sphere

(a) Show that:


!
2p R
p
qd r2 sin j · (z0 r cos j)
ˆ ˆ ˆ
Fresultant = 3/2
drdjdq k
0 0 0 4p# 0 r2 2rz0 cos j + z20

Solution: Use spherical coordinates:

Fresultant
ˆ 2p ˆ p ˆ R
= dF
0 0 0
ˆ 2p ˆ p ˆ R
q d dV xi yj (z z0 )k
= 3/2
0 0 0 4p# 0 ( x + y2 + ( z
2 z0 )2 )
ˆ 2p ˆ p ˆ R
qd r sin f cos q i r sin f sin q j (r cos f z0 ) k
= · r2 sin f drdfdq
4p# 0 0 0 0 (r2 sin2 f + (r cos f z0 )2 )3/2

Page 10
MATH 2023 Triple Integrals Problem Set #6

Note that the i and j components are zero since:


ˆ 2p ˆ 2p
sin q dq = cos q dq = 0.
0 0

After simplification of the k-component, one can obtain the required result.

(b) Try to compute the above integral, either by software or by hand, and show that:

qdR3 qQ
Fresultant = 2
k= k
3# 0 z0 4p# 0 z20

where Q is the total amount of charges in the sphere.


[Remark 1: This result shows that the resultant force exerted on the q-particle by the
charged sphere will be the same if one replaces it by a particle at the origin with the
same amount of charges.]
[Remark 2: Using the Gauss’s Law for Electricity, the above result can be obtained
easily by considering the surface flux of Fresultant . We will discuss that later, and will
derive the Gauss’s Law using the Divergence Theorem (assuming Coulomb’s Law).]

Solution: Compute the integral in (a). Being a human being in the 21th Century,
you should do it using Mathematica or WolframAlpha. Don’t waste your time
doing it by hand (unless you are required to in later E&M course).

Page 11
0 … t … 2p
is the circle r1t2 = 8 1, 3 cos t, 3 sin t 9 ,
8 x, y, z 9
45. F = on the line segment from 11, 1, 12 to
1x + y 2 + z 223>2
2

110, 10, 102


segment from 10, 0, 02 to 11, 2, 32. 8 x, y, z 9
46. F = 2 2
on the line segment from 11, 1, 12 to 18, 4, 22
MATH x + z2
2023+ •y Spring 2015-16 • Multivariable Calculus
egment from 11, 4, 12 to 13, 6,Problem
32. Set #7 •Circulation
47–48. Line Integrals, Conservative
Consider the following vectorVector Fields,
fields F and closedCurl Operator
oriented curves C in the plane (see figures).
Doa.not useonthe
Based Green’s
the picture, Theorem
make inabout
a conjecture any whether
problem in this set.
the circula-
helix r1t2 = 8 3 cos t, 3 sin t, t 9 , for tion of F on C is positive, negative, or zero.
1. (F) Let F = (y b. xCompute
)i + xjthe R2 , andand
oncirculation C be the counter-clockwise
interpret the result. circular path with radius
2 centered at the
47.origin.
F = 8 ySee
- x,the
x 9 ; figure
C: r1t2 =below:
8 2 cos t, 2 sin t 9 , for 0 … t … 2p
8 t, 2t, - 4t 9, for 1 … t … 2.
y F ! !y " x, x"

e a scalar line integral to find the length 2


.

cos t>4, t>2 9 , for 0 … t … 2


'
t, 50 cos t 9 , for 0 … t … 2p F. r > o
2
d curves C, evaluate 1C F # T ds.
tor fields in the plane Given the follow- x

bola r1t2 = 8 4t, t 2 9 , for 0 … t … 1


micircle r1t2 = 8 4 cos t, 4 sin t 9 , for

t.ir co
' '
segment from 11, 12 to 15, 102
8 x, y 9
he line segment from 12, 22(a) On102
to 110, the above
48. figure,
F = 2 highlight ; Cthe
is theportion
boundaryof thesquare
of the pathwith
C at which F · r0 > 0.
vertices
1x + y 221>2
(b) On the above figure, highlight
1{2, {22, traversed (with another color) the portion of the path C at which
counterclockwise.
F · r0 < 0.
he curve r1t2 = 8 t 2, 3t 2 9 , for 1 … t … 2 ˛ !x, y"
(c) Calculate the line integral F · dr from F!
y the definition.
(x2 # y2)1/2 Is the result alone sufficient to
C
determine
e r1t2 = 8 t, 4t 9 , for 1 … t … 10 whether F is conservative or not?
2
Solution:
n the force field F, find the work required First parametrize the path:
en oriented curve.
r(t) = (2 cos t)i + (2 sinCt)j, 0  t  2p.
h consisting of the line segment from
d by the line segment from 10, 02 toThat we have x = 2 cos t"2and y = 2 sin t. 2
10, 42is, x

consisting of the line segment ˛ ˆ 2p ˆ 2p


0
ollowed by the line segment from F · dr = F · r (t) dt = ((y x )i + xj) · (( 2 sin t)i + (2 cos t)j) dt
C 0 0 "2
ˆ 2p
bola y = 2x 2 from 10, 02 to 12, 82 = 2(y x ) sin t + 2x cos t dt
0
e y = 10 - 2x from 11, 82 to 13, 42 ˆ 2p
= 2(2 sin t 2 cos t) sin t + 2(2 cos t) · cos t dt
0
ˆ 2p
= 4 sin2 t + 4 sin t cos t + 4 cos2 t dt
0
ˆ 2p ˆ 2p
2
Recall from single-variable calculus that sin t dt = cos2 t dt, so we have:
0 0
˛ ˆ 2p ˆ 2p
F · dr = 4 sin t cos t dt = 2 sin 2t dt = [ cos 2t]2p
0 = 0.
C 0 0
˛
We cannot argue whether or not F is conservative by just showing F · dr = 0
˛ C

for ONE closed curve – we need F · dr = 0 for ALL closed curves.


C

Page 1
15.2 Line Integrals 1095
MATH 2023 Line Integrals, Conservative Vector Fields, Curl Operator Problem Set #7
3 3
in ! Convert the line integral to an ordi- 43–46. Work integrals in ! Given the force field F, find the work
(d)
the parameter and evaluate it. Calculate r ⇥ F, i.e.
required theancurl
to move ofonF.theIsgiven
object the oriented
result alone
curve. sufficient to determine whether
F is conservative
43. F =or8 x,not?
y, z 9 on the tilted ellipse r1t2 = 8 4 cos t, 4 sin t, 4 cos t 9 ,
is the circle r1t2 = 8 2 cos t, 0, 2 sin t 9 ,
for 0 … t … 2p
Solution:44. F = 8 - y, x, z 9 on the helix r1t2 = 8 2 cos t, 2 sin t, t>2p 9 , for
0 … t … 2p
is the circle r1t2 = 8 1, 3 cos t, 3 sin t 9 , i j k
8 x, y, z 9 ∂ ∂ ∂
45. F =r ⇥ F= ∂x on ∂y ∂z segment from 11, 1, 12 to
the line
2
1x + y + z 223>2
2
y x x 0
110, 10, 102
segment from 10, 0, 02 to 11, 2, 32. ∂
8 x, y, z 9 ∂y
∂ ∂ ∂ ∂ ∂
= i segment
∂z ∂x ∂z j + ∂x ∂y k
46. F = 2
x + y 2 + zx2
on the line
0 y xfrom0 11, 1, 12yto 18,x 4, 22
x
✓ ◆
segment from 11, 4, 12 to 13, 6, 32. ∂ vector
47–48. Circulation Consider the following ∂ fields F and closed
0i plane
=the
oriented curves C in 0j (see x
+ figures). (y x ) k
∂x ∂y
a. Based on the picture, make a conjecture about whether the circula-
helix r1t2 = 8 3 cos t, 3 sin t, t 9 , for = 0i 0j + 0k = 0.
tion of F on C is positive, negative, or zero.
b. Compute the circulation and interpret the result. 2
Since F is defined everywhere, the domain of F is R which is simply-connected.
47. F = 8 y - x, x 9 ; C: r1t2 = 8 2 cos t, 2 sin t 9 , for 0 … t … 2p
By Curl Test, we conclude that F is conservative.
8 t, 2t, - 4t 9, for 1 … t … 2.
y F ! !y " x, x"

(e) Find a potential function f such that F = r f , or show that such an f does not exist.
e a scalar line integral to find the length 2
Is the result alone sufficient to determine whether F is conservative or not?

cos t>4, t>2 9 , for 0 … t … 2 Solution: By inspection, it is not difficult to see that:
n t, 50 cos t 9 , for 0 … t … 2p ✓ ◆
x22
d curves C, evaluate 1C F # T ds.
ctor fields in the plane Given the follow- F = r xy . x
2
bola r1t2 = 8 4t, t 2 9 , for 0 … t … 1 x2
Therefore, f ( x, y) can be taken to be xy 2, and so F is conservative (by defini-
micircle r1t2 = 8 4 cos t, 4 sin t 9 , for
tion).

segment from 11, 12 to 15, 102 C: r(t) ! !2 cos t, 2 sin t"


x y
2. (F) Let F = p i +8p x, y 92 j, and C be the counter-clockwise square path with
2
x +F y= 2 +; yC2 is the boundary of the square with vertices
x2 1>2
he line segment from 12, 22 to 110, 102 48. 2
vertices (2, 2), (2, 2), (1x 2,+2y) 2and ( 2, 2). See the figure below:
1{2, {22, traversed counterclockwise.
he curve r1t2 = 8 t , 3t
2 29
, for 1 … t … 2 !x, y"
y F!
(x2 # y2)1/2

ne r1t2 = 8 t, 4t 9 , for 1 … t … 10
2
en the force field F, find the work required
'
en oriented curve. F. r > o
C
th consisting of the line segment from
"2 2 x
d by the line segment from 10, 02 to 10, 42
consisting of the line segment
followed by the line segment from
"2

bola y = 2x 2 from 10, 02 to 12, 82


FFEO
e y = 10 - 2x from 11, 82 to 13, 42

Do (a)-(e) of Problem #1 with this F and C instead.

Page 2
MATH 2023 Line Integrals, Conservative Vector Fields, Curl Operator Problem Set #7

˛
Solution: Part (c): To calculate F · dr from the definition, we split the path C into
C
four segments:
From (2, 2) to ( 2, 2): r1 (t) = h2, 2i + t(h 2, 2i h2, 2i) = h2 4t, 2i, 0  t  1.
ˆ 1 ˆ 1
(2 4t)i + 2j
F · r10 (t) dt = p · ( 4i + 0j) dt
0 0 (2 4t)2 + 22
ˆ 1
4(2 4t)
= p dt
0 (2 4t)2 + 22
ˆ 2
u
= p dt (Let u = 2 4t)
2 u2 + 4
=0 (Odd function!)

From ( 2, 2) to ( 2, 2): r2 (t) = h 2, 2 4ti, 0  t  1.


ˆ 1 ˆ 1
2i + (2 4t)j
F · r20 (t) dt = p · (0i 4j) dt
0 0 22 + (2 4t)2
ˆ 1
4(2 4t)
= p dt
0 2 + (2 4t)2
2

=0
Note the integral is the same as the previous one.
From ( 2, 2) to (2, 2): r3 (t) = h 2 + 4t, 2i, 0  t  1.
ˆ 1 ˆ 1
0 ( 2 + 4t)i 2j
F · r3 (t) dt = p · (4i + 0j) dt
0 0 ( 2 + 4t)2 + 22
ˆ 1
4( 2 + 4t)
= p dt
0 ( 2 + 4t)2 + 22
ˆ 1
4(2 4t)
= p dt
0 2 + (2 4t)2
2

=0

From (2, 2) to (2, 2): r4 (t) = h2, 2 + 4ti, 0  t  1.


ˆ 1 ˆ 1
2i + ( 2 + 4t)j
F · r40 (t) dt = p · (0i + 4j) dt
0 0 22 + ( 2 + 4t)2
ˆ 1
4( 2 + 4t)
= p dt
0 2 + ( 2 + 4t)2
2

=0

Finally, adding up the above line segments, we get:


˛
F · dr = 0 + 0 + 0 + 0 = 0.
C
Note that we
˛ can’t use this result alone to argue if F is conservative, as we have just
shown that F · dr for ONE closed curve C (but not for ALL closed curves C).
C

Page 3
MATH 2023 Line Integrals, Conservative Vector Fields, Curl Operator Problem Set #7

For Part (d):

i j k
∂ ∂ ∂
r⇥F = ∂x ∂y ∂z
y
p x2 2 p 0
x +y x 2 + y2
∂ ∂ ∂ ∂ ∂ ∂
∂y ∂z ∂x ∂z ∂x ∂y
= p 0 i 0 j+ k
y x y
p2 2 p x2 2 p
x 2 + y2 x +y x +y x 2 + y2
!
∂ y ∂ x
= 0i 0j + p p k
∂x x2 + y2 ∂y x2 + y2
n⇣ y ⌘ ⇣ x ⌘o
= ( x2 + y2 ) 3/2 · 2x ( x2 + y2 ) 3/2 · 2y k
2 2
= 0k = 0

Although r ⇥ F = 0, the domain of F is R2 \{(0, 0)} which is NOT simply-connected.


The Curl Test cannot be used here, and so this result alone cannot conclude on
whether F is conservative.
For Part (e): We can verify that:
✓q ◆
F( x, y) = r x 2 + y2 .

p
Therefore, one can take the potential function f ( x, y) = x2 + y2 , and so F is conser-
vative from definition.

3. (F) Let C be the curve of intersection of the cylinder x2 + y2 = 1 and the plane z = y.
(a) Sketch the cylinder, the plane and the curve C in the same diagram.

Solution:
>

^
.
( l , 0.01

C- I , 0,0) a

a
s

ˆ
(b) Let F = yi + zj xk. Calculate the line integral F · dr where G is a portion of C
G
from ( 1, 0, 0) to (1, 0, 0). There are two possible such G’s. Do both.
Is the result alone sufficient to determine whether F is conservative or not?

Page 4
MATH 2023 Line Integrals, Conservative Vector Fields, Curl Operator Problem Set #7

Solution: We need to first parametrize the path G. There are two such possi-
ble paths, namely the counter-clockwise path and clockwise path (when looking
from the top).
For the counter-clockwise path, the curve lies on the cylinder x2 + y2 = 1 and
therefore projects down to the unit circle centered at the origin on the xy-plane.
This unit circle is parametrized by x = cos t and y = sin t. Therefore, the red
path also has x and y coordinates given by x = cos t and y = sin t. Furthermore,
the red path lies on the plane z = y, and so we have z = sin t. To sum up, the
parametric equation for the red path is:

r1 (t) = (cos t)i + (sin t)j + (sin t)k, p  t  2p.

The bounds for t are chosen so that r1 (p ) = h 1, 0, 0i and r1 (2p ) = h1, 0, 0i,
which are the coordinates of the starting and ending points of the red path.

ˆ
F · dr
red path
ˆ 2p
= (yi + zj xk) · r10 (t) dt
p
ˆ 2p
= ((sin t)i + (sin t)j (cos t)k) · (( sin t)i + (cos t)j + (cos t)k) dt
p
ˆ 2p ˆ 2p
2 2
= ( sin t + sin t cos t cos t) dt = ( 1 + sin t cos t) dt
p p
 2p
1
= t cos2 t = p
2 p

For the clockwise path, we can parametrize it by replacing all t’s in r1 (t) by t,
i.e.:

r2 (t) = (cos( t))i + (sin( t))j + (sin( t))k


= (cos t)i (sin t)j (sin t)k

In order to give starting point ( 1, 0, 0) and ending point (1, 0, 0), we can set the
bounds for t to be p  2p, then r2 (p ) = h 1, 0, 0i and r2 (2p ) = h1, 0, 0i.

ˆ
F · dr
green path
ˆ 2p
= (( sin t)i + ( sin t)j (cos t)k) · (( sin t)i + ( cos t)j + ( cos t)k) dt
p
ˆ 2p  2p
1
= (1 + sin t cos t) dt = t cos2 t =p
p 2 p

Since we can find two different paths with the same starting and ending points
so that the line integral of F over them are not equal, we conclude that F is not
conservative.

Page 5
MATH 2023 Line Integrals, Conservative Vector Fields, Curl Operator Problem Set #7

(c) Find a potential function f such that F = r f , or show that such an f does not exist.
Is the result alone sufficient to determine whether F is conservative or not?

Solution: Set up:

∂f
=y
∂x
∂f
=z
∂y
∂f
= x
∂z
Integrating the first equation gives:

f ( x, y, z) = xy + g(y, z)

Then:
∂f ∂g
= x + (y, z)
∂y ∂y
Combining with the second equation, we get:

∂g ∂g
z = x+ (y, z) =) z (y, z) = x.
∂y ∂y

Now that the RHS is a function of x while the LHS is a function of y and z. It is
a contradiction. Therefore, such an f does not exist and so F is not conservative
by definition.

Alternatively, we can also check that:

i j k
∂ ∂ ∂
r⇥F = ∂x ∂y ∂z = i+j k 6= 0
y z x

Conservative vector field must have zero curl. Now that curl of F is non-zero, the
vector field F is not conservative.

4. (F) Determine whether or not each of the following vector fields is conservative or not.
If so, find its potential function f such that F = r f .
(a) F = (e y ze x )i + (e z xe y )j + (e x ye z )k

Solution: Set up:

∂f y x
=e ze
∂x
∂f z y
=e xe
∂y
∂f x z
=e ye
∂z

Page 6
MATH 2023 Line Integrals, Conservative Vector Fields, Curl Operator Problem Set #7

By integrating the first equation, we get:


y x
f ( x, y, z) = xe + ze + g(y, z) where g(y, z) is an arbitrary function
∂f y ∂g
Then, by differentiation we get ∂y = xe + ∂y , and combined with the second
equation, we must have:
∂g z
=e .
∂y
Integrating this, we get g(y, z) = ye z + h(z) for some arbitrary function h(z),
and so
f ( x, y, z) = xe y + ze x + ye z + h(z).
∂f
Again by differentiating, we get: ∂z = e x ye z + h0 (z). Combine with the third
equation, we get h0 (z) = 0 and so h is a constant.
It can be easily verified that r ( xe y + ze x + ye z + C ) = F, so F is conservative
with potential funciton f ( x, y, z) = xe y + ze x + ye z + C.

(b) F = ( x2 xy)i + (y2 xy)j

Solution: Set up:

∂f
= x2 xy
∂x
∂f
= y2 xy
∂y

Integrating the first equation we get:

x3 x2 y
f ( x, y) = + g(y)
3 2
∂f x2
By differentiation, we get ∂y = 2 + g0 (y). Combining with the second equa-
tion, we must have
x2
y2 xy = + g 0 ( y ).
2
However, that would imply

x2
= g0 (y) y2 + xy.
2
LHS is a function of x only, while RHS depends on both x and y. Therefore, such
a function f cannot exist and therefore F is not conservative.

Alternatively, one can show F is not conservative by showing:

r ⇥ F = (x y)k.

Therefore, r ⇥ F is non-zero, and so F is not conservative.

Page 7
MATH 2023 Line Integrals, Conservative Vector Fields, Curl Operator Problem Set #7

5. (F) Determine the values of A and B for which the vector field below is conservative:
✓ 2 ◆
x
F( x, y, z) = Ax ln z i + By2 z j + + y3 k,
z
where the domain of F is the upper-half space {( x, y, z) : z > 0}.
For each such pair of A and B, find the potential function f for the vector field.

Solution: Note that the domain of F is the upper-half space, which is simply-connected!
Therefore, we have:
F is conservative () r ⇥ F = 0
By straight-forward computations (omitted here), we get:

(A 2) x
r ⇥ F = (3 B ) y2 i + j + 0k
z
Therefore, F is conservative if and only if A = 2 and B = 3.
For this pair of A and B, we solve the equation F = r f for f :

∂f
= 2x ln z
∂x
∂f
= 3y2 z
∂y
∂f x2
= + y3
∂z z
Integrating the first equation, we get:

f ( x, y, z) = x2 ln z + g(y, z).

∂f x2 ∂g
Then, we have = + , and by comparison with the third equation, we get
∂z z ∂z
∂g
= y3 , and so g(y, z) = y3 z + h(y). Substitute back into f , it comes down to solving
∂z
h:
f ( x, y, z) = x2 ln z + y3 z + h(y)
∂f
By considering = 3y2 z + h0 (y) and the second equation, we conclude h0 (y) = 0
∂y
and so h(y) = C. The potential function for the vector field is: f ( x, y, z) = x2 ln z +
y3 z + C where C is any real constant.

6. (FF) Consider the path C:


⇣ ⌘ ⇣ ⌘
r(t) = cos2M t i + sin N t j + tk, 0  t  p.

Here M is the age of the Earth, and N is the age of the Universe. Assume both M and N
are positive finite integers.
Evaluate the line integral:
ˆ
e y ze x
dx + e z
xe y
dy + e x
ye z
dz
C

Page 8
MATH 2023 Line Integrals, Conservative Vector Fields, Curl Operator Problem Set #7

Solution: The vector field represented by this line integral is:


y x z y x z
F = (e ze )i + (e xe )j + (e ye )k

It appeared in Problem #4(a) in which we showed it is conservative with a poten-


tial y x z
ˆ function f ( x, y, z) = xe + ze + ye . Therefore, to compute the line integral
F · dr, we simply need to find the starting and ending points of the path C:
C

r(0) = i + 0j + 0k = h1, 0, 0i
r(p ) = i + 0j + pk = h1, 0, p i

By Fundamental Theorem of Line Integrals, we get:


ˆ
F · dr = f (1, 0, p ) f (1, 0, 0) = (e0 + pe 1 + 0) (e0 + 0 + 0) = pe 1
C

Alternatively,
ˆ given that F is a conservative vector field, one can calculate the line
integral F · dr by choosing an easier path joining the same endpoints as C. Clearly,
C
a straight-line path L is an easier path.
From above, the starting and ending points of C are (1, 0, 0) and (1, 0, p ) respectively.
The straight-line L can be parametrized by:

r L (t) = h1, 0, 0i + t(h1, 0, p i h1, 0, 0i) = h1, 0, tp i, 0  t  1.

ˆ ˆ
F · dr = F · r0L (t) dt
L L
ˆ 1
= (e y ze x )i + (e z
xe y
)j + (e x
ye z
)k · r0L (t) dt
0
ˆ 1
y x z y x z
= (e ze )i + (e xe )j + (e ye )k · (1i + 0j + pk) dt
0
ˆ 1 ˆ 1
x z 1 1
= p (e ye ) dt = p (e 0) dt = pe
0 0

Note that along the straight-line L, we have x = 1, y = 0 and z = tp.


ˆ ˆ
1
Finally, since F is conservative by Problem #4(a), we have F · dr = F · dr = pe .
C L

7. (FF) Given a conservative vector field F in R3 , the potential energy of F is a scalar-valued


function V ( x, y, z) such that F = rV. Suppose r(t) is the path of a particle with mass
m traveling in accordance to the Newton’s Second Law F(r(t)) = mr00 (t). Then its kinetic
energy is defined to be:
1 2
KE = m r0 (t) .
2
The total (kinetic + potential) energy of the particle at time t is therefore given by:
1 2
E(t) := m r0 (t) + V (r(t)).
2

Page 9
MATH 2023 Line Integrals, Conservative Vector Fields, Curl Operator Problem Set #7

Show that the total energy is conserved, i.e. E0 (t) = 0 for all time t.
[Hint: the only fact you need to know about Physics is the Newton’s Second Law given
above. It is purely a math problem.]

2
Solution: The key idea is to write |r0 (t)| as r0 (t) · r0 (t). Also, it’s essential to observe
that along the path, the potential energy V is first of all a function of x, y and z, and
( x, y, z) are functions of t. Therefore, one can use the chain rule to find dV dt .

✓ ◆
dE d 1 0 0 dV
= mr (t) · r (t) +
dt dt 2 dt
✓ ◆
1 00 0 0 00 ∂V dx ∂V dy ∂V dz
= m r (t) · r (t) + r (t) · r (t) + + +
2 | {z } ∂x dt ∂y dt ∂z dt
product rule | {z }
chain rule
✓ ◆ ✓ ◆
00 0 ∂V ∂V ∂V dx dy dz
= mr (t) · r (t) + i+ j+ k · i+ j+ k
∂x ∂y ∂z dt dt dt
| {z }
a good trick to learn
0 0
= F · r ( t ) + rV · r ( t )
= rV · r 0 ( t ) + rV · r 0 ( t )
= 0.

xi + yj + zk xi + yj
8. (FF) Denote er = p and er = p , which are the unit radial vector
x 2 + y2 + z2 x 2 + y2
fields in R3 and R2 respectively.
(a) Show that if F( x, y, z) = f (r)er where f is a function depending only on r =
p
x2 + y2 + z2 , then r ⇥ F = 0 on the domain of F. Is this result alone sufficient
to claim that F is conservative?

Solution: Using r2 = x2 + y2 + z2 , one can differentiate both sides by x, y and z


individually and show:

∂r x ∂r y ∂r z
= , = , = .
∂x r ∂y r ∂z r

Now consider the vector field F = f (r)er whose components are given by:

xi + yj + zk f (r) f (r) f (r)


F = f (r) = xi + yj + zk
r r r r

Then,
i j k
∂ ∂ ∂
r⇥F = ∂x ∂y ∂z
f (r) f (r) f (r)
r x r y r z

We are going to compute the i-component as an example (the j- and k-components

Page 10
MATH 2023 Line Integrals, Conservative Vector Fields, Curl Operator Problem Set #7

are similar). The i-component is given by:


∂ ∂ ⇢ ✓ ◆ ✓ ◆
∂y ∂z f (r)
∂ ∂ f (r)
f (r) i=
f (r) z y i
r y r z
∂y
r ∂z r
⇢ ✓ ◆ ✓ ◆
d f (r) ∂r d f (r) ∂r
= ·z · y i.
dr r ∂y dr r ∂z
⇣ ⌘ ⇣ ⌘
f (r) f (r) f (r)
Here we have used the chain rule on ∂y∂
r and ∂
∂z r , as r is a function
of r, and r is a function of ( x, y, z). Note that:
✓ ◆ ✓ ◆
d f (r) ∂r d f (r) ∂r
·z ·y
dr r ∂y dr r ∂z
✓ ◆ ✓ ◆
d f (r) y d f (r) z
= · ·z · ·y
dr r r dr r r
= 0.

Therefore, the i-component is zero. Similarly one can also show that the j- and
k-components are zero.
Assuming f is C1 – it should have been stated in the problem, the domain of F
is R3 \{(0, 0, 0)} since er is undefined only at the origin. Therefore, the domain
of F is simply-connected, and now that we showed r ⇥ F = 0. By curl test, we
conclude that F is conservative.
p
(b) Show that if G( x, y) = g(r )er where g is a function depending only on r = x2 + y2 ,
then r ⇥ G = 0 on the domain of G. Is this result alone sufficient to claim that G is
conservative?

Solution: The way to show r ⇥ G = 0 is very similar to part (a). Here we need
to know:
∂r x ∂r y g (r )
= , = , and G = ( xi + yj)
∂x r ∂y r r
One can then calculate the curl of G using the chain rule.
However, this result alone cannot conclude whether G is conservative. Since er is
undefined at (0, 0) but R2 \{(0, 0)} is NOT simply-connected. The curl test does
not apply here.

Page 11
MATH 2023 Line Integrals, Conservative Vector Fields, Curl Operator Problem Set #7

9. (F) Regard each English letter as a solid region in R2 . Which capital letters are simply-
connected? Which small letters are simply-connected?

Solution:
Simply-connected capital letters: CEFGHIJKLMNSTUVWXYZ
Simply-connected small letters: cfhklmnrstuvwxyz
Note that the small i and j are not connected, and hence not simply-connected.

10. (FF) The notation R3 \ X means the xyz-space R3 with the set X removed. Determine
whether R3 \ X is simply-connected when X is each of the following:
(a) X is the origin
(b) X is the entire y-axis
(c) X is the positive y-axis
(d) X is the solid sphere x2 + y2 + z2  1
(e) X is the surface sphere x2 + y2 + z2 = 1
(f) X is the solid cylinder x2 + y2  1
(g) X is the solid half-cylinder x2 + y2  1 and z 0.
(h) X is the surface cylinder x2 + y2 =1
(i) X is the surface half-cylinder x2 + y2 = 1 and z 0
(j) X is a solid torus
(k) X is a surface torus
(l) X is a simple closed curve
Give an example of a proper subset X of R3 such that both X and R3 \ X are simply-
connected. [Note: “proper” means X cannot be empty, and cannot be the whole R3 .]

Solution: R3 \ X is simply-connected for those X’s in: (a)(c)(d)(g)(i), whereas R3 \ X is


not simply-connected for those X’s in: (b)(e)(f)(h)(j)(k)(l).
Here is one of many examples of X so that both X and R3 \ X are both simply-
connected: When X is the upper-half space {( x, y, z) : z > 0}. Then R3 \ X is the
lower-half space.

Page 12
MATH 2023 • Spring 2015-16 • Multivariable Calculus
Problem Set #8 • Green’s Theorem

1. (F) Use the Green’s Theorem to evaluate


˛
2 2
(4y2 + e x ) dx (2x + ey ) dy
C

where C is each of the following (assume C is counter-clockwise oriented):


(a) the square with vertices (0, 0), (1, 0), (1, 1) and (0, 1)

2
Solution: The line integral is associated with the vector field F = (4y2 + e x )i
2
(2x + ey )j. By direct computation, we get:

r⇥F = 2(1 + 4y)k =) (r ⇥ F) · k = 2(1 + 4y).

By Green’s Theorem:
˛ ˆ 1ˆ 1
y
F · dr = (r ⇥ F) · k dxdy

=:
C 0 0
1ˆ 1

:
ˆ
X
= 2(1 + 4y) dxdy
0 0
= 6

(b) the square with vertices (1, 0), (0, 1), ( 1, 0) and (0, 1)

Solution: Denote the solid square by R, then by Green’s Theorem:


˛ ¨
F · dr = (r ⇥ F) · k dA
C R
¨ ¨ ¨
= 2(1 + 4y) dA = 2 dA 8y dA
R R R

÷:
¨
= 2 Area(R) 8y dA y
R
¨
= 4 8y dA
R
yao
Since¨
y is an odd function ˛and the region R is symmetric about the x-axis, we
have 8y dA = 0. Hence F · dr = 4.
R C

(c) the triangle with vertices (0, 0), (1, 0) and (0, 1)

Solution: The hypotenuse of the triangle is given by equation y = 1 x. Using


the Green’s Theorem:

0
R
>
:
nfxtfsl
I
x
˛

C
F · dr =
ˆ x =1 ˆ y =1 x

x =0 y =0
(r ⇥ F) · k dydx =
ˆ x =1 ˆ y =1 x

x =0 y =0
2(1 + 4y) dydx =
7
3
.

Page 1
MATH 2023 Green’s Theorem Problem Set #8

(d) the unit circle x2 + y2 = 1

Solution: The enclosed region is a unit circle. It is best to use polar coordinates.
˛ ˆ 2p ˆ 1
F · dr = (r ⇥ F) · k rdrdq
C 0 0
ˆ 2p ˆ 1
= 2(1 + 4y) rdrdq
0 0
ˆ 2p ˆ 1
= 2(1 + 4r sin q )r drdq
0 0
= 2p.

2. (FF) The purpose of this problem is to explore a line integral for computing areas.
(a) Let C be a simple closed curve in R2 and the area enclosed by C is denoted by A.
Show that:
1
˛
A= y dx + x dy
2 C

Solution:
1 1 1
˛ ˛ ¨
y dx + x dy = h y, x, 0i · dr = (r ⇥ h y, x, 0i) · k dA.
2 C 2 C 2 R

Here R is the region enclosed by C. By direct computations, we get:

r ⇥ h y, x, 0i = 2k.

Hence
1 1
˛ ¨
y dx + x dy = 2 dA = Area of R.
2 C 2 R

x2 y2
(b) Let E be the ellipse + = 1 where a, b > 0. Find the area bounded by E using
a2 b2
the result of (a).

Solution: The ellipse can be parametrized by:


r(t) = ( a cos t)i + (b sin t)j, 0  t  2p.
In other words, x = a cos t and y = b sin t.
1
˛
A= y dx + x dy
2 C
1 t=2p
ˆ
= {z }t) + |( a cos
(b sin t) d(|a cos t) d(b sin t)
2 t =0 | {z } {z } | {z }
y x x y

1 t=2p
ˆ
= ab sin2 t dt + ab cos2 t dt
2 t =0
1 t=2p
ˆ
= ab dt = abp.
2 t =0

Page 2
MATH 2023 Green’s Theorem Problem Set #8

(c) Let P be a n-sided polygon with vertices ( x0 , y0 ), ( x1 , y1 ), . . . , ( xn 1 , yn 1 ). See the


figure below for an example when n = 6. For convenience, we denote ( xn , yn ) =
( x0 , y0 ). Using (a), show that the area A( P) bounded by the polygon P is given by:
1 n
2 iÂ
A( P) = ( xi 1 yi xi yi 1) .
=1

( x4 , y4 )

↳ '
( x2 , y2 )

in
L
L
: ↳
( x5 , y5 )

[
( x3 , y3 ) 2

( x0 , y0 ) >

Li
( x1 , y1 )

Solution: Denote Li to be the straight-line from ( xi 1 , yi 1 ) to ( xi , yi ), which is


parametrized by:

ri ( t ) = h xi 1 , yi 1 i + t h xi xi 1 , yi yi 1 i, 0  t  1.
| {z } | {z }
starting point direction

Then on Li , we have x = xi 1 + t ( xi xi 1) and y = yi 1 + t ( yi yi 1 ), and so:

dx = ( xi xi 1 ) dt and dy = (yi yi 1 ) dt.

The polygon can then be represented as the directed path L1 + L2 + . . . + Ln , or


n
simply  Li . By (a), we have:
i =1

1 1 n
˛ ˆ
A( P) = y dx + x dy = Â y dx + x dy
2 Âin=1 Li 2 i =1 Li
n ˆ t =1
1
2 iÂ
= ( yi 1 + t ( yi yi 1 )) ( xi xi 1 ) dt
=1 t =0 | {z }
y dx

1 n t =1
ˆ
+ Â (x 1 + t ( xi xi 1 )) (yi yi 1 ) dt
2 i =1 t =0 | i {z }
x dy
 ✓ ◆ t =1
1 n ( yi yi 1 )t
2
= Â ( xi xi 1 ) yi 1t +
2 i =1 2 t =0
n  ✓ ◆ t =1
1 ( xi xi 1 ) t2
2 iÂ
+ ( yi yi 1) xi 1t +
=1
2 t =0
n n
1 yi + yi 1 1 xi + xi 1
2 i 2 iÂ
= ( xi xi 1) · + ( yi yi 1) ·
=1
2 =1
2

which yields the desired result after simplifications.

Page 3
MATH 2023 Green’s Theorem Problem Set #8

3. (FF) Consider the following system of differential equations:

dx dy
= f ( x, y) = g( x, y)
dt dt
∂f ∂g
where f and g are C1 on R2 . Given that + > 0 on R2 , show that the system cannot
∂x ∂y
have a non-constant periodic solution. We say a solution ( x (t), y(t)) is periodic if there
exists T > 0 such that ( x (0), y(0)) = ( x ( T ), y( T )).
Hint: Proof by contradiction. Apply Green’s Theorem on F = g( x, y)i + f ( x, y)j.

Solution: Suppose the system has a non-constant periodic solution ( x (t), y(t)). Let
T > 0 be the first time such that ( x ( T ), y( T )) = ( x (0), y(0)), then the curve:

r(t) = h x (t), y(t)i, 0tT

is a simple closed curve in R2 . Denote this simple closed curve by C and let R be the
region enclosed by C. Apply Green’s Theorem on the vector field F = g( x, y)i +
f ( x, y)j over C:
˛ ¨ :
to
F · dr = (r ⇥ F) · k dA *( →
o
C R
¨ ✓ ✓ ◆◆
∂f ∂g
= k · k dA
R ∂x ∂y
¨ ✓ ◆
∂f ∂g
= + dA > 0
R ∂x ∂y
| {z }
given>0

On the other hand, the line integral can be shown to be zero:


˛ ˆ t= T
F · dr = h g( x, y), f ( x, y)i · h x 0 (t), y0 (t)i dt
C t =0 | {z } | {z }
F r0 (t)
ˆ t= T
= g( x, y) x 0 (t) + f ( x, y) y0 (t) dt.
t =0

From the given differential equations, we have

g( x, y) x 0 (t) + f ( x, y) y0 (t) = g( x, y) f ( x, y) + f ( x, y) g( x, y) = 0.

Therefore, we have: ˛
F · dr = 0
C
which contradicts to the previous result, so the system cannot have non-constant
periodic solution.
[FYI: This result is called the Bendixson-Dulac’s Theorem. First established in 1901 by
Ivar Bendixson. This short proof using Green’s Theorem is later discovered by Henri
Dulac in 1933.]

Page 4
MATH 2023 Green’s Theorem Problem Set #8

y x
4. (FF) Consider the vector field F = i+ 2 j which is defined at every point
x2 +y 2 x + y2
on R2 except the origin.
(a) Verify that r ⇥ F = 0 at every point in R2 except the origin.

Solution: Straight-forward.
˛
(b) Show, by direct computation, that F · dr is non-zero where C is the unit circle,
C
counter-clockwise oriented, with centered at the origin.

Solution: The unit circle is parametrized by r(t) = (cos t)i + (sin t)j, 0  t 
2p. Hence,
ˆ t=2p ✓ ◆
sin t cos t
˛
F · dr = i+ j · ( (sin t)i + (cos t)j) dt
C t =0 cos2 t + sin2 t cos2 t + sin2 t | {z }
| {z } r0 (t)
F
2p 2 2p
sin t + cos2t
ˆ ˆ
= 2
dt = 1 dt = 2p.
2
cos t + sin t
0 0

(c) The following students are confused about the above vector field F in relation to
some facts and theorems stated in class. Pretend that you are a teaching assistant of
this course, point out their misconceptions.
i. Student A said, “Given that r ⇥ F = 0, the Curl Test asserts that F is conservative
and so the closed-path line integral in (b) should be zero. How come the answer
for (b) is non-zero???!!!”

Solution: The domain of F is R2 \{(0, 0)} which is NOT simply-connected.


The curl test cannot be used here.

ii. Student B said, “Given that r ⇥ F = 0, the Green’s Theorem asserts that
˛ ¨ ¨
F · dr = (r ⇥ F) · k dA = 0 · k dA = 0
C R R

for any closed-path C. Why can the answer in (b) be non-zero???!!!” ”

Solution: The unit circle C encloses the origin at which F is not defined.
Green’s Theorem cannot be used for this curve C.
⇣ y⌘
iii. Student C said, “It can be verified that F = r tan 1 and so F is conservative
x
y
with potential function f ( x, y) = tan 1 . Any line integral of a conservative
x
vector field over a closed curve must be zero. How come can the closed-path
integral in (b) be non-zero???!!!”

Solution: The domain of the potential function f needs to be the same as


that of a vector field F. In our case, the domain of F is R2 \{(0, 0)} whereas
y y
the domain of tan 1 x is R2 \{y-axis}. Hence, tan 1 x cannot be regarded as
the (global) potential function of F. We cannot show F is conservative in this
way.

Page 5
MATH 2023 Green’s Theorem Problem Set #8

5. (FF) In the figure shown below, G1 and G2 are circular arcs centered at the origin. L1
and L2 are straight-lines. Consider the closed path C = L1 + G1 + L2 + G2 .

G2

G1

x
3 L1 1 1 L2 3

˛
Compute the line integral F · dr of each vector field below using the Green’s Theorem
C
in an appropriate way:
(a) F = y3 i x3 j

Solution: The vector field is C1 everywhere in R2 . No problem to apply Green’s


Theorem. Direct computations show:

r⇥F = 3( x2 + y2 )k =) (r ⇥ F) · k = 3( x 2 + y2 ).

By Green’s Theorem:
˛ ¨
F · dr = (r ⇥ F) · k dA
C R
ˆ ˆ 3 p
= 3( x2 + y2 ) r drdq
0 1
ˆ pˆ 3
= 3 r3 drq = 60p.
0 1

y 3 x 3
(b) F = i+ j
(x 3)2 + ( y 3)2 (x 3)2 + ( y 3)2

Solution: By somewhat lengthy computations, one can verify that r ⇥ F = 0.


The domain of F is R2 \{(3, 3)}. Fortunately, the closed path above does not
enclose (3, 3) – no problem to use Green’s Theorem:
˛ ¨
F · dr = (r ⇥ F) ·k dA = 0
C R | {z }
=0

y 2 x
(c) F = i+ 2 j
x2 + ( y 2) 2 x + (y 2)2

Solution: By another somewhat lengthy computations, we get r ⇥ F = 0. The


domain of F is R2 \{(0, 2)}. However, the closed path C encloses this bad point
(0, 2) – we can’t use Green’s Theorem directly.

Page 6
MATH 2023 Green’s Theorem Problem Set #8

To handle this path, we construct a “hole” with radius 1 centered at (0, 2). Denote
the boundary of the hole by G3 = G3+ + G3 as shown in the figure below. Note
that G3 is a clockwise circle.
y

G2 G2+
G3 r -
G3+

G1 1 G1+
x
3 L1 1 L2 3

Consider the red and blue paths individually. Each of the red and blue path does
not enclose the bad point (0, 2), we can apply Green’s Theorem without problem:
ˆ ˆ ˆ ˆ ¨
F · dr + F · dr + F · dr + F · dr = (r ⇥ F) ·k dA = 0
G2+ G3+ G1+ L2 R+ | {z }
=0
ˆ ˆ ˆ ˆ ¨
F · dr + F · dr + F · dr + F · dr = (r ⇥ F) ·k dA = 0
G2 L1 G1 G3 R | {z }
=0

Summing up and use the fact that Gi = Gi + Gi+ (for i = 1, 2, 3), we get:
ˆ ˆ ˆ ˆ ˛
F · dr + F · dr + F · dr + F · dr + F · dr = 0.
G2 L1 G1 L2 G3
| {z }
C = G2 + L1 + G1 + L2

Hence, ˛ ˛
F · dr + F · dr = 0
C G3
˛ ˛
To find F · dr, it suffices to find F · dr. Note that G3 is clockwise, it is
C G3
parametrized by:

r(t) = (0 + cos( t))i + (2 + sin( t))j, 0  t  2p.

Then, x = cos t, y = 2 sin t, and so x2 + (y 2)2 = 1.


2p ✓ ◆
(2 sin t) 2 cos t
˛ ˆ
F · dr = i+ j · (( sin t)i (cos t)j) dt
G3 0 1 1
ˆ 2p
= ( sin2 cos2 t) dt = 2p.
0

Therefore, ˛ ˆ
F · dr = F · dr = 2p.
C G3

Page 7
MATH 2023 Green’s Theorem Problem Set #8

6. (FF) Consider the flow of fluid (shown in blue in the figure below) which is represented
by the vector field:
✓ ◆ ✓ ◆
y 2y x+1 2( x 1)
F= + i+ j
( x + 1)2 + y2 ( x 1)2 + y2 ( x + 1)2 + y2 ( x 1)2 + y2
C is an arbitrary simple closed curve (red in the figure) which encloses all points at which
F is not defined.
2

:
1

g-

On
0

94.41 4 th
-1 >

-2

-2 -1 0 1 2

(a) At which point(s) the vector field F is/are not defined? Is the domain of F simply-
connected?

Solution: F is NOT defined at ( 1, 0) and (1, 0). The domain of F is

R2 \{( 1, 0), (1, 0)}

which is NOT simply-connected.

(b) Verify that r ⇥ F = 0 at every point in R2 where F is defined.

Solution: Straight-forward, but quite lengthy.

(c) Show ˛ that from the definition of line integrals:


i. F · dr = 2p for any counter-clockwise circle G centered at ( 1, 0) with radius
G
less than 2.

Solution: G is parametrized by:

r(t) = h 1 + # cos t, # sin ti, 0  t  2p.

On this path, the vector field is given by:



# sin t 2# sin t # cos t 2(# cos t 2)
F= + 2
, .
# 2 2 2
(# cos t 2) + # sin t #2 (# cos t 2)2 + #2 sin2 t

Page 8
MATH 2023 Green’s Theorem Problem Set #8

r0 (t) = h # sin t, # cos ti


2#2 sin2 t 2# cos t(# cos t 2)
F · r0 (t) = 1
(# cos t 2)2 + #2 sin2 t (# cos t 2)2 + #2 sin2 t
2#2 4# cos t
=1
(# cos t 2)2 + #2 sin2 t
2#2 4# cos t
=1
#2 4# cos t + 4
ˆ 2p
2#2 4# cos t
˛
F · dr = 1 dt
G 0 #2 4# cos t + 4
= 2p.

Mathematica was used to compute this difficult integral.


˛
ii. F · dr = 4p for any counter-clockwise circle g centered at (1, 0) with radius
g
less than 2.

Solution: Similar to (i). Parametrize the path by r(t) = h1 + # cos t, # sin ti.

(d) Using the above results, show that:


˛
F · dr = 2p
C

for any simple closed curve C in R2 that encloses all points at which F is not defined.

Solution: C encloses points at which F is not defined. We need to drill two


circular holes centered at ( 1, 0) and (1, 0). Then, apply Green’s Theorem on the
closed path C + L1 G L1 + L2 g L2 , which does not enclose ( 1, 0) and
(1, 0), we get:
˛
F · dr
C + L1 G L1 + L2 g L2
˛ ˆ ˛ ˆ ˆ ˛ ˆ
= F · dr + F · dr F · dr F · dr + F · dr F · dr F · dr
C L1 g L1 L2 g L2
¨
= (r ⇥ F) ·k dA = 0
R | {z }
=0

After cancellations, we get:


˛ ˛ ˛
F · dr F · dr F · dr = 0
C g G

From (c), we conclude that:


˛ ˛ ˛
F · dr = F · dr + F · dr = 4p + 2p = 2p.
C g G

Page 9
MATH 2023 • Spring 2015-16 • Multivariable Calculus
Problem Set #9 • Surface Integrals, Stokes’ Theorem

1. (F) Consider the right circular cone surface (just the shell, and the flat top is not included)
with base radius R and height h, and with z-axis as the central axis and the origin as the
vertex. See the figure below):

Suppose the cone has uniform surface density σ and its total mass is m.
(a) Write down a parametrization r(u, v) of the cone, and indicate the range of the pa-
rameters. It is OK to use other letters for the pair of parameters.
(b) Find the surface area of the cone.
¨
(c) Find the moment of inertia Iz := ( x2 + y2 )σ dS. about the z-axis. Express your
S
final answer in terms of the mass m.
(d) Compute the surface flux of the vector field F = i through the cone. Choose n̂ to be
the upward unit normal. Do not use Stokes’ Theorem in this problem.

Solution: There are at least two ways to parametrize the surface cone.
h
Using Cylindrical Coordinates: The cone surface is represented by z = Rr (use similar
triangles to figure out the ratio of sides)

h
r(r, θ ) = (r cos θ )i + (r sin θ )j + rk, 0 ≤ r ≤ R, 0 ≤ θ ≤ 2π.
R

By direct computations (omitted here), one can get:


   
∂r ∂r h h
× = − r cos θ i + − r sin θ j + rk
∂r ∂θ R R
r 2 2 r
2
× ∂r = h r + r2 = r 1 + h
∂r
∂r ∂θ R2 R2

Therefore,
¨ ˆ θ =2π ˆ r=R

∂r
× ∂r drdθ

surface area = dS = ∂r ∂θ
S θ =0 r =0
ˆ 2π ˆ R
r r
h2 h2
= r 1+ drdθ = πR2 1+
0 0 R2 R2

Page 1
MATH 2023 Surface Integrals, Stokes’ Theorem Problem Set #9

¨ ˆ 2π ˆ r=R
r
2 3 h2
Iz = δr dS = δr 1+ drdθ
S 0 r =0 R2
r
πR4 h2
= δ· 1+ 2
2 R
m πR4
= q ·
πR2 1 + h
2 2
R2
mR2
=
2
¨ ˆ 2π ˆ r=R  
∂r ∂r
i · n̂ dS = i· × drdθ
S 0 r =0 ∂r ∂θ
ˆ 2π ˆ R
h
= − r cos θ drdθ
0 0 R
ˆ 2π  ˆ R !
h
=− cos θ dθ r dr = 0
R 0 0

Using Spherical Coordinates: The surface cone is represented by the equation ϕ =


tan−1 Rh . For simplicity, denote α = tan−1 Rh , then the cone is represented by ϕ = α
and so:
p
r(ρ, θ ) = (ρ sin α cos θ )i + (ρ sin α sin θ )j + (ρ cos α)k, 0 ≤ ρ ≤ h2 + R2 , 0 ≤ θ ≤ 2π.

Direct computations give:

∂r ∂r
× = (−ρ sin α cos α cos θ )i + (−ρ sin α cos α sin θ )j + (ρ sin2 α)k
∂ρ ∂θ

∂r ∂r
× = ρ sin α
∂ρ ∂θ

ˆ θ =2π ˆ √
ρ = h2 + R2
surface area = ρ sin αdρdθ
θ =0 ρ =0
R
= π (h2 + R2 ) sin α = π (h2 + R2 ) sin(tan−1 )
h
R p
= π ( h2 + R2 ) · √ = πR h2 + R2
2
h +R 2
ˆ θ =2π ˆ ρ=√h2 + R2
Iz = δρ2 sin2 α · ρ sin α dρdθ
θ =0 ρ =0
( h2 + R2 )2
= δ· · 2π sin3 α
4
3
( h2 + R2 )2 mR2

m R
= √ · · 2π · √ =
πR h2 + R2 4 h2 + R2 2

Page 2
MATH 2023 Surface Integrals, Stokes’ Theorem Problem Set #9

¨ ˆ θ =2π ˆ √
ρ = h2 + R2  
∂r ∂r
i · n̂ dS = i· × dρdθ
S θ =0 ρ =0 ∂ρ ∂θ
ˆ θ =2π ˆ √
ρ = h2 + R2
= (−ρ sin α cos α cos θ ) dρdθ
θ =0 ρ =0
ˆ 2π  ˆ √2 2 h +R
!
=− cos θ dθ ρ dρ · sin α cos α = 0.
0 0

2. (F) Consider the parametrization of a torus (i.e. donut):

r(u, v) = (( R + a cos u) cos v) i + (( R + a cos u) sin v) j + ( a sin u) k

where 0 ≤ u ≤ 2π and 0 ≤ v ≤ 2π. Here R and a are constants such that R > a > 0.

Suppose the torus has uniform surface density σ and its total mass is m.
(a) Find the surface area of the torus.

(b) Find the moment of inertia Iz := ( x2 + y2 )σ dS about the z-axis. Express your final
S
answer in terms of m.
(c) Compute the surface flux of the vector field F = k through the torus. Choose n̂ to be
the outward unit normal. Do not use Stokes’ Theorem in this problem.

Solution:
∂r ∂r
×
∂u ∂v
= (− a( R + a cos u) cos u cos v)i + (− a( R + a cos u) cos u sin v)j
+ (− a( R + a cos u) sin u)k

∂r ∂r
∂u × ∂v = a( R + a cos u)

Page 3
MATH 2023 Surface Integrals, Stokes’ Theorem Problem Set #9

ˆ 2π ˆ 2π

∂r ∂r
surface area = ∂u × ∂v dudv

0 0
ˆ 2π ˆ 2π
= a( R + a cos u) dudv = 4π 2 aR
0 0
ˆ 2π ˆ 2π
Iz = δ( R + a cos u)2 · a( R + a cos u) dudv
0 0
= δ · 2π aR(3a2 + 2R2 )
2

m
= · 2π 2 aR(3a2 + 2R2 )
4π 2 aR
m
= (3a2 + 2R2 )
2
¨ ˆ 2π ˆ 2π  
∂r ∂r
k · n̂ dS = k· × dudv
T 0 0 ∂u ∂v
ˆ 2π ˆ 2π
= − a( R + a cos u) sin u dudv = 0.
0 0

3. (FF) In Chapter 2, we claimed without proof that ∇ f ( P) is perpendicular to the level


surface f = c at P (we proved the case of level curves only). In this problem, we are going
to complete the proof for level surfaces.
Let f ( x, y, z) be a C1 function, and S be the level surface f ( x, y, z) = c. Consider a
∂r ∂r
parametrization r(u, v) for S, then if one can show ∂u and ∂v are both perpendicular to
∇ f , then we are done because the normal vector ∂u × ∂v to the surface S will then be
∂r ∂r

parallel to ∇ f . By considering f (r(u, v)) = c, show that ∇ f · ∂u


∂r
= 0.
[The fact that ∇ f · ∂r
∂v = 0 can be shown in a similar way.]

Solution: Since r(u, v) = x (u, v)i + y(u, v)j + z(u, v)k represents a point on the level
surface f ( x, y, z) = c, we have
f ( x (u, v), y(u, v), z(u, v)) = c or in short f (r(u, v)) = c.
By chain rule,
f (r(u, v)) = c
∂ ∂c
=⇒ f (r(u, v)) =
∂u ∂u
∂ f ∂x ∂ f ∂y ∂ f ∂z
=⇒ + + =0
∂x ∂u ∂y ∂u ∂z ∂u
   
∂f ∂f ∂f ∂x ∂y ∂z
=⇒ i+ j+ k · i+ j+ k =0
∂x ∂y ∂z ∂u ∂u ∂u
∂r
=⇒ ∇ f · =0
∂u
Therefore, ∇ f is perpendicular to the tangent vector ∂u
∂r
. Similarly, one can show ∇ f
is perpendicular to the tangent vector ∂v . It concludes that ∇ f is a normal vector to
∂r

the level surface f = c.

Page 4
MATH 2023 Surface Integrals, Stokes’ Theorem Problem Set #9 SECT

EXAMPLE
4. (F) Suppose S is a level surface f ( x, y, z) = c of a C1 function f V. Show 1 Evaluate xC F ⴢ dr, where F共x, y, z
that:
curve of intersection of the plane y  z 苷 2 and
¨ ¨ be counterclockwise when viewed from above.)
∇ f · zn̂ dS = ± |∇ f | dS
S S SOLUTION The curve C (an ellipse) is shown in Figu
evaluated directly, it’s easier to use Stokes’ Theor
where ± depends on the choice of unit normal n̂.


S C
y+z=2 i j k
  
curl F 苷
Solution: Since S is a level surface f = c, its unit normal vector is given by: x y z
y 2 x z2
∇f
0n̂ = ± . Although there are many surfaces with boundary
D |∇ f |
y elliptical region S in the plane y  z 苷 2 that is b
Therefore, we have: x then C has the induced positive orientation. The p
disk x 2  y 2 1 and so using Equation 16.7.10
¨ ¨ ¨ ¨
FIGURE∇3f |∇ f |2
∇ f · n̂ dS = ± ∇f · dS = ± dS = ± |∇ f | dS.
S S |∇ f | S |∇ f | S y F ⴢ dr 苷 yy curl F ⴢ dS 苷 yy 共1
C
S D
2
Here we have used the fact that v · v = |v| . 2 1
苷y y 共1  2r sin  兲 r d
0 0

5. (F) Let S be the part of the sphere x2 + y2 + z2 = 4 that lies inside the cylinder x2 + 苷
and above the xy-plane. Denote C to be the boundary of S with orientation indicated0 in
y2 = 1  2 sin 
2 3 y
2
冋 r2 r3

the diagram below: 苷 12 共2兲  0 苷 

z V EXAMPLE 2 Use Stokes’ Theorem to compute


≈+¥+z@ = 4 F共x, y, z兲 苷 xz i  yz j  xy k and S is the part o
S
lies inside the cylinder x 2  y 2 苷 1 and above th
C SOLUTION To find the boundary curve C we solve th
x 2  y 2 苷 1. Subtracting, we get z 2 苷 3 and so z
0 circle given by the equations x 2  y 2 苷 1, z 苷 s
y
r共t兲 苷 cos t i  sin t j  s3
x ≈+¥=1

FIGURE 4
so r共t兲 苷 sin t i  cos t j
(a) Write down the parametrizations of both the surface S and the curve C. For the
surface S, choose a suitable coordinate system so that the Also, we have have constant
parameters
bounds. F共r共t兲兲 苷 s3 cos t i  s3 s
Therefore, by Stokes’ Theorem,
Solution:
2
π yy curl F ⴢ dS 苷 yC F ⴢ dr 苷 y0
r(φ, θ ) = (2 sin φ cos θ )i + (2 sin φ sin θ )j + (2 cos φ)k; 0 ≤ φ ≤ , 0 S≤ θ ≤ 2π
√ 6
2
r(t) = (cos t)i + (sin t)j + 3k; 0 ≤ t ≤ 2π 苷 y (s3 cos t
0

˛ 2
苷 y 0 dt 苷 0
(b) Consider the vector field F( x, y, z) = xzi + yzj + xyk. Compute both F · dr and s3 0
¨ C

(∇ × F) · n̂ dS directly. Verify that they are equal.


S

Page 5
MATH 2023 Surface Integrals, Stokes’ Theorem Problem Set #9


Solution: Along C, we have x = cos t, y = sin t and z = 3.
˛ ˆ 2π
F · dr = ( xzi + yzj + xyk) · r0 (t) dt
C 0
ˆ 2π √ √ 
= 3 cos t i + 3 sin t j + cos t sin t k · (− sin t i + cos t j) dt
0
ˆ 2π
= 0 dt = 0.
0
¨
To compute (∇ × F) · n̂ dS, we first compute:
S

∇ × F = ( x − y)i + ( x − y)j.

Using the parametrization r(φ, θ ), we get:

∂r ∂r
n̂ dS = × dφdθ
∂φ ∂θ
= 4 sin2 φ cos θ i + 4 sin2 φ sin θ j + (4 sin θ cos θ ) k dφdθ
  

Therefore,
¨ ˆ 2π ˆ π
(∇ × F) · n̂ dS = ( x − y)(4 sin2 φ)(cos θ + sin θ ) dφdθ
S 0 0
ˆ 2π ˆ π
= 2 sin φ(cos θ − sin θ ) 4 sin2 φ (cos θ + sin θ ) dφdθ
0 0 | {z }
x −y
ˆ 2π  ˆ π 
=8 (cos2 θ − sin2 θ ) dθ sin3 φ dφ
0 0
=0

6. (F) Let C be the simple closed curve given parametrized by:


r(t) = (cos t)i + (sin t)j + (sin 2t)k, 0 ≤ t ≤ 2π.
(a) Show that the curve lies on the surface z = 2xy.

Solution: Use the identity sin 2t = 2 sin t cos t.

(b) Use the Stokes’ Theorem to evaluate the line integral:


˛
2 x2
e x dx + yzdy + dz.
C 2
[Why is it difficult to compute this line integral directly?]
˛
Solution: The line integral can be written as F · dr where
C

2 x2
F = e x i + yzj + k
2

Page 6
MATH 2023 Surface Integrals, Stokes’ Theorem Problem Set #9

By straight-forward computations, we get:

∇ × F = −yi − xj + 0k.

Let Σ be the surface z = 2xy enclosed by C. It can be parametrized by cylindrical


coordinates:

r(r, θ ) = (r cos θ )i + (r sin θ )j + (2r2 sin θ cos θ ) k, 0 ≤ r ≤ 1, 0 ≤ θ ≤ 2π.


| {z }
z=2xy

Using Stokes’ Theorem, we have:


˛ ¨
F · dr = (∇ × F) · n̂ dS
C Σ
ˆ 2π ˆ 1

∇(z − 2xy) ∂r ∂r
= (−yi − xj) · × drdθ
0 0 | {z } |∇(z − 2xy)| ∂r ∂θ
∇×F | {z } | {z }
n̂ dS
ˆ 2π ˆ 1
−2yi − 2xj + k p
= (−yi − xj) · p r 1 + 4r2 drdθ
0 0 1 + 4x2 + 4y2
ˆ 2π ˆ 1
1 p
= 2( x 2 + y2 ) √ · r · 1 + 4r2 drdθ
0 0 1 + 4r2
ˆ 2π ˆ 1
= 2r3 drdθ = π.
0 0

7. (F) Let C be a simple closed smooth curve in the plane 2x + 2y + z = 2. Show that the
line integral ˛
2ydx + 3zdy − xdz
C
depends only on the area of the region enclosed by C on the above given plane and the
orientation of C, but not on the position or shape of C.

˛
Solution: The line integral can be written as F · dr where
C
F = 2yi + 3zj − xk.
By calculation, we get: ∇ × F = −3i + j − 2k. Let Σ be the region enclosed by C on
the plane 2x + 2y + z = 2, then the unit normal to the plane is given by:
2i + 2j + k
n̂ = √
9
Stokes’ Theorem asserts that:
˛ ¨
F · dr = ± (∇ × F) · n̂ dS (± depends on orientation of C)
C
¨Σ
2i + 2j + k
=± (−3i + j − 2k) · √ dS
Σ 9
¨
6
= ∓√ 1 dS = ∓2 × area of Σ
9 Σ

Page 7
MATH 2023 Surface Integrals, Stokes’ Theorem Problem Set #9
S
8. (FF) Consider the curve of intersection C of the plane y + z = 2 and the cylinder x2 +
y2 = 1, with orientation shown in the diagram below. The surface S is the planar region
V EXAMPLE 1 Evaluate xC F ⴢ dr, where F共x,
enclosed by C, and its projection onto the xy-plane is denoted by D.
curve of intersection of the plane y  z 苷 2 a
be counterclockwise when viewed from above
z
SOLUTION The curve C (an ellipse) is shown in F
evaluated directly, it’s easier to use Stokes’ Th


S C
y+z=2 i j
 
curl F 苷
x y
y 2 x

D 0 Although there are many surfaces with bounda


y elliptical region S in the plane y  z 苷 2 that
x then C has the induced positive orientation. Th
disk x 2  y 2 1 and so using Equation 16.7.
FIGURE 3write down a parametrization of S such that the
(a) Using a suitable coordinate system,
parameters have constant bounds. y F ⴢ dr 苷 yy curl F ⴢ dS 苷 yy
C
S D

Solution: Use cylindrical coordinates: 2 1


苷y y 共1  2r sin 
0 0


r(r, θ ) = (r cos θ )i + (r sin θ )j + (2 − r sin θ ) k
2 r2 r3
苷 y 2
| {z }
z =2− y sin
0 2 3
where 0 ≤ r ≤ 1 and 0 ≤ θ ≤ 2π. 苷 12 共2兲  0 苷 
˛
(b) Evaluate the line integral F · dr wherezF is given by: V EXAMPLE 2 Use Stokes’ Theorem to comp
C ≈+¥+z@ = 4 F共x, y, z兲 苷 xz i  yz j  xy k and S is the pa
lies inside the cylinder x 2  y 2 苷 1 and above
F( x, y, z) = −yS2 i + xj + z2 k
C SOLUTION To find the boundary curve C we solv
x 2  y 2 苷 1. Subtracting, we get z 2 苷 3 and s
0 circle given by the equations x 2  y 2 苷 1, z 苷
Solution: y
r共t兲 苷 cos t i  sin t j 
x ×F
∇ = ≈+¥=1
(1 + 2y )k
so r共t兲 苷 sin t i  cos t j
Using Stokes’ Theorem (note that F is defined and C1 everywhere in R3 ), we get:
FIGURE 4

˛ ¨ Also, we have
F · dr = (∇ × F) · n̂ dS F共r共t兲兲 苷 s3 cos t i  s
C S
ˆ 2π ˆ 1 Therefore,
by Stokes’ Theorem,
j + k ∂r ∂r
= (1 + 2y)k · √ × drdθ
0 0 2 ∂r ∂θ yy curl F ⴢ dS 苷 yC F ⴢ dr 苷 y
ˆ 2π ˆ 1
1 + 2r sin θ S
= √ |rj + rk| drdθ 2
0 0 2 苷 y (s3 co
ˆ 2π ˆ 1 0
1 + 2r sin θ √
= √ 2r drdθ 2
0 0 2 苷 s3 y 0 dt 苷
ˆ 2π ˆ 1 0

= (r + 2r2 sin θ ) drdθ


0 0

Page 8
MATH 2023 Surface Integrals, Stokes’ Theorem Problem Set #9

y x 1
(c) Let G( x, y, z) = − i+ 2 j+ k.
+y x2
2 x +y 2 z+1
i. Verify that ∇ × G = 0 at every point in the domain of G. Does this result
determine that G is conservative or not?

Solution: Showing ∇ × G = 0 is straight-forward. Note that the domain of


G is R3 removing the z-axis and the horizontal plane z = −1. It is not a
simply-connected domain. We cannot conclude that G is conservative or not
based the result ∇ × G = 0.

ii. Denote Γ to be the projection of C onto the xy-plane. Using the Stokes’ Theorem
in an appropriate way, show that:
˛ ˛
G · dr = G · dr.
C Γ

Solution: Similar to Worksheet #22, Q3.


˛
iii. Evaluate G · dr using the above result.
C

Solution: Γ can be parametrized by r(t) = ˛(cos t)i + (sin t)j + 0k, where
0 ≤ t ≤ 2π. We can compute the line integral G · dr directly.
Γ
˛ ˆ 2π  
y x 1
G · dr = − 2 i+ 2 j+ k · ((− sin t)i + (cos t)j) dt
Γ 0 x + y2 x + y2 z+1 | {z }
r0 (t)
ˆ 2π  
sin t cos t 1
= − i+ j+ k · (−(sin t)i + (cos t)j) dt
0 1 1 0+1
ˆ 2π
= sin2 t + cos2 t dt = 2π.
0

9. (F) Two of the four Maxwell’s Equations (Faraday’s and Ampère’s Laws) assert that:

1 ∂B
∇×E = −
c ∂t
∂E
∇ × B = µ0 J + µ0 ε 0
∂t
where E is the electric field, B is the magnetic field, J is the current, and c, µ0 and ε 0
are positive constants. Using Stokes’ Theorem, show that for any (stationary) simply-
connected orientable surface S with boundary C, we have:
˛ ¨
1 ∂
E · dr = − B · n̂ dS
c ∂t S
˛C ¨ ¨

B · dr = µ0 J · n̂ dS + µ0 ε 0 E · n̂ dS
C S ∂t S

[You don’t need to know any physics to do this problem.]

Page 9
MATH 2023 Surface Integrals, Stokes’ Theorem Problem Set #9

Solution:
¨ ¨
1 ∂ 1 ∂
− B · n̂ dS = − B · n̂ dS
c ∂t c S ∂t
S
¨
= (∇ × E) · n̂ dS (Given)
˛ S

= E · dr (Stokes’ Theorem)
C

Similarly,
¨ ¨ ¨  
∂ ∂E
µ0 J · n̂ dS + µ0 ε 0 E · n̂ dS = µ0 J + µ0 ε 0 · n̂ dS
∂t ∂t
S S
¨ S

= (∇ × B) · n̂ dS
˛ S

= B · dr
C

y
10. (FF) Let F( x, y, z) = 0, − 2z , 2 .

(a) Show that ∇ × F = i.

Solution: Straight-forward

(b) Find vector fields G and H such that ∇ × G = j and ∇ × H = k.

Solution: Mimic the vector field F in part (a), and some trial-and-errors:
Dz xE
∇ × , 0, − =j
D2 y x 2E
∇ × − , ,0 = k
2 2
y
Let G = 2 , 0, − 2x and H = − 2 , 2x , 0 .

z

[Note that the problem only requires us to find one such G and one such H. In
fact it can be shown that all other possible G’s are given by:
Dz xE
, 0, − + ∇f
2 2
where f is any C2 scalar functions defined everywhere in R3 . Similar for H.]

Page 10
MATH 2023 Surface Integrals, Stokes’ Theorem Problem Set #9

(c) Let C be an arbitrary simple closed curve on the xy-plane in the three dimensional
space, and S is any surface above the xy-plane with boundary curve C. See the figure
below.

Show that:
¨
( ai + bj + ck) · n̂ dS = c × area of the region on the xy-plane enclosed by C.
S

Here a, b and c are all constants.

Solution: Using (a) and (b), we get:

ai + bj + ck = a(∇ × F) + b(∇ × G) + c(∇ × H) = ∇ × ( aF + bG + cH).

Using Stokes’ Theorem, we get:


¨ ¨
( ai + bj + ck) · n̂ dS = (∇ × ( aF + bG + cH)) · n̂ dS
S
˛S
= ( aF + bG + cH) · dr (Stokes’)
¨C
= (∇ × ( aF + bG + cH)) · k dA (Green’s)
¨R
= ( ai + bj + ck) · k dA
¨ R

= c dA = c × area of R
R

(d) Using the results of (a), (b), and the Stokes’ Theorem, redo Problems #1(d) and #2(c).

y
Solution: For #1(d), use the fact that ∇ × 0, − 2z , 2 = i and Stokes’ Theorem:

¨ ¨ D z yE
i · n̂ dS = (∇ × 0, − , ) · n̂ dS
2 2
cone
˛ cone D z yE
= 0, − , · dr (Stokes’)
boundary circle 2 2

The boundary circle of the cone is parametrized by:

r(t) = ( R cos t)i + ( R sin t)j + hk, 0 ≤ t ≤ 2π.

Page 11
MATH 2023 Surface Integrals, Stokes’ Theorem Problem Set #9

Therefore,
˛ ˆ 2π  
D z yE h R
0, − , · dr = 0, − , sin t · h− R sin t, R cos t, 0i dt
boundary circle 2 2 0 2 2
ˆ 2π
hR
= − cos t dt = 0.
0 2

#2(c) is to calculate the



yflux of
k through the surface torus. From part (c) we
know that k = ∇ × − 2 , 2x , 0 . Therefore, k is a solenoidal vector field. From
Worksheet #22 Q2 we can use the Stokes’ Theorem (after cutting the torus along
two circles so that it becomes simply-connected) to conclude that the surface flux
is zero.

Optional – about the Gauss-Bonnet’s Theorem

11. Given a oriented surface S with parametrization r(u, v), we denote:

∂r ∂r ∂r ∂r ∂r ∂r
E= · F= · G= ·
∂u ∂u ∂u ∂v ∂v ∂v
∂2 r ∂2 r ∂2 r
e = 2 · n̂ f = · n̂ g = 2 · n̂
∂u ∂u∂v ∂v

The Gauss curvature at the point r(u, v) is defined to be:

eg − f 2
K (u, v) := .
EG − F2

The geometric intuition behind the Gauss curvature may be covered in MATH 4223. In
Differential Geometry, there is a beautiful theorem – the Gauss-Bonnet’s Theorem – which
asserts that if S is closed, oriented and smooth (without corners), then:

K dS = 4π (1 − number of holes of S)
S

Therefore, if S is a sphere, the above surface integral should be 4π as there is no hole.


If S is a torus (which has one hole), the above surface integral should be 0. Verify this
theorem for the sphere and torus, by parametrizing them and compute the above integral
directly over the sphere and the torus.
As an optional problem, you may use computer softwares to ease your calculations.

Page 12
MATH 2023 • Spring 2015-16 • Multivariable Calculus
Problem Set #10 • Divergence Theorem

1. (F) Use the Divergence Theorem to find the outward flux F · n̂out dS for each of the
S
following F and S:
(a) F = xi + yj + zk and S is the surface of any square cube of length b.

Solution: It is easy to see that ∇ · F = 3. The solid D enclosed by S is the solid


square cube of length b. Divergence Theorem shows:
‹ ˚
F · n̂out dS = ∇ · F dV
S
˚ D

= 3 dV = 3 × volume of D
D
= 3b2 .

(b) F = x3 i + 3yz2 j + (3y2 z + x2 )k and S is the sphere with radius a > 0 centered at the
origin.

Solution: ∇ · F = 3( x2 + y2 + z2 ) = 3ρ2 . The solid D enclosed by S is the solid


sphere with radius a centered at the origin, i.e. D = {ρ ≤ a}.
‹ ˚
F · n̂ dS = ∇ · F dV
S D
ˆ 2π ˆ π ˆ a
= 3ρ2 · ρ2 sin φ dρdφdθ
0 0 0
12πa 5
= .
5

(c) F = x2 i + y2 j + z2 k and S is the boundary surface of the cylinder D defined by


x2 + y2 ≤ 1 and 0 ≤ z ≤ 4.

Solution: ∇ · F = 2( x + y + z). The solid D is described by inequalities 0 ≤ r ≤ 1,


0 ≤ θ ≤ 2π and 0 ≤ z ≤ 4 in cylindrical coordinates:

‹ ˆ 2π ˆ 1ˆ 4
F · n̂ dS = ∇ · F rdzdrdθ
S 0 0 0
ˆ 2π ˆ 1ˆ 4
= 2(r cos θ + r sin θ + z) rdzdrdθ
0 0 0
= 16π

Remark: To simplify the computations, it is good to keep in mind that:


ˆ 2π ˆ 2π
cos θ dθ = sin θ dθ = 0.
0 0

Page 1
V EXAMPLE 1 Find the flux of the vector field F共
sphere x 2 ⫹ y 2 ⫹ z 2 苷 1.
MATH 2023 Divergence Theorem SOLUTION FirstProblem Setthe
we compute #10
divergence of F :
‹ ⭸ ⭸
div F 苷 共z兲 ⫹ 共y兲
2. (F) Evaluate F · n̂out dS where ⭸x ⭸y
S
The unit sphere S is the boundary of the unit ball B
 2

F = xyi + y2 + e xz j + sin( xy)k Divergence Theorem gives the flux as

The solution in Example 1 should be


and S is the surface boundarycompared
of thewithregion
N
D defined by z ≤ 1 − x2 , z ≥ 0, y ≥ 0 and
y ≤ 2 − z. See the figure below:
in Section 16.7.
the solution in Example 4 yy F ⴢ dS 苷 yyy div F dV 苷 yyy 1 dV
S B B

z V EXAMPLE 2 Evaluate yy F ⴢ dS, where


S
(0, 0, 1)
y=2-z
F共x, y, z兲 苷 xy i ⫹ ( y 2 ⫹ e
ER 16 VECTOR CALCULUS and S is the surface of the region E bounded by th
0
the planes z 苷 0, y 苷 0, and y ⫹ z 苷 2. (See Figu
puter to graph the surface S and the tangent Evaluate xC F ⴢ dr, where C is the curve with initial point
SOLUTION It would be extremely difficult to evaluate
d in part (a). 共0, 0, 2兲 and terminal point 共0, 3, 0兲 shown
(1, 0, 0) in ythe figure.
(0, 2, 0)
(We would have to evaluate four surface integrals
do not evaluate, an integral for the surface area x z Furthermore, the divergence of F is much less com
z=1-≈
(0, 0, 2)
FIGURE 2 ⭸ ⭸
z兲 苷
z2 x2 y2 共xy兲 ⫹ ( y 2 ⫹ e xz 2 ) ⫹ div F 苷
2 i ⫹ j⫹
2Comment k ⭸x ⭸y
1⫹x 1⫹y 1 ⫹ z 2 on why it is preferable to use the Divergence Theorem instead of computing
0
the surface flux directly.
dS correct to four decimal places. (0, 3, 0)
Therefore we use the Divergence Theorem to trans
(1, 1, 0)
y triple integral. The easiest way to evaluate the trip
e surface integral. (3, 0, 0) region:
Solution:
e S is the part of the paraboloid z 苷 x ⫹ y
2 2
x
the plane z 苷 4 ∇ · F = 3y E 苷 兵 共x, y, z兲 ⫺1 艋 x 艋 1, 0 艋 z 艋

38. Let
兲 dS, where S is the part of the plane ‹ ˚
y that lies inside the cylinder x 2 ⫹ y 2 苷 4 共2 x 3 ⫹ 2 x y 2 ⫺ 2y兲 i ⫹ 共2y 3 ⫹ 2 x 2 y ⫹ 2 x兲 j
F · n̂y兲dS
F共x, 苷 = ∇ · F dV 2
S D x ⫹ y2
ere F共x, y, z兲 苷 x z i ⫺ 2y j ⫹ 3x k and S is ˆ z=is1− ˆ 2−zfigure. 2 ˆ
⫹ y 2 ⫹ z 2 苷 4 with outward orientation Evaluate x䊊C F ⴢ drx, = 1
where C x
shown yin=the
= 3y dydzdx
x =−1 zy=0 y =0
ere F共x, y, z兲 苷 x i ⫹ x y j ⫹ z k and S is the
2

aboloid z 苷 x 2 ⫹ y 2 below the plane z 苷 1 184 C


rientation
=
35
Easier to use Divergence Theorem as the surface S has 4 faces. To compute the surface
0 x
kes’ Theorem is true for the flux
vectordirectly
field we would need to split the surface flux into 4 parts and parametrize them
2
i ⫹ y 2 j ⫹ z 2 k, where S is individually.
the part of the
苷 1 ⫺ x 2 ⫺ y 2 that lies above the xy-plane and
orientation.
heorem to evaluate xxS 3. F ⴢ)dS
curl(F , where
Let D be the solid
39. square
Find xxS Fcube
ⴢ n dS,of length
where 2 z兲with
F共x, y, 苷 x ione
⫹ y jcorner unit
⫹ z k and S is cube removed. See
2
yz i ⫹ yz 2 j ⫹ z 3e xy k, S is the part of the
the figure below. the outwardly oriented surface shown in the figure (the
2
⫹ z 2 苷 5 that lies above the plane z 苷 1, and boundary surface of a cube with a unit corner cube removed).
pward.
z
heorem to evaluate xC F ⴢ dr, where
y i ⫹ yz j ⫹ z x k, and C is the triangle with
0兲, 共0, 1, 0兲, and 共0, 0, 1兲, oriented counter- (0, 2, 2)
iewed from above. (2, 0, 2)

gence Theorem to calculate the surface integral 1


ere F共x, y, z兲 苷 x 3 i ⫹ y 3 j ⫹ z 3 k and S is the
solid bounded by the cylinder x 2 ⫹ y 2 苷 1 and 1
1
0 and z 苷 2. y
Divergence Theorem is true for the vector x S (2, 2, 0)
苷 x i ⫹ y j ⫹ z k, where E is the unit ball
艋 1.
40. If the components of F have continuous second partial deriva-
utward flux of tives and S is the boundary surface of a simple solid region,
show that xxS curl F ⴢ dS 苷 0.
xi⫹yj⫹zk Page 2
F共x, y, z兲 苷
共x 2 ⫹ y 2 ⫹ z 2 兲 3兾2 41. If a is a constant vector, r 苷 x i ⫹ y j ⫹ z k, and S is an ori-
ented, smooth surface with a simple, closed, smooth,
ipsoid 4 x 2 ⫹ 9y 2 ⫹ 6z 2 苷 36. positively oriented boundary curve C, show that
MATH 2023 Divergence Theorem Problem Set #10

Evaluate the outward flux F · n̂out dS where F = xi + yj + zk. Comment on why it is
S
preferable to use the Divergence Theorem instead of computing the flux directly.

Solution:
∇·F = 3
‹ ˚
F · n̂ dS = ∇ · F dV
S
˚D
= 3 dV = 3 × volume of D
D
= 3(2 − 13 ) = 21.
3

The surface S has 9 faces!!! Without the Divergence Theorem, we will need to compute
the surface flux by split it into 9 parts!

4. (FF) Let C be an arbitrary simple closed curve on the xy-plane in the three dimensional
space, and S is any surface above the xy-plane with boundary curve C. See the figure
below.

Using the Divergence Theorem, show that:


¨
( ai + bj + ck) · n̂ dS = c × area of the region on the xy-plane enclosed by C.
S

Here a, b and c are all constants.

Solution:
∂a ∂b ∂c
∇ · ( ai + bj + ck) = + + = 0 + 0 + 0 = 0.
∂x ∂y ∂z
However, note that S is not a closed surface, but S ∪ R is closed. Apply the Divergence
Theorem on S ∪ R instead:
‹ ˚
( ai + bj + ck) · n̂ dS = ∇
| {z· F} dV = 0.
S∪ R solid enclosed
0
¨ ¨ ˚
( ai + bj + ck) · n̂ dS = ( ai + bj + ck) · (−k) dS = − c dS = −c × area( R)
R R R

Page 3
MATH 2023 Divergence Theorem Problem Set #10

Since:
¨ ¨ ¨
( ai + bj + ck) · n̂ dS + ( ai + bj + ck) · n̂ dS = ( ai + bj + ck) · n̂ dS = 0
S R S∪ R

we conclude that:
¨ ¨
( ai + bj + ck) · n̂ dS = − ( ai + bj + ck) · n̂ dS = c × area of R.
S R

5. (FF) Suppose f ( x, y, z) is a C2 function on R3 such that ∇2 f ( x, y, z) = 0 on R3 . Here


∇2 f means the Laplacian of f , i.e. ∇2 f = ∇ · ∇ f = f xx + f yy + f zz .
(a) Show that: ‹ ˚
f ∇ f · n̂ dS = |∇ f |2 dV
S D
for any closed oriented surface S enclosing the solid region D.

Solution:
‹ ˚
f ∇ f · n̂ dS = ∇ · ( f ∇ f ) dV
S
˚D
= (∇ f · ∇ f ) + f ∇ · ∇ f dV
˚D
= |∇ f |2 + f ∇2 f dV
D |{z}
=0
˚
= |∇ f |2 dV
D

(b) If, furthermore, assume that f ( x, y, z) = 0 for any ( x, y, z) on S, what can you say
about f ( x, y, z) for any ( x, y, z) in D?

Solution: If f = 0 on S, then the surface integral:


‹ ‹
f ∇ f · n̂ dS = 0 ∇ f · n̂ dS = 0.
S S

Then from (a), we get: ˚


|∇ f |2 dV = 0
D

Since |∇ f |2 ≥ 0, the only chance that the above integral is zero is that ∇ f = 0
at every point in D. This means f is a constant function is D. By continuity, this
constant must match with the value of f on the boundary S, hence f ≡ 0 in D.

Page 4
MATH 2023 Divergence Theorem Problem Set #10

6. (FF) Suppose S is a closed oriented level surface f ( x, y, z) = c of a C2 function f . Denote


D to be the solid enclosed by S. Show that:
‹ ˚
|∇ f | dS = ± ∇2 f dV
S D

where ± depends on whether ∇ f points inward or outward on the surface S.

∇f
Solution: Note that S is the level surface f = c. Hence n̂ = ± |∇ f | .
˚ ˚
2
∇ f dV = ∇ · ∇ f dV
D
‹ D

= ∇ f · n̂ dS
S

∇f
=± ∇f · dS
S |∇ f |
‹ ‹
|∇ f |2
=± dS = ± |∇ f | dS
S |∇ f | S

7. (FF) Given two C2 functions u( x, y, z) and v( x, y, z) defined on R3 . Let S be a closed


oriented surface and D is the solid enclosed by S.
(a) Rewrite ∇ · (u∇v − v∇u) using curl, grad and div.

Solution:
div(u grad(v) − v grad(u)).

(b) Show that ‹ ˚


u∇2 v − v∇2 u dV

(u∇v − v∇u) · n̂ dS =
S D

Solution:
‹ ˚
(u∇v − v∇u) · n̂ dS = ∇ · (u∇v − v∇u) dV
S
˚D
= (∇u · ∇v + u∇ · ∇v − ∇v · ∇u − v∇ · ∇u) dV
˚D
= (u∇2 v − v∇2 u) dV
D

(c) Assume further that ∇u( x, y, z) · n̂ = ∇v( x, y, z) · n̂ = 0 for any ( x, y, z) on S, show


that ˚ ˚
u∇2 v dV = v∇2 u dV.
D D

Solution: Simply apply the result of (b) using the given conditions that ∇u · n̂ =
∇v · n̂ = 0 on S.

Page 5

S-ar putea să vă placă și